You are on page 1of 174

Contents

POLITY & GOVERNANCE  1


UPI QR Code-CBDC Interoperability............................... 64
Women’s Reservation Act 2023 ....................................... 1
India Post Payments Bank (IPPB)...................................... 65
One Nation, One Election.................................................... 4
Self-Regulatory Organisation (SRO) For Fintechs............66
India-Bharat Controversy..................................................... 6
RBI Removes Incremental CRR..........................................68
Special Session of Parliament............................................. 9
World Bank’s Online Gig Work Report............................69
National Judicial Data Grid (NJDG)................................ 11
Fugitive Economic Offender.............................................. 71
Mercy Petition..................................................................... 13
Crypto Regulation Discussion at G20............................... 71
Section 6A of DSPE Act as Unconstitutional....................14
Gresham’s Law .................................................................. 72
Justice Amitava Roy Committee Report.......................... 16
ENVIRONMENT & GEOGRAPHY 73
Constitution Benches.......................................................... 17
Global Biofuels Alliance......................................................73
Post Office Bill, 2023........................................................... 19
One Year of Project Cheetah...............................................75
Autonomous District Council............................................ 20
Climate Ambition Summit..................................................76
Laïcité: The French Secularism...........................................22
Planetary Boundaries...........................................................76
Immunity of Legislators from Bribery Charges...............24
Pollution Tax.........................................................................79
INTERNATIONAL RELATIONS 25 Laws Governing Forests of the Northeast........................80
G20 @ India........................................................................... 25
Designation of National Disasters Tag..............................82
India-Middle East-Europe Mega Economic Corridor.....33
Atlantification of the Arctic.................................................83
20th ASEAN-India Summit................................................ 36
Heat Index.............................................................................85
Indus Water Treaty (IWT)...................................................39
Mission To Clean Ganga.................................................... 86
India- Saudi Arabia Strategic Partnership Council..........41
Global Stocktake Report..................................................... 88
International Solar Alliance............................................... 44
UNEP’s Report on Wastewater..........................................89
China’s U-Shaped Line........................................................46
World Ozone Day.................................................................91
Khalistan Movement............................................................47
Green Nudges.......................................................................91
Five Eyes Alliance................................................................ 50
Rafflesia................................................................................. 92
International Organisation of Legal...................................50
Species in News.....................................................................93
Metrology Certificates..........................................................
Wollemi Pine..........................................................................93
Treaty on Traditional Knowledge......................................51
Bhoj Wetland.........................................................................94
Asia Pacific Forum on Human Rights................................52
Campbell Bay National Park.............................................. 94
United Nations Commission on International 
53
Trade Law (UNCITRAL).................................................... SCIENCE & TECHNOLOGY 96
Digital Personal Data Protection Act, 2023...................... 96
ECONOMY54
GM Crops in India..............................................................100
Unemployment Measurement.......................................... 54
Nipah Virus........................................................................ 102
National Logistics Policy (NLP).........................................55
Duchenne Muscular Dystrophy........................................103
Pearl Millet Cultivation.......................................................57
Preeclampsia.......................................................................104
India’s Falling Cotton Production......................................58
Monoclonal Antibody........................................................104
Bumper Crop and Rising Cereal Prices.............................61
Global Impact of High BP................................................. 104
Food Insecurity in India.......................................................62
Human Embryo Model Without Sperm or Egg............ 106
Payment System Operator..................................................63
First Cloned Mammal....................................................... 107

 I
Dark Patterns..................................................................... 110 SNAP NEWS 148
India’s First Largest Indigenously Dual-Use Items....................................................................148
111
Developed Nuclear Power Plant......................................
Mangri Orang......................................................................148
Hybrid Nanoparticles to Cure Cancer............................ 112
CE-20 Cryogenic Engine................................................... 149
New Science Awards..........................................................113
Bharat Drone Shakti 2023..................................................149
Shanti Swarup Bhatnagar Awards................................. 115 Kunbis..................................................................................149
DEFENCE & SECURITY 116 Investor Education and Protection Fund Authority.....150
Exercise SIMBEX 23........................................................... 116 Vanadium Discovery in Gujarat.......................................150
Exercise Varuna 2023.........................................................116 Pirola.....................................................................................151

India-Indonesia-Australia Trilateral Kumkis..................................................................................151


Maritime Exercise................................................................116 Amigurumi Dolls................................................................152
Operation Sajag...................................................................117 The Organized Crime and Corruption
Reporting Project................................................................152
SOCIAL ISSUES 118 Notice To Airmen (NOTAM)........................................... 153
International Literacy Day.................................................118
Gujarat Declaration (WHO)..............................................153
Diksha E-Education Platform to Offer AI Help..............120
Ancient Red Sea Trade Route...........................................154
Vidya Sameeksha Kendras................................................121
G-20 Satellite Mission for Environment
Bima Sugam........................................................................ 123 and Climate Observation.................................................. 154

HISTORY & CULTURE  125 Brandt Line..........................................................................154


New Parliament Building..................................................125 Bhojshala and Kamal Maula’s Mosque...........................155
Old Parliament Building....................................................126 Nairobi Declaration........................................................... 155
Chausath Yogini Temple....................................................127 Divya Kalamela................................................................. 156
UNESCO World Heritage Sites in India..........................127 Vagus Nerve...................................................................... 156
Adi Shankaracharya (788-820 AD)...................................130 Ramon Magsaysay Award............................................... 157
Dr. Sarvepalli Radhakrishnan (1888-1975).................... 130 Asian Games...................................................................... 157
Sir M Visvesvaraya (1861-1962)....................................... 131 Asia Cup............................................................................. 158
Hindi Diwas........................................................................132 Places in News................................................................... 159
Kokborok Language......................................................... 132 Article From Yojana & Kurukshetra
161
Lingaraj Temple................................................................. 133 (September 2023)
105th Anniversary of the Battle of Haifa......................... 135 Accountability And Financial
Mitāksarā Law.................................................................... 135 Administration [From Yojana]..........................................161
Operation Polo....................................................................136 Make In India: Challenges, Opportunities
Project to Reconstruct an Ancient ‘Stitched Ship’..........138 and Outcomes [From Kurukshetra]................................ 162

Festivals in News................................................................139 Drainage Systems of India 166


World’s Most Isolated Tribes and Their Habitats......... 140 The Indus River System.................................................... 166
GOVERNMENT SCHEMES 141 Ganga River System...........................................................167

Expansion of Ujjwala Yojana............................................141 Yamuna River System....................................................... 168

Pradhan Mantri Matsya Sampada Yojana (PMMSY)....142 Brahmaputra River System...............................................169


Ayushman Bhav Health Scheme......................................143 Godavari River System..................................................... 169
Adopt a Heritage 2.0 Programme....................................145 Krishna River System........................................................ 169
Shreyas Scheme...................................................................146 Narmada River System..................................................... 170
PM Dakshata Aur Kushalata Sampanna  Mahanadi River System................................................... 170
147
Hitgrahi (PM-DAKSH) Yojana.......................................... Cauvery River System...................................................... 170

II 
SECTION

A POLITY & GOVERNANCE

WOMEN’S RESERVATION ACT 2023


Why in News: Recently, the Nari Shakti Vandan Adhiniyam
Related Constitutional Provisions
became law in India after the Indian President gave her
Article 15 (3): The State is empowered to make “special
 
nod to the women’s reservation bill (106th Constitutional
provisions”, legislative or otherwise, to secure women’s
Amendment Act).
socio-political advancement.
• This had been pending for 27 years
Article 325: Guarantees equal rights for both sexes, and
 
entitles women to enjoy economic, social, cultural, and
political rights on an equal footing with men.
Articles 330A and 332A newly proposed articles want
 
to establish women’s reservation in the Lok Sabha and
state legislative assemblies.
Article 334A included a sunset provision that would
 
gradually end this affirmative action policy after 15
years.
Article 82 and Article 170(3) of the Constitution would
 
need to be changed in order to implement delimitation,
which is a requirement for the implementation of
reservations.
Article 243D of the Constitution has a provision for
 
the reservation of seats for SCs, STs, and women in
Panchayats.

Rationale behind Women’s Reservation Act


• Under representation of women: Currently, just 14 per
cent of Lok Sabha MPs are women (78 in total). And
women make roughly 11 per cent of the Rajya Sabha
Understanding the Women’s Reservation Act 2023
MPs.
• As per this act, 33% of seats in the Lok Sabha, state
legislative assemblies and in assemblies of UTs with • Global comparison: Although the number of women
legislature, will be reserved for women. MPs has increased significantly since the first Lok Sabha,
¾ However, the reservation will be implemented after it remains much lower in comparison to several other
the new census and delimitation. countries.
• Additionally, it also provides for reservations for ¾ According to data from PRS, Rwanda (61 per cent),
Scheduled Castes (SCs), Scheduled Tribes (STs) within South Africa (43 per cent) and even Bangladesh (21
the 33% quota– horizontal reservation. per cent), are ahead of India in this matter.
• The Act provides that the reservation will continue for 15 • India ranks 144 out of 193 countries in the representation
years. Seats reserved for women will be rotated after each of women in Parliament, according to the Inter-
delimitation exercise. Parliamentary Union’s latest report.
• The quota will not apply to the Rajya Sabha or State
• Need to bridge the gender gap: According to various
Legislative Councils.
international reports, development in India is being

POLITY & GOVERNANCE  1


severely hampered by the breadth of the gender gap • Traditional constraints: Patriarchal values, lack of
and limited female participation in traditionally male education and low economic empowerment hinder the
dominated institutions and social strata. participation of women in political arena.

• Gender sensitization in law making: As a study by the • Social norms: Lack of education and awareness hinder
American Economic Association showed, “countries women’s involvement in active politics.
with higher share of women in national parliament are Significance
more likely to pass and implement gender sensitive
• This act holds immense significance as it addresses the
laws.”
gender disparity in Indian politics
• Positive impacts: A 2010 study by the Harvard • It provides a platform for women to participate actively
Kennedy School showed that female representation in in decision-making processes.
village councils increased female participation and • This act will encourage women participation in politics
responsiveness to concerns such as drinking water, and would take democracy to its real protectors, i.e., the
infrastructure, sanitation and roads. common women and men of the country.

2 POLITY & GOVERNANCE


Will this lead to dramatic increase in number of women in
Parliament and State Assemblies?

• The present Lok Sabha has a total of 543 members, out of


which 78 (14.39 per cent) are female members.

• The average number of women MLAs in Assemblies


across the nation accounts for only 8 per cent.

• Now, both the Lok Sabha and States/UTs assemblies will


see rise in female members, but the rise will be more
dramatic in States/UTs.

Women’s Participation in Indian Politics


1. Current Representation in the Lok Sabha

Conclusion

• The Women’s Reservation Act is a historic and


transformative legislation that seeks to empower women
in Indian politics.

• Its journey through various governments reflects both


2. India’s Global Ranking- According to the International the progress made and the obstacles encountered in
Parliamentary Union (IPU), India ranks 141st in 185 achieving greater female representation in legislative
countries in women representation. bodies.

• This act could pave the way for significant advancements


in women’s participation in the governance of the
country.

• It stands as a testament to the evolving conversation


on gender equality and the determination of various
stakeholders to address this critical issue in Indian society
and politics.

Way Ahead:

• Political empowerment of women is rightly perceived


as a powerful and indispensable tool for eliminating
gender inequality and discrimination.

• Alternate methods such as reservation in political parties


and dual member constituency can also be explored as
a way to democratize [introduce democratic principles]
political participation by various sections.

POLITY & GOVERNANCE  3


• The idea is to hold these elections simultaneously, either
EXPECTED QUESTION FOR UPSC CSE
on a single day or within a specific time frame.

Prelims MCQ • The idea revolves around decreasing the frequency of


elections and simplifying the electoral process.
Q. With reference to the ‘Nari Shakti Vandan Adhiniyam
2023,’ consider the following statements: • This will involve the restructuring of the Indian election
cycle in a manner that elections to the states and the
centre synchronise.
1. It reserves one-third of seats for women in
panchayats and municipalities. What is the background?
• Simultaneous elections were the norm since
2. It reserves one-third of all seats for women in the independence until 1967.
State Legislative Assemblies.
• But following dissolution of some Legislative Assemblies
3. Under it, the reservation will be implemented from in 1968 and 1969 and that of the Lok Sabha in December
1st January 2024. 1970, elections to State Assemblies and Parliament have
been held separately.
How many of the above statements are correct?
• The idea of reverting to simultaneous polls was
(a) Only one     (b) Only two mooted in the annual report of the Election Commission
in 1983.
(c) All three    (d) None • In 1999, the Law Commission’s Report also referred to
Answer: b (2 and 3 only)
it.

• The recent push came ahead of the 2014 Lok Sabha


Descriptive Question
polls.
Q. Discuss the main parameters to assess the state of
women’s participation in politics in India. Also Law Commission 2018 Draft Report
highlight the challenges in Women’s Representation
in Parliament and State Legislatures. [15 Marks] [250 • In 2018, the Law Commission of India under
Words]
Justice BS Chauhan said simultaneous elections
cannot be held within the existing framework
For Approach Answer, Scan:
of the Constitution.168It suggested at least “five
Constitutional recommendations” for holding
simultaneous polls.

¾ Article 83 (2): It says the Lok Sabha’s term should not


exceed five years but it may be dissolved sooner.

¾ Article 85 (2) (B): A dissolution ends the very life of


the existing House and a new House is constituted
after general elections.
ONE NATION, ONE ELECTION
¾ Article 172 (1): A state assembly, unless sooner
Why in news: The government has constituted a committee dissolved, shall continue for five years
headed by former President of India, Ram Nath Kovind to
explore the possibility of “one nation, one election”. ¾ Article 174 (2) (B): The Governor has the power to
dissolve the assembly on the aid and advice of the
What are simultaneous polls and One Nation, One Election? cabinet. The Governor can apply his mind when
• “Simultaneous polls” or “simultaneous elections” refer the advice comes from a Chief Minister whose
to the practice of holding multiple elections for different majority is in doubt.
levels of government at the same time, typically on a
fixed schedule. ¾ Article 356: Imposition of President’s Rule in
states.
• The concept of “One Nation, One Election” in India aims
to synchronise elections for the Lok Sabha (the lower
house of India’s Parliament) and all state assemblies.

4 POLITY & GOVERNANCE


POLITY & GOVERNANCE  5
Current Scenario: Presently, only specific States such as prospective successor. E.g. this method is followed in
Arunachal Pradesh, Sikkim, Andhra Pradesh, and Odisha Germany.
conduct their assembly polls concurrently with the Lok • It must be ensured that spirit of federalism, which is a
Sabha elections. basic structure of Indian Constitution, must be upheld
even in case of simultaneous elections of state assemblies
Measures to be undertaken before implementing ONOE and Parliament.
• Consensus Building: Consensus among political parties Conclusion
is essential. ONOE should not be imposed unilaterally
It’s important to note that the “One Nation, One Election”
but should be the result of a consensus-driven approach.
concept is a subject of ongoing debate and discussion in
Establishing a bipartisan or multi-party committee to
discuss and deliberate on the issue can be a starting India, and opinions on its feasibility and desirability vary
point. widely among political parties, experts, and the public. The
decision to implement such a system would require careful
• Legal Framework: Develop a comprehensive legal and consideration of these arguments and a thorough evaluation
constitutional framework for ONOE. This may involve of its potential impact on India’s democratic framework and
amending the Constitution and various electoral laws to
governance.
accommodate simultaneous elections. Legal experts and
constitutional scholars should be consulted during this
process.
EXPECTED QUESTIONS FOR UPSC CSE

• Pilot Projects: Consider conducting pilot projects in a Prelims MCQ


few states or regions to test the feasibility and practicality
Q. 
The committee headed by the former president
of ONOE. These pilot projects can help identify Ramnath Kovind is associated with which of the
challenges and solutions before implementing it following?
nationally.
(a) One nation, one election
• Synchronization of Electoral Rolls: Ensure that electoral
rolls are synchronized across all levels of government. (b) Uniform Civil Code (UCC)
This includes harmonizing voter registration and (c) Reservation to women in Parliament of India
identification processes to avoid duplication.
(d) Schedules Tribes of the Northeast India
• Election Commission’s Role: Define the role and Answer: a
responsibilities of the Election Commission of India (ECI)
in overseeing and conducting simultaneous elections. Descriptive Question
The ECI will need to adapt its processes and procedures
Q. 
Simultaneous elections are bad for India’s
accordingly.
democracy”. Discuss [10 Marks] [150 Marks]
• Educating Voters: Launch a nationwide voter education For Approach Answer, Scan:
campaign to inform citizens about the changes in the
electoral schedule and the importance of participating in
all elections.

• International Best Practices: Study international


best practices and experiences of countries that have
implemented similar systems to learn from their successes
and challenges.

• Election Infrastructure: Upgrade and modernize election


infrastructure, including electronic voting machines INDIA- BHARAT CONTROVERSY
(EVMs) and voter registration systems, to handle the
increased workload of simultaneous elections. Why in News: Opposition politicians have shared
photographs of an official invitation to a G20 dinner hosted
Way Forward by “The President of Bharat,” rather than the normal
• Hybrid Accountability Model: Explore a hybrid “President of India.”
approach that retains staggered elections for select states
while implementing ONOE for others. There is conjecture that the country’s name would be changed
from India to Bharat, despite the fact that Article 1 of the
• Introduce a constructive vote of no confidence, which
Constitution uses both terms interchangeably: India, that is
allows the legislature to withdraw support from the
government only if there is positive majority for a Bharat, shall be a Union of States.

6 POLITY & GOVERNANCE


How did “Bharat” and “India” come into the Indian
Constitution?

• Nehru used the terms India, Bharata, and Hindustan in


his book “Discovery of India,” but when it came time to
name India in the Constitution, “Hindustan” was left out
but “Bharat” and “India” were kept.
• On September 17, 1949, the Name and territory of the Bharat:
Union was brought up for consideration during the • Bharata, “Bharat”, or “Bharatvarsha” has its roots in
Constituent Assembly deliberations. Puranic literature and the epic Mahabharata.
• As soon as the words “India, that is Bharat, shall be • Bharata is described as the land between the sea and
a Union of States” were read aloud in Article 1, a rift snow in the Puranas.
developed among the members.
• Social scientists view Bharata as a religious and socio-
• Many members opposed the usage of the term “India,” cultural entity, rather than a political or geographical one.
which they believed to be a reminder of the country’s
• It refers to the subcontinental territory where the
colonial past.
Brahmanical system prevails.
• The first article of the constitution, according to HV
• Bharata was also the name of an ancient king who was
Kamath, should read, “Bharat, or in the English language,
the ancestor of the Rig Vedic tribe of the Bharatas and the
India, shall be.”
progenitor [ancestor] of all the people in the subcontinent.
• The United Provinces’ hill districts’ representative,
Hargovind Pant, stated unequivocally that the people of Hindustan:
Northern India “wanted Bharatvarsha and nothing else.” • The word Hindustan is assumed to stem from ‘Hindu,’
• On September 18, 1949, the Constituent Assembly the Persian equivalent form of the Sanskrit ‘Sindhu’
approved the text of Article 1 of the Constitution, which (Indus), which became popular following the Persian
stated that “India, that is Bharat, shall be a Union of conquest of the Indus valley in the sixth century BC.
States.” • The word ‘Hindustan’ was used to denote the entire
Indo-Gangetic plain at the time of the early Mughals
Recently, Countries That Have Changed Their Names (16th century).
 Ceylon to Sri Lanka (2011) • In the mid-to-late eighteenth century, Hindustan
frequently referred to the Mughal emperor’s dominion,
 Czech Republic to Czechia (2016) which included much of South Asia.
 Swaziland to Eswatini (2018) India:
• The Greeks replaced ‘Sindhu/Hindu’ with ‘Indus’, and
 Macedonia to North Macedonia (2019)
by the time Macedonian King Alexander invaded India
 Holland to The Netherlands (2020) in the 3rd century BC, ‘India’ was associated with the
region beyond the Indus.
 Turkey to Turkiye (2022) • British maps began using ‘India’ in the late 18th century,
and ‘Hindustan’ lost its association with South Asia.

POLITY & GOVERNANCE  7


• The term’s appeal may have been due to its Greco-Roman associations, European history, and adoption by scientific
organizations like the Survey of India.

8 POLITY & GOVERNANCE


Why India has two names?
EXPECTED QUESTION FOR UPSC CSE
• India has two names, “India” and “Bharat,” due to
historical and linguistic reasons. Prelims MCQ

• The name “India” comes from the Sanskrit word Q. Consider the following statements:
“Sindhu,” meaning the Indus River, and was used by
1. Both Bharat and India are the constitutionally
Western civilizations to refer to the Indus Valley region.
valid names for the territory of India as a nation.
• India is a linguistically diverse country with thousands 2. 
The name of the country mentioned in the
of languages and dialects, and “Bharat” is associated Constitution of India is considered an integral
with India’s rich cultural and historical heritage. part of the Constitution’s basic structure.
• Both names are constitutionally recognized and hold Which of the statements given above is/are correct?
significance for different aspects of India’s identity. (a) 1 only       (b) 2 only

Were there any recent attempts to change the name of our (c) Both 1 and 2      (d) Neither 1 nor 2
Country? Answer: (a)

• In 2020, the Supreme Court denied a PIL seeking to Q2. The word ‘Hindu’ as reference to the people of
remove the word “India” from the Constitution and Hind (India) was first used by: [UPSC CSE 1995]
replace it with the word “Bharat” in order to help Indians
overcome their colonial heritage. (a) the Greeks
• According to the Supreme Court, “India is already (b) the Romans
called Bharat in the Constitution itself.” (c) the Chinese
• The Indian Prime Minister mentioned the “Panch Pran” (d) the Arabs
in his Independence Day speech of 2022, emphasizing Answer: (d)
the need to decolonize minds and take pride in India’s
civilisational legacy.
• The Indian Prime Minister was referred to as the “Prime SPECIAL SESSION OF PARLIAMENT
Minister of Bharat” in a government leaflet on his
planned travel to Indonesia for the 20th ASEAN-India
Why in News: Recently, a special session of the Indian
Summit and the 18th East Asia Summit.
Parliament took place in the new Parliament building.
Current Scenario
What is a special session of Parliament?
• Use of “India” Internationally: India has consistently
used the name “India” in all international and multilateral • A unique meeting called by the parliament that does not
fora. take place during the normal sessions is referred to as a
special session.
• This highlights the practicality and standardisation of
using the English name “India” in global diplomacy and • The term ‘special session’ is not explicitly mentioned in
communication. the Constitution or in the rulebooks of the two Houses of
the Parliament. It meets in accordance with the terms of
• Dual-Language Approach: India follows a dual-language
Article 85(1) of the Constitution.
approach in official documents and diplomatic contexts.
• Modern Usage: The use of both “Jai Hind” and “Jai Article 85(1) states –
Bharat” is noted in contemporary India, reflecting the “The President shall from time to time summon each
coexistence of different cultural and linguistic traditions. House of Parliament to meet at such time and place as he/
Signifying a recognition of the diverse historical and she thinks fit, but six months shall not intervene between
cultural threads that make up the fabric of the nation.
its last sitting in one session and the date appointed for its
first sitting in the next session.”
Conclusion
Changing a nation’s name is a multifaceted process that • In accordance with this clause, the President has the
involves legal, administrative and diplomatic steps. It can power to call a special session of the House in the event
impact national identity, international relations and economic an emergency proclamation is made.
opportunities but can also present administrative, legal, and When does Parliament meet?
social challenges. The decision to change a country’s name is
• India’s Parliament has no fixed calendar of sittings.
often a complex one, requiring careful consideration of the
• In 1955, a Lok Sabha committee had proposed a timetable
potential benefits and drawbacks.
for parliamentary sessions.

POLITY & GOVERNANCE  9


• It recommended that the Budget session of Parliament ¾ The Law Minister and the Minister of State
begin on February 1 and go on till May 7, and the for External Affairs are special invitees to the
Monsoon session start on July 15 and end on September Committee.
15. • The President is informed about the Committee’s
• The committee suggested that the Winter session, the decision, who then summons Members of Parliament to
last session of the year, commence on November 5 (or the meet for the session.
fourth day after Diwali, whichever is later) and finish on • The presiding officer of the special session of Parliament
December 22. is the Speaker of the Lok Sabha.

• While the government agreed to this calendar, it was Who Call the Special Session of parliament?
never implemented. • There are no specific guidelines on how or when such a
session can be convened.
Who decides when Parliament meets? • However, Article 352, which deals with the proclamation
• The government determines the date and duration of of Emergency, refers to a ‘special sitting’ of the House.
parliamentary sessions. • This clause was added through the 44th Amendment
Act in 1978, which included safeguards against the
• The Cabinet Committee on Parliamentary Affairs takes Emergency.
this decision.
• The President may call a special meeting to vote against
¾ It currently has ten Ministers. the Emergency at the request of one-tenth of the Lok
Sabha’s members.

EXPECTED QUESTION FOR UPSC CSE


Constitutional provisions:
Prelims MCQ
• The Constitution specifies that six months should not Q1. 
Consider the following statements regarding
elapse between two parliamentary sessions (Article special sessions in the Indian Parliament:
85).
1. Special session has been conducted for the first
time in September 2023
• The framers of the Constitution borrowed it from the
Government of India Act of 1935. 2. Special sessions are conducted only for historical
legacies like Indian freedom struggle.
¾ It allowed the British Governor General to call a 3. No special session has been conducted at the
session of the central legislature at his discretion, midnight.
requiring that the gap between two sessions should
How many of above statements is/are correct?
not be more than 12 months.
(a) Only one     (b) Only two
• The Constituent Assembly reduced the gap between (c) All three     (d) None
sessions to six months. Answer: d

10 POLITY & GOVERNANCE


Q2. Consider the following statements: (a) Only one    (b) Only two
1. The term ‘Special Session’ has been defined (c) All three    (d) None
in the Second Schedule of the Constitution of Answer: b (2 and 3 only)
India.

2. Presiding officers may suspend procedural tools


like ‘Question Hour’ during the Special Session NATIONAL JUDICIAL DATA GRID
of the Parliament. (NJDG)
3. Article 352 of the Constitution empowers the
government to convene the special sitting of the Why in News: Recently, The Supreme Court has integrated
Houses of the Parliament. its case data with the National Judicial Data Grid (NJDG) as
part of an ‘Open Data Policy (ODP)’ to ensure transparency,
How many of the above statements are correct?
accountability, and value creation in government data.

What is the National Judicial Data Grid?

POLITY & GOVERNANCE  11


• The NJDG portal is a national repository of data relating • Pendency Management:
to cases instituted, pending, and disposed of by the courts
across the country. ¾ NJDG is a monitoring tool that provides real-
time data on pending cases at various levels of the
• It is a database of orders, judgments and case details of judiciary in order to identify, manage, and minimize
18,735 District and subordinate Courts and High Courts case pendency.
created as an online platform under the e-Courts Project. • Identifying Bottlenecks:
• Its key feature is that the data is updated in real-time and
¾ The NJDG aids in identifying judicial bottlenecks,
has granular data up to the Taluka level.
enabling policymakers to assess legislative changes,
• The NJDG is a product of Phase II of the e-Courts project, and provides year-wise pendency data for Chief
a Centrally Sponsored Scheme. Justice to prioritize case disposal.

• Enhanced Legal Insights through Data Integration:


• It has been developed by the National Informatics
Centre (NIC) in collaboration with the in-house ¾ The NJDG uses data-driven approaches to track land
software development team of the Supreme Court’s disputes, enabling policymakers to make informed
Computer Cell. decisions about legal reforms and amendments,
leveraging Land Records data from 26 states.
How Does the Data Help? Data for Mains answers
The NJDG functions as a vital monitoring tool that serves 1. Only 0.16% of India’s pending cases are in the Supreme
several crucial functions in the Indian legal system: Court

2. Nature of pending cases at the Supreme Court

12 POLITY & GOVERNANCE


3. Temporal Distribution of Pendency in the Supreme Court

4. The high clearance rate of the Supreme Court

Conclusion
• The National Judicial Data Grid (NJDG) is a powerful tool Descriptive Question
that has ushered in an era of transparency, efficiency, and
data-driven decision-making within the Indian judiciary. Q. Discuss National Judicial Data Grid with special
• By aggregating and analyzing case data from thousands emphasis on e-Courts project. [10 Marks] [150
of courts, it empowers judges, policymakers, and legal marks]
practitioners to navigate the complex web of Indian
litigation more effectively.
• As it continues to evolve and expand, the NJDG is set to play MERCY PETITION
an even more pivotal role in shaping the future of India’s
legal landscape, ensuring that justice is swift and informed.
Why in News: The Bharatiya Nagarik Suraksha Sanhita
EXPECTED QUESTION FOR UPSC CSE (BNSS) 2023, the proposed replacement for the Criminal
Prelims MCQ Procedure Code (CrPC), contains new provision on mercy
Q1. Consider the following statements with reference petitions in death sentence cases.
to the ‘National Judicial Data Grid (NJDG)’:
1. It is a flagship project implemented by the About Mercy Petitions
Supreme Court of India.
• Mercy petition can be filed by an accused to the President
2. 
It is a system for monitoring pendency and
disposal of the cases in the Supreme Court. asking her to change the order of capital punishment
into life imprisonment.
3. 
It provides a comprehensive database of
judgements of District and Subordinate Courts. • A convict who has been awarded death punishment can
file for a mercy petition within a period of seven days
How many of the above statements are correct?
after the date of the rejection of his appeal or special
(a) Only one     (b) Only two leave petition from the Supreme Court.

(c) All three     (d) None • It is the last constitutional resort which the accused
Answer: c has.

POLITY & GOVERNANCE  13


EXPECTED QUESTION FOR UPSC CSE
Prelims MCQ
Q. Consider the following statements:
1. A decision by the President of India on a mercy
petition filed by a death-sentenced convict is
subject to judicial review by the Supreme Court.
2. The Supreme Court does not have the authority
to review presidential directives issued to agents
of the executive department.
Which of the statements given above is/are correct?

(a) 1 only      (b) 2 only

(c) Both 1 and 2    (d) Neither 1 nor 2


Answer: b

SECTION 6A OF DSPE ACT AS


UNCONSTITUTIONAL
Why in News: The Supreme Court ruled that its 2014
judgment in Subramanian Swamy vs Director CBI, which
struck down Section 6A of the 1946 Delhi Special Police
Establishment Act (DSPE Act), would have a retrospective
effect.

Note:
Retrospective effect - The retrospective law is a law that
How does this provision contravene existing legal has backdated effect or is effective since before the time
precedent? it is passed. The retrospective law is also referred to as ex
post facto law.
• In October 2006, in Epuru Sudhakar & Another vs
Andhra Pradesh and Others, the SC held that the powers
of the President or Governor under Articles 72 and 161 What does the Present Judgement say?
are subject to judicial review.
• The Constitution Bench had declared Section 6A
¾ Such a decision can be challenged on grounds that: of the DSPE Act unconstitutional on the ground
that it violated Article 14 of the Constitution due
 it was passed without application of mind;
to its classification of government servants into two
 it is mala fide; categories, one to whom Section 6(A) applied and the
other to whom it did not.
 it was passed on extraneous or wholly
irrelevant considerations; • The Bench in Subramanian Swamy case had not decided
whether the ruling would only have a prospective effect
 relevant materials were kept out of or whether it would also apply retrospectively.
consideration;
¾ The present Bench dealt with the argument that
 it suffers from arbitrariness. striking down Section 6A(1) created an offence, and
thus the judgment which declared it unconstitutional
• However, the BNSS specifically says that the President’s could not have a retrospective effect.
decision on the mercy plea shall be final, and any
question as to the arriving of the decision by the President • The Bench held that once a law is declared to be
shall not be enquired into in any Court. unconstitutional, being violative of Part III of the
Constitution, then it is “void ab initio, still born,
• Under the BNSS, there is no time limit for the President unenforceable and non est in view of Article 13(2) of the
to decide the mercy pleas. Constitution.”

14 POLITY & GOVERNANCE


¾ Article 13(2) provides that the State shall not make
Further, when a law is declared unconstitutional, it is
any law which takes away or abridges the rights
regarded as -
conferred by this Part (Part III) and any law made in
• “void ab initio,” meaning it was void from the contravention of this clause shall, to the extent of the
beginning, contravention, be void.
• “still born,” signifying it was never validly established, • The Bench thus declared that: “Section 6A of the DSPE
• “unenforceable,” indicating it cannot be applied, and Act is held to be not in force from the date of its insertion,
• “non est,” suggesting it has no legal existence. that is, September 11, 2003.”

Article 20(1) of Indian Constitution vs. Section 6A of DSPE Act

 Article 20(1) lays down that no person shall be convicted of any offence except for violation of a law in force at the time
of the commission of the alleged act.

 The Court examined whether Article 20(1) of the Constitution, which safeguards against retrospective criminal laws,
applied to the validity of Section 6A.

 It clarified that Article 20(1) primarily focuses on convictions or sentences under ex post facto laws, not the trial itself.
 Article 20(1) has no bearing in the context of the declaration of Section 6A as unconstitutional as Article 20 (1) doesn’t
bar retrospective application of procedural changes in a criminal trial.

Implications  
Clarity on Retrospective Application- The ruling
clarifies the retrospective application of constitutional
 
Past Cases- This decision impacts cases involving declarations. Laws declared unconstitutional are void
government servants prosecuted under Section 6A before from their inception.
its declaration as unconstitutional. Any convictions or
sentences based on this section may be questioned. Way Forward
The Supreme Court’s decision to declare Section 6A of the
 
Future Cases- Going forward, the CBI and other
DSPE Act unconstitutional and apply it retrospectively has
investigative agencies will not need prior approval to
far-reaching consequences for cases involving government
prosecute government servants under Section 6A.
servants and CBI investigations. It sets a precedent for the

POLITY & GOVERNANCE  15


retrospective effect of such declarations and strengthens the
Which one of the following is correct in respect of the
principle that unconstitutional laws are null and void from
above statements?
the outset (start).
(a) 
Both Statement-I and Statement-II are correct
EXPECTED QUESTION FOR UPSC CSE and Statement-II is the correct explanation for
Statement-I
Prelims MCQ
(b) Both Statement-I and Statement-II are correct and
Q1. Consider the following statements:
Statement-II is not the correct explanation for
Statement I: Delhi Special Police Establishment (DSPE) Statement-I
Act, 1946 shields government servants from prosecution (c) Statement-I is correct but Statement-II is incorrect
without prior approval.
(d) Statement-I is incorrect but Statement-II is correct
Statement II: Section 6A of the Delhi Special Police Answer: d
Establishment (DSPE) Act, 1946 is unconstitutional.

JUSTICE AMITAVA ROY COMMITTEE REPORT


Why in News: Recently, the Supreme Court sought the views of the Centre and the states on the Justice Amitava Roy
Committee report underlining that the correctional justice system is “evidently gender exclusionary”.

16 POLITY & GOVERNANCE


Why was Justice Amitava Roy Committee formed?
How many of the above statements are correct?
• The Supreme Court formed a Committee on (a) Only one    (b) Only two
Prison Reforms in September 2018, chaired by
(c) All three     (d) None
former judge, Justice Amitava Roy. Answer: b (1 and 2 only)

• Aim of the committee: To examine


overcrowding, legal aid availability, remission, Descriptive Question
parole, reasons for violence in prison, and Q. 
Discussing the need of prison reforms in India,
medical facilities. enumerate the recommendations by Amitava Roy
committee. [15 marks] [250 words]
• The committee submitted its final report in
December 2022.
CONSTITUTIONAL BENCHES
Committee Recommendations

• Mandatory segregation of undertrials, convicts Why in News: The Supreme Court of India has established
and first-time offenders inside jails, while a new five-judge Constitution Bench led by Chief Justice of
producing them in courts, during their hospital India (CJI) D Y Chandrachud.
visits, etc,
Related Cases
• The introduction of telemedicine facilities for The cases involve challenges to Section 6A of The Citizenship
inmate healthcare, Act, 1955, the extension of reservations for Scheduled Castes
• Strengthening vocational training and (SCs) and Scheduled Tribes (STs) in the Lok Sabha and state
education programs, legislatures, and the legal immunity of lawmakers from
prosecution for accepting bribes for votes.
• Considering alternatives to imprisonment for
petty offenses in favour of community service, Case 1: Assam Public Works vs. Union of India & Ors.
• In this case, the Constitution Bench will examine the
• Providing proper counseling for inmates with
constitutionality of Section 6A of The Citizenship Act, 1955.
psychological disorders,
• This section was introduced in 1985 after the signing of
• Production of senior citizens and sick prisoners the Assam Accord between the Government of India and
in courts should be done through video- Assam movement leaders.
conferencing medium.
About the Assam Accords and the Section 6A of the
These recommendations aim to address the gender Citizenship Act, 1956
disparities and improve the overall conditions and
• The Assam Accord, signed in 1985, aimed to address
treatment of female prisoners in India. These will
the issue of illegal immigration from Bangladesh into
set the ball rolling for much needed prison reforms
Assam.
and implementation of Justice Mulla committee
• It established the cutoff date for identifying illegal
recommendations.
immigrants as March 24, 1971, and laid out provisions for
their detection and deportation, the creation of Foreigners
EXPECTED QUESTIONS FOR UPSC CSE Tribunals for citizenship verification, and constitutional
Prelims MCQ safeguards for the indigenous Assamese population.
• On the other hand, Section 6A of the Citizenship
Q. Consider the following statements: Amendment Act (CAA) was specifically inserted to
protect the rights and status of individuals who were
1. Prison is a State subject under List II of
included in the National Register of Citizens (NRC)
the Seventh Schedule in the Constitution.
in Assam but were declared as foreigners.
2. Pakwasa Committee in 1949 suggested • This section offers a pathway to Indian citizenship
the system of utilizing prisoners as for certain groups, mainly Bengali Hindus and other
labour for road work without any minorities from Bangladesh, who arrived in Assam
intensive supervision over them.
before December 31, 2014, while excluding Muslims.
3. The Model Prison Manual 1960 does not • Section 6A has been a subject of controversy and protest
allow the establishment of Open Prisons due to its selective approach to granting citizenship.
in India.

POLITY & GOVERNANCE  17


Case 2: Ashok Kumar Jain vs. Union of India and Ors. and government actions do not infringe upon or violate
This case challenges the validity of The Constitution (79th the constitutional rights of citizens.
Amendment) Act, 1999, which extended reservations • Checking the legislature and executive: If a law or
for SCs, STs, and Anglo-Indians in Lok Sabha and state government action is challenged as unconstitutional, the
legislative assemblies beyond the initial 10-year period Constitutional Bench can review it and strike it down if
mandated by Article 334 of the Constitution. it violates the constitution. Eg. Striking down of National
judicial Appointment Commission by the Supreme Court.
Case 3: Sita Soren v. Union of India
• Ensuring constitutional consistency: It ensures that
In this case, the Bench will determine whether Article different branches of government, including the
194(2) of the Constitution grants immunity to Members of legislature and executive, adhere to the principles and
Legislative Assemblies (MLAs) from criminal prosecution framework established by the constitution.
related to the offering or acceptance of bribes.
• Protecting minority rights: It ensures that the majority
What is a Constitutional Bench? does not infringe upon the rights of minorities, upholding
the principle of equality and justice.
• A Constitution Bench in the Supreme Court of India is a
special bench composed of at least five judges. • Promoting Rule of Law: It reinforces the idea that no one,
• These benches are established in specific circumstances including the government, is above the law.
and hold a significant role in interpreting and upholding • Preserving Democracy: It contributes to the preservation
the Indian Constitution. of democratic principles by ensuring that democratic
1. Establishment: institutions and processes are in line with constitutional
• The Chief Justice of India has the authority to constitute a requirements.
Constitution Bench.
Constitution Benches have played a crucial role in deciding
2. Circumstances for Establishment: some of India’s most notable and pivotal Supreme Court
• Interpretation of the Constitution: Article 145(3) of the cases. These cases have had a profound impact on Indian
Indian Constitution stipulates that a Constitution Bench jurisprudence and constitutional law. Examples include:
should be constituted to decide cases that involve “a
substantial question of law as to the interpretation” K. Gopalan v. State of Madras: This case dealt with
 
of the Constitution of India. These cases often deal with preventive detention and had significant implications
fundamental constitutional principles and rights. for civil liberties.
• President of India seeking SC’s opinion: When the Kesavananda Bharati v. State of Kerala: This landmark
 
President of India seeks the Supreme Court’s opinion on a
case introduced the “Basic Structure Doctrine,”
matter of fact or law under Article 143 of the Constitution.
which safeguarded certain core features of the Indian
This article provides for advisory jurisdiction, allowing
the President to seek guidance from the Supreme Court Constitution from being amended.
on important issues related to public welfare. Ashoka Kumar Thakur v. Union of India: This case
 
• Conflicting Judgments: Constitution benches are addressed reservations for Other Backward Classes
convened when multiple three-judge benches of the (OBCs) and had far-reaching consequences for
Supreme Court have issued conflicting judgments on the affirmative action policies in India.
same legal issue. This situation necessitates a clear and
definitive interpretation of the law by a larger bench.
3. Ad Hoc Basis: EXPECTED QUESTION FOR UPSC CSE

Constitution Benches are established on an ad hoc
basis, meaning they are formed as and when the Prelims MCQ
specified conditions mentioned above arise. They are Q. Consider the following statements:
not permanent benches but are constituted to address
specific cases and issues. Statement I: All persons of Indian origin who came
before the 1st day of January, 1966 to Assam from
4. Significance:
Bangladesh shall be deemed to be citizens of India.
• Interpreting the constitution: It resolves complex legal
issues, constitutional disputes, and questions related to Statement II: Section 6A of the Citizenship Act, 1955
the constitution’s interpretation and application. has been declared unconstitutional by the Supreme
• Guardian of constitutional rights: It ensures that laws Court.

18 POLITY & GOVERNANCE


Which one of the following is correct in respect of the POST OFFICE BILL, 2023
above statements?
Why in News: The Rajya Sabha introduced the new
(a) 
Both Statement-I and Statement-II are correct
Post Office Bill in 2023 during the monsoon session of
and Statement-II is the correct explanation for
Parliament. This bill aims to replace the outdated 1898 Act
Statement-I
and usher [brings] in a new era for India’s postal services.
(b) 
Both Statement-I and Statement-II are correct and
Addressing the Limitations of the 1898 Act
Statement-II is not the correct explanation for
Statement-I • The Indian Post Office Act of 1898 was primarily focused
on traditional mail services, and its provisions were ill-
(c) Statement-I is correct but Statement-II is incorrect equipped to accommodate the changing landscape of
(d) Statement-I is incorrect but Statement-II is correct postal services.
Answer: c • It lacked provisions for the evolving role of post offices in
offering citizen-centric services, which are increasingly
important in today’s digital age.

POLITY & GOVERNANCE  19


Key Issues and Analysis
Descriptive Question
• The Bill does not specify procedural safeguards for
interception of articles transmitted through India Post. Q. Examine the history of posts in India and critically
Lack of safeguards may violate freedom of speech and evaluate the provisions of the new Post Office Bill,
expression, and right to privacy of individuals. 2023. [15 Words] [15 Marks]
• The grounds for interception include ‘emergency’,
which may be beyond reasonable restrictions under the AUTONOMOUS DISTRICT COUNCIL
Constitution.
Why in news: To solve the ongoing conflict in Manipur and
• The Bill exempts India Post from liability for lapses
in postal services. Liability may be prescribed through assuage Kukis, the state has proposed to the Centre that the
Rules by the central government, which also administers existing autonomous hill councils be given more autonomy.
India Post. This may lead to conflict of interest.
About autonomous hill (or district) councils
• The Bill does not specify any offences and penalties. • Pre-Independence:
For instance, there are no consequences for unauthorised
¾ When the British took over the-then Assam, their
opening of postal articles by a postal officer. This may
imposition of formal laws on the land was resisted
have adverse implications for the right to privacy of
ferociously by tribal populations living in the hills,
consumers.
who had their own customary laws.
Way Forward ¾ To avoid confrontation, the British divided the hill
• Digital addressing may ease the process of sorting and regions of Assam into “excluded” and “partially
facilitate accurate delivery of mails and parcels. This excluded” areas through the Government of India
provision in the new bill may even facilitate the delivery Act, 1935.
of parcels by drone. ¾ In these areas, federal or provincial laws would not
apply until the governor felt they were needed for
• The new Post Office Bill (2023) recognizes the evolving
peace and development.
role of post offices, introduces provisions for enhanced
security, addresses the challenges in the courier ¾ The ostensible [declared] aim of the provision was to
industry, and acknowledges the changing dynamics of allow tribal populations to govern themselves.
communication in India. • Post- independence:
Conclusion ¾ When India became independent, this provision was
adopted with improvements into the Sixth Schedule
Post Office Bill, 2023 will repeal the Indian Post Office Act, of the Constitution.
1898 – a colonial-era law – and “consolidate and amend the ¾ It was based on recommendations made by a
law relating to the Post Office in India”. The Bill represents committee under then Assam premier Gopinath
a positive step towards modernizing India’s postal services. Bordoloi.
It balances the need for security with the demands of a  It had recommended creation of autonomous
competitive market. district councils (ADCs) in the six hill districts
of Assam so that the tribal people could protect
EXPECTED QUESTIONS FOR UPSC CSE their identity and resources:
Prelims MCQ ¾ United Khasi-Jaintia Hills District,
¾ Garo Hills District,
Q1. With reference to the ‘Post Office Bill, 2023,’
¾ Lushai Hills District,
consider the following statements:
¾ Naga Hills District,
1. It will repeal the Indian Post Office Act, 1898. ¾ North Cachar Hills District, and
2. It states that the Post Office will have the ¾ Mikir Hills District.
exclusive privilege of issuing postage stamps. ¾ Some of these hill districts later became states:

Which of the statements given above is/are correct?  Khasi-Garo districts became Meghalaya,
 Naga Hills District became Nagaland,
(a) 1 only      (b) 2 only
 Lushai Hills became Mizoram.
(c) Both 1 and 2 (d) Neither 1 nor 2 ¾ The existing ADCs were either subsumed, were
Answer: (c) renamed, or new ones were created.
¾ Tripura was added to the list in 1986.

20 POLITY & GOVERNANCE


¾ Currently, there are 10 ADCs under the Sixth ¾ The ADCs are empowered with civil and judicial
Schedule in the North East: powers, can constitute village courts within their
 three each in Assam, Meghalaya and Mizoram, jurisdiction to hear trial of cases involving the tribes.
and ¾ Governors of states, that fall under the Sixth Schedule
specifies the jurisdiction of high courts for each of
 one in Tripura
these cases.
¾ Manipur has six ADCs, but these came into existence
in 1971 under an act of Parliament. ¾ ADCs also have powers to form courts to hear cases
where both parties are members of Scheduled
Functioning of ADCs
Tribes and the maximum sentence is less than 5
• Under the Sixth Schedule, the ADCs and Regional years in prison.
Councils have the authority to create laws, receive
funding from the Consolidated Fund of India, and • Legislative powers:
develop the region in the fields of health care, education,
¾ The councils are also empowered to make legislative
roads, and regulations.
laws on matters like land, forests, fisheries, social
• Parliamentary laws either do not apply to autonomous security, entertainment, public health, etc. with due
districts and regions, or have varying degrees of approval from the governor.
application.
• Limited role of parliament and state legislatures:
• ADCs have up to 30 members and a five-year term.
¾ The Bodoland Territorial Council in Assam is an ¾ The roles of the central and state governments are
exception to this with more than 40 members. restricted from the territorial jurisdiction of these
• Each autonomous district and regional council consist of autonomous regions.
not more than 30 members, of which four are nominated ¾ Acts passed by Parliament and state legislatures
by the governor and the rest via elections. may or may not be applied in these regions unless
• The members remain in power for a term of five years. the President and the governor gives her or his
approval, with or without modifications in the laws
Powers of ADCs
for the autonomous regions.
• Judicial powers:

Sixth Schedule

• According to Article 244 of the Constitution, the Sixth Schedule was enacted in 1949 in order to protect
indigenous groups through the establishment of autonomous administrative divisions known as Autonomous
District Councils (ADCs).

• The Sixth Schedule contains provisions that grant indigenous tribes significant autonomy, and it currently applies
to the Northeastern states of Assam, Meghalaya, Mizoram (three Councils each), and Tripura (one Council).

� Assam has the Bodoland Territorial Council, Karbi Anglong Autonomous Council, and Dima Hasao
Autonomous District Council;

� Meghalaya has the Garo Hills Autonomous District Council, Jaintia Hills Autonomous District Council,
and Khasi Hills Autonomous District Council;

� Tripura has the Tripura Tribal Areas Autonomous District Council; and

� Mizoram has the Chakma Autonomous District Council, Lai Autonomous District Council and the Mara
Autonomous District Council.

� Ladakh currently has the Ladakh Autonomous Hill Development Council (LAHDC).

• They are only executive powers that allow the LAHDC to function as an administrator to manage development,
as specified in the LAHDC Act of 1997.

• While the LAHDC has jurisdiction over land, it cannot enact regulations restricting the use or transfer of any
land.

• It can only assign and stipulate how government land should be used.

POLITY & GOVERNANCE  21


How are Manipur ADCs different from Sixth Schedule ADCs?

Parameters Manipur ADCs Sixth schedule ADCs


Constitutional Status These are dependent on the state Assembly due to the These draw their power from the
provisions of the Act. Indian Constitution.

Legislative Powers Limited to personal matters of marriage, divorce and Wider legislative powers spanning
social customs. several matters of governance.

Manipur ADCs have to route their proposals through Sixth schedule ADCs only need
Hill Area Committees (comprising MLAs from the hills) the assent of the Governor for their
and present it to the state Assembly. proposals to become laws.

Manipur ADCs are subservient to the Deputy This is not the case with other ADCs.
Commissioner, who is appointed by the state
government.

Financial Powers These are dependent on the state government for These get central grants through the
financial devolution. state.

Conclusion
LAÏCITÉ: THE FRENCH SECULARISM
• The ADCs in the past decades have been successful to a
certain extent in preserving the tribal identity. Why in News: Recently, the French government announced
that the practice of wearing an abaya, a traditional Islamic
• So, to address the tribal concerns autonomous councils
robe, would be banned in state-run schools as it violated the
can be used as an effective way forward to achieve the
principle of Laïcité, which is the French idea of secularism.
spirit of Constitution.
About Laïcité
EXPECTED QUESTION FOR UPSC CSE
• Laïcité is a complex and politically charged term that
Prelims MCQ represents the formal separation of the State and Church
• The concept advocates for the complete removal of
Q. With reference to the Autonomous District Council, religious values from the public sphere and replacing
consider the following statements: them with secular values like liberty, equality, and
fraternity.
1. It was established on the recommendations of
the Gopinath Bordoloi Committee. Historical Context: The Law of 1905

2. It has power to assess and collect the revenues. • Laïcité gained concrete form with the enactment of The
Law of 1905 on the separation of church and state.
3. It can make regulations for the control of trading • This law guarantees freedom of conscience and worship
activities by non-tribals. as long as it doesn’t conflict with public order.
How many of the above statements are correct? • It stipulates that the Republic will not financially support
any form of worship and religion should remain a
(a) Only one    (b) Only two private matter.

(c) All three    (d) None Shift in Demographics


Answer: (c)
• Laïcité faced fewer challenges during 20th century
because France was relatively homogenous.
Descriptive Question
• Later in the 1950s and 1960s, there was an influx of
Q. Discuss the historical evolution and significance of immigrants, primarily from Muslim-majority countries
Autonomous District Councils (ADCs) in India's due to decolonization of North Africa.
federal structure. [10 Marks] [150 Words] • This demographic shift led to occasional tensions.

22 POLITY & GOVERNANCE


Events Shaping the Debate Related Controversies

• The year 1989 marked a pivotal moment when three


Muslim girls were expelled from their school in Creil
for refusing to remove their headscarves.

• In 2015, a Muslim girl was banned from class for wearing


a “too openly religious” black skirt.

• In 2020, a school teacher was beheading for showing


controversial cartoons to his pupils

• Subsequent decades saw global events like the 9/11 attack • The acceptability of long garments like abayas remained
and domestic incidents such as the Charlie Hebdo ambiguous.
shooting and the Nice church attack, contributing to
anti-Muslim sentiments. While the French President insists that Laïcité cements a
united France, concerns persist that rigid adherence to it
• In response, France passed laws in 2004 and 2011
prohibiting “ostentatious” religious symbols and face- could exacerbate social divisions and hinder the pursuit of
covering veils in public spaces. universalism.

POLITY & GOVERNANCE  23


Therefore, Laïcité remains an evolving concept in France,
How many of the above statements are correct?
with ongoing debates about its application and impact on
religious and cultural diversity in the country. (a) Only one   (b) Only two
(c) All three   (d) None
EXPECTED QUESTIONS FOR UPSC CSE Answer: a (1 only)

Prelims MCQ
Descriptive Question
Q1. Consider the following statements:
1. 
Secularism is a part of the basic structure of the Q. Examine and compare the approaches to secularism
Constitution of India. in India and France. Discuss the historical, legal, and
cultural factors that have shaped the understanding
2. 
The term secularism was incorporated into the and practice of secularism in these two countries.
Constitution by the 44th Amendment Act, 1978. Also, highlight the challenges and debates
3. 
The Indian concept of secularism connotes a surrounding secularism in contemporary times and
complete separation between religion and the how each nation addresses them. [15 marks] [250
state. words]

IMMUNITY OF LEGISLATORS • Later on, the top court in 1998 quashed the case against
FROM BRIBERY CHARGES the JMM MPs, citing immunity under Article 105(2).
Way Forward
Why in news: The Supreme Court will reevaluate its 1998 • Articles 105(2) and 194(2) are here to make sure that
ruling in PV Narasimha Rao vs. State, referring it to a larger lawmakers speak without fear, but they don’t mean
bench to determine if lawmakers can be prosecuted for lawmakers can break the law.
bribery despite constitutional legal immunity. • Now the decision of bigger bench will shape how
lawmakers can be prosecuted for bribery, a matter that’s
Verdict in the PV Narasimha Rao Case, also known as the incredibly important for India’s democracy.
JMM Bribery Case of 1993 was given in 1998.
EXPECTED QUESTION FOR UPSC CSE
Provisions that grant legislators immunity from prosecution
Prelims MCQ
• Article 105 of the Constitution deals with the powers,
privileges, etc. of the Houses of Parliament and of the Q. Consider the following statements:
members and committees thereof.
¾ This provision exempts MPs from any legal action
1. Article 194 declares that a defamation suit cannot
be filed against any Member of parliament for a
for any statement made or act done in the course of
statement made by him in either House of the
their duties.
Parliament.
¾ For example, a defamation suit cannot be filed for a
statement made in the House. 2. 
Article 105 provides legal immunity to the
Attorney General of India from anything said
¾ In cases where a member oversteps or exceeds the by him in the Parliament.
contours of admissible free speech, the Speaker of the
House will deal with it, as opposed to the court. Which of the statements given above is/are correct?
• Article 194(2) extends this immunity to MLAs and states. (a) 1 only
• In the present case, the court has to decide if the legal (b) 2 only
immunity enjoyed by parliamentarians extends to
prosecution for demanding or taking a bribe.
(c) Both 1 and 2
(d) Neither 1 nor 2
Supreme Court’s ruling on the matter?
• Earlier in 1993 JMM bribery case, under which SC has
Answer: (d)
mentioned that protection under Article 105(2) or 194(2)
and the immunity granted could not extend to cases
concerning bribery for making a speech or vote in the
House.

24 POLITY & GOVERNANCE


SECTION

B INTERNATIONAL RELATIONS

G20 @ INDIA
Why in News: Recently, G20 Summit concludes with Historic New Delhi leaders’ declaration, showcasing India’s diplomatic
triumph amidst global geopolitical challenges and fracture between the East and West and developmental divide between the
North and South.

INTERNATIONAL RELATIONS  25
Major • Global Biofuels Alliance: The launch of the Global Biofuels Alliance saw participation of leaders from
Outcomes Brazil, India, and the United States. This alliance is a priority under India’s G20 Presidency and aims to
of 18th G20 promote cooperation in biofuel production and consumption.
Summit • India-Middle East-Europe Economic Corridor (IMEE-EC): It is a major connectivity and economic
integration initiative announced by several nations, including India, the US, Saudi Arabia, the European
Union, the United Arab Emirates, France, Germany, and Italy.
• African Union as Permanent G20 Member: A historic development was the inclusion of the African
Union (AU) as a permanent member of the G20. This move was welcomed by all G20 leaders and
signifies a commitment to greater inclusivity and representation in global decision-making.

G20 New Delhi Leaders’ Declaration


On Ukraine • Emphasis on adherence to the UN Charter’s principles.
War • A call for refraining from using force to acquire territory or threaten the sovereignty and independence
of any state.
• A rejection of the use or threat of nuclear weapons.
• Stress on peaceful conflict resolution, diplomacy, and dialogue.

On Grain/Food/ • A call for Russia and Ukraine to ensure immediate and unimpeded deliveries of grain, food, and
Energy Security fertilizers/inputs.
• Highlighting the importance of sustaining food and energy security.
• Concerns about potential volatility in food and energy markets.

On Global • Macro-policy cooperation: Enhance macro-policy cooperation and help support Agenda 2030 for
Economic Sustainable Development.
Situation • Promoting public-private engagement Greater engagement with private enterprises to drive
sustainable economic transitions
• Startups and MSMEs: Startups and MSMEs as key pillars of socio-economic transformation by
driving innovation and creating employment.
• Startup20, a new Engagement Group, launched by the Indian Presidency.

On Trade for • Effective Regulatory Cooperation: Aims to reduce regulatory divergences and associated trade
Growth costs, prevent unnecessary trade frictions, monitor trade and investment-related measures, and solve
existing irritants.
• WTO reform: G20 has recommitted towards WTO reforms, to improve its functioning and to
strengthen trust in the multilateral trading system.
• G20 Generic Framework for Mapping Global Value Chains: GVC mapping will help developing
countries move up the value chain, and spur inclusive and resilient economic growth, with the ability
to withstand future shocks.
• High-Level Principles on Digitalization of Trade Documents: These principles will guide a
successful transition towards a more efficient and accessible global trade ecosystem powered by
digital technology.
• Jaipur Call for Action: For fostering access to information for MSMEs The Jaipur Call for Action and
its successful implementation will support the accessibility of trade related information and will spur
the integration of MSMEs in the world trade.
On skill and • Policy Principles will help address global skill gaps and facilitate cross-country comparability.
labour • G20 Toolkit for Designing and Introducing Digital Upskilling and Reskilling Programs.
• G20 Roadmap to Facilitate the Cross-Country Comparison of Digital Skills, that will facilitate
cross-border comparison of digital skills.
• Centre of Excellence (COE): The virtual Center of Excellence (CoE), maintained by the UNESCO,
will help exchange information and be a repository of best practices on digital skilling initiatives,
occupational standards, skill taxonomies etc.

26 INTERNATIONAL RELATIONS


On Fighting • High-Level Principles on Strengthening Law Enforcement related International Cooperation and
Corruption Information Sharing for Combatting Corruption drawing inspiration from PM’s 9-Point Action Plan
against Fugitive Economic Offenders.
• High-Level Principles on Strengthening Asset Recovery Mechanisms for Combatting Corruption.
• HLPs on Promoting Integrity and Effectiveness of Public Bodies and Authorities Responsible for
Preventing and Combatting Corruption
On Achieving • G20’s work and actions for the next 7 years to accelerate progress towards SDGs, including by
SDG increasing financing, harnessing data for development, supporting just transitions globally, and
promoting women-led development.
• Aims to provide governments in the G20 countries and beyond, as well as other tourism actors with
voluntary tools and recommendations to leverage the sector’s capability to progress the SDGs.
• Aims to guide future analysis on the progress made on the 17 SDGs and for financing select SDGs
to complement the G20 Sustainable Finance Roadmap, and will make recommendations to the
stakeholders
On Eliminating • An important step towards resilient, climate-smart and sustainable agriculture, and achieving the
Hunger and SDG of “Zero Hunger”.
Malnutrition • MAHARISHI, a concrete step taken under the Indian presidency, brings focus to climate-sustainable
practices by mainstreaming superfoods such as millets which are climate-resilient and nutritious,
connecting international researchers and institutions, promoting wider dissemination of the research
findings and help in identifying gaps and needs.
Strengthening • Interim Medical Countermeasures (MCM) Coordination Mechanism This mechanism envisions the
Global establishment of R&D and manufacturing networks enabling accountable and timely access to safe,
Health and effective, quality, and affordable medical countermeasures including VTDs (Vaccines, Therapeutics
Implementing & Diagnostics).
One Health • International cooperation to disrupt the proliferation of illicit drugs and precursor chemicals: For
Approach the first time, G20 Leaders focus on addressing the damaging aspects of illicit drugs and precursor
chemicals from the point of view of the health agenda and call for strong international cooperation.
• Recognition of the role of Traditional & Complementary Medicine (T&CM): Potential role of
evidence-based Traditional and Complementary Medicine (T&CM) in health and WHO’s efforts in
this direction have been recognized. First ever Global Traditional Medicine Summit was held.
Delivering • Collaboration to enhance academic and research collaborations among higher education institutions
Quality in G20 member states.
Education • Support learners to acquire foundational literacy and numeracy skills substantially by 2030
Mainstreaming • G20 High-Level Principles (HLP) on Lifestyles for Sustainable Development (LiFE): HLPs provide
Lifestyles for an impetus to ambitious actions for achieving development, environment and climate goals etc.
Sustainable
Development
(LiFE)
Designing • Resource Efficiency and Circular Economy Industry Coalition (RECEIC) Industry-led coalition
a Circular will help facilitate and foster multi-stakeholder engagement including public private partnerships,
Economy World capacity building and knowledge sharing to enhance resource efficiency and accelerate circular
economy transition.

On Hydrogen • G20 High-Level Voluntary Principles on Hydrogen HLPs, a first in G20, will help drive the production,
utilization, and global trade of hydrogen produced from zero and low emission technologies and
its derivatives such as ammonia, with an aim to decarbonize hard to abate sectors and contribute to
global net zero GHG emissions/carbon neutrality.
• A Green Hydrogen Innovation Centre to be set up at the International Solar Alliance.

On Energy • Voluntary Action Plan on Doubling the Rate of Energy Efficiency Improvement by 2030. Action Plan
Efficiency outlines potential energy efficiency measures across the building, industry, and transport sectors,
and highlights the importance of finance and sustainable consumption patterns.

INTERNATIONAL RELATIONS  27
On Renewable • Voluntary Action Plan for Promoting Renewable Energy to Accelerate Universal Energy Access
Energy recognizing the significant role played by renewable energy in the global energy mix, the Action
Plan will help achieve the Universal Energy Access target by 2030 under SDG7, by promoting
renewable energy at an accelerated pace.

• Tripling of renewable energy capacity by 2030, globally. In a first, G20 agreed to pursue efforts
towards tripling the renewable energy technologies capacity globally, through existing targets and
policies, in line with national circumstances, by 2030. This will help in rapid scaling up of RE capacity
globally, and accelerate clean energy transitions.

On critical • Voluntary High-Level Principles for Collaboration on Critical Minerals for Energy Transitions
minerals HLPs emphasize the need to build reliable, diverse, responsible, and sustainable value chains of
critical minerals, semiconductors, and related technologies.

Energy cost • Voluntary Action Plan for Lowering the Cost of Finance for Energy Transitions.
cutting
• Action Plan will help facilitate access to low-cost finance for existing as well as new and emerging
clean and sustainable energy technologies for supporting the energy transitions, considering that the
world needs an annual investment of over USD 4 trillion, with a high share of renewable energy in
the primary energy mix.

Setting up of • The GBA is a global initiative aimed at promoting the production and use of sustainable biofuels to
Global Biofuels address climate change.
Alliance (GBA)
• The alliance was initiated by India, the United States, and Brazil, and it has nine founding members,
including India, the US, Brazil, Argentina, Bangladesh, Italy, Mauritius, South Africa, and the
United Arab Emirates, with Canada and Singapore as observer countries.

• In total, 19 countries and 12 international organizations have agreed to join the GBA.

• The alliance seeks to intensify the use of sustainable biofuels, facilitate global biofuels trade, and
provide technical support for national biofuel programs.

• Biofuels are considered cleaner alternatives to conventional fossil fuels and play a vital role in
reducing carbon emissions.

Climate and • Develop mechanisms to support the timely and adequate mobilization of resources for climate
Sustainable
• G20 Sustainable Finance Technical Assistance Action Plan (TAAP): As an important step aimed
Finance
at reducing the scarcity of skilled professionals in sustainable financing, the G20 developed and
endorsed a multi-year.

• Developed country parties expected to meet the USD 100 billion climate finance goal per year, for
the first time, in 2023.

Conserving, • Gandhinagar Implementation Roadmap (GIR) & Gandhinagar Information Platform (GIP): These
Protecting, will help restore land impacted by forest fires and mining, and reinforce the implementation of the
Suitably Using, G20 Global Land Initiative (GLI).
and Restoring • This will support achieving the G20 ambition of reducing land degradation by 50% by 2040.
Ecosystems
• Scale up efforts to protect, conserve and sustainably manage forests and combat deforestation.

Ending Plastic • Ending Plastic Pollution: G20’s reaffirmation to end plastic pollution by supporting the development
Pollution of an internationally legally binding instrument, and building on the G20 Marine Litter Action Plan
elucidated in the Osaka Blue Ocean Vision (To reduce marine plastic litter to zero by 2050).

28 INTERNATIONAL RELATIONS


Financing • G20 Principles for Financing Cities of Tomorrow will help bridge the infrastructure financing gap
Cities of and raise alternate financing resources.
Tomorrow • G20/ADB Framework on Capacity Building on Urban Administration Framework will guide local
governments in assessing and enhancing their overall institutional capacity for the effective delivery
of public services.

Reinvigorating • Reinvigorating Multilateralism UNGA 75/1 (UNSC reforms) agreed for the first time in the G20;
Multilateralism strong push towards reforms for better, bigger and more effective Multilateral development Banks
(MDBs)
Reforming • Effective MDBs: Need for enhancing the representation and voice of developing countries in
International decision-making in global international economic and financial institutions was underscored;
Financial • Roadmap to implement the Independent Review on MDBs Capital Adequacy Framework (CAF)
Institutions • Roadmap provides a stocktake of the status of implementation of CAF recommendations by each
MDB as reported by the Banks, as well as forward-looking G20 guidance on how to pace up the
implementation of the voluntary recommendations, subject to each MDB’s governance framework
and internal mandates.

Building • A flagship deliverable of the Presidency, DPI’s effectiveness in enabling easier access and delivery of
Digital Public services through innovative public and private sector solutions, has been recognized.
Infrastructure • The DPI framework will ensure equitable access to and last-mile delivery of essential services at a
(DPI) societal scale, streamline governance, propel economic growth, enhance data security, drive financial
inclusion, and foster sustainable development.
• Global Digital Public Infrastructure Repository (GDPIR): GDPIR, a virtual repository of DPI,
proposed to be created by the Presidency, aims to share best practices and experiences in the
development and deployment of DPI to bridge existing information and knowledge sharing gaps.
• One Future Alliance (OFA), a Presidency proposal, aims to bring together governments, the private
sector, academic and research institutions, donor agencies, civil society organizations, existing
mechanisms, and other relevant stakeholders to synergize global efforts in the DPI ecosystem, with the
view to build capacity and provide technical assistance and adequate funding support for implementing
robust, inclusive, human-centric, and sustainable DPI in Lower and middle income countries (LMICs).

Harnessing • Promote Responsible AI and 'pro innovation' regulatory approach with the aim of maximising
Artificial benefits while accounting for the risks associated with AI.
Intelligence (AI • Promote international cooperation on responsible, pro-innovation use of AI, and its international
governance/regulation.
Reducing • Reducing Disaster Risk and Building Resilient Infrastructure: A new WG launched by the Indian
Disaster Risk Presidency, to drive a fundamental transformation in the global approach to disasters from a reactive to
and Building a proactive, people-centered, all-of-society approach to managing and reducing disaster risk.
Resilient • The Working Group is being supported by the incoming Brazilian Presidency, and was also endorsed
Infrastructure at the UN World Conference on DRR.
Harnessing • Chennai High-Level Principles for a Sustainable and Resilient Blue/Ocean-based Economy will help
and Preserving address sustainable economic growth, protection, conservation, restoration, and sustainable use of
Ocean-based the marine environment, social equity, gender equality, and human development.
Economy

On Economies • A commitment to protect the vulnerable through equitable growth and enhancing economic and
& Financial financial stability.
Markets • Reaffirmation of the April 2021 exchange rate commitment by G20 finance ministers and central
bank governors.
• Endorsement of high-level recommendations for regulating crypto-assets.
• Plans to discuss the cryptocurrency roadmap in October 2023.
• Renewed commitment to discourage protectionism and market-distorting practices.

INTERNATIONAL RELATIONS  29
On Climate • A need to accelerate the phase-down of unabated coal power.
Change
• A commitment to facilitate low-cost financing for developing countries for a transition to a low-
carbon economy.

• A goal to triple renewable energy capacity globally by 2030.

• A commitment to scale up sustainable finance.

• The use of carbon pricing and incentives toward carbon neutrality and net zero.

• Recognition of the need for increased global investments to meet Paris Agreement climate goals.

• Acknowledgement of the need for significant financial support for developing countries.

• A call to set an ambitious, transparent, and trackable New Collective Quantified Goal of climate
finance in 2024, starting at $100 billion annually.
On Global Debt • A commitment to address debt vulnerabilities in developing countries to promote resilient growth.
Vulnerabilities • A call for a swift conclusion of debt treatment for Ethiopia.

On Health • Commitment to strengthening the global health architecture.


• Efforts to enhance the resilience of health systems and support the development of climate-resilient
and low-carbon health systems in collaboration with multilateral banks.

India-Middle • The India-Middle East-Europe Economic Corridor (IMEC), announced at the G20 Summit,
East-Europe represents a significant development in India’s quest for deeper connectivity with regions to its
Economic northwest. The project is a part of the Partnership for Global Infrastructure Investment (PGII) — a
Corridor West-led initiative for funding infrastructure projects across the world, seen as a counter to China’s
(IMEC-EC) Belt and Road Initiative (BRI).

• Covered in detail in subsequent pages.

Inclusion of What is the AU?


the newest G20 The African Union (AU) is an intergovernmental organization consisting of 55 member states
 
member-The located on the African continent.
African Union
It was launched in 2002 in Durban, South Africa and serves as the successor to the Organization of
 
African Unity (OAU), which was formed in 1963.

The AU’s primary goal is to create an integrated, prosperous, and peaceful Africa driven by its own
 
citizens.

The AU Commission, its secretariat, is based in Addis Ababa, and collectively, the AU member states
 
have a GDP of approximately $3 trillion with a population of around 1.4 billion people.

Significance of AU’s Inclusion in G20

The inclusion of the AU in the G20 represents a significant step toward a more inclusive and
 
representative global governance structure.

It gives African interests and perspectives a voice and visibility in the G20, a group focused on global
 
economic and financial issues.

India’s push for the AU’s inclusion aligns with its efforts to strengthen its ties with African nations
 
and garner support for its aspirations, such as permanent membership in the United Nations Security
Council (UNSC), where Africa holds 55 votes.

30 INTERNATIONAL RELATIONS


Bridging the • Commitment to halve digital gender gap by 2030: To advance gender equality, foster greater
Gender Digital economic growth, and empower women as change and decision-makers to accelerate a more
divide inclusive and sustainable development.

• Institutional capacities and mechanisms against online abuse of women and girls: By promoting
strategies that help build a safe and secure environment for women and girls in public and private
spaces, as well as online and offline domains, to achieve their full, meaningful, and effective
participation in education, employment, and entrepreneurial domains

Creation of • Women Empowerment Working Group has been formalized Under India’s G20 Presidency, a
a Working decision to create a full-fledged working group on women empowerment has been made.
Group on the
• The group will prioritize gender equality, women empowerment, and leadership, and bring
Empowerment convergence across sectors at all levels.
of Women

Countering • Renewed attention to Counter Terrorism: Strong condemnation of terrorism in all its forms and
Terrorism manifestations - a positive message towards international peace and security.
and Money
Laundering

Sherpa Track • The Sherpas of member countries are the personal emissaries of the Leaders.

• They concentrate on socio-economic issues such as agriculture, anti-corruption, climate, digital


economy, education, employment, energy, environment, health, tourism, trade and investment.

• The Sherpa Track oversees inputs from 13 Working Groups, 2 Initiatives – Research Innovation
Initiative Gathering (RIIG) and G20 Empower, and various Engagement Groups, all of whom meet
throughout the year and develop their Issue Notes and Outcome Documents in parallel.

• These substantive discussions then feed consensus-based recommendations to the Sherpa


Meetings.

Finance Track • It is headed by the Finance Ministers and Central Bank Governors, who generally meet four times a
year, with two meetings being held on the sidelines of World Bank /International Monetary Fund
meetings.
• Focus areas: Fiscal and Monetary policy issues such as global economy, infrastructure, financial
regulation, financial inclusion, international financial architecture, and international taxation.

India’s G -20 Presidency

India’s • India holds the Presidency of the G20 from December 1, 2022 to November 30, 2023.
Presidency

G 20 Logo • The G20 Logo draws inspiration from the vibrant colours of India’s national flag – saffron, white
and green, and blue.

• It juxtaposes planet Earth with the lotus, India’s national flower that reflects growth amid challenges.

• The Earth reflects India’s pro-planet approach to life, one in perfect harmony with nature.

• Below the G20 logo is “Bharat”, written in the Devanagari script.

G20 Theme • The theme of India’s G20 Presidency - “Vasudhaiva Kutumbakam” or “One Earth · One Family ·
One Future” - is drawn from the ancient Sanskrit text of the Maha Upanishad.

INTERNATIONAL RELATIONS  31
India’s Cultural • Bharat Mandapam (inspired from Anubhav Mandapam).
Showcase • Bronze statue of Lord Nataraja (Chola style).
• Konark Chakra of Odisha’s Sun Temple and Image of Nalanda University (used as iconic backdrops).
• Thanjavur Painting and Dhokra art.
• Brass statue of Lord Buddha sitting under the Bodhi tree.
• Diverse musical heritage (Hindustani, Folk, Carnatic, Devotional)

India’s G20 • Green Development, Climate Finance & LiFE


Priorities
 Climate Finance and Technology: India emphasizes the need for support in terms of climate finance
and technology transfer to help developing nations address climate challenges effectively.

 Just Energy Transitions: India is committed to ensure that the transition to clean energy is just
and equitable, particularly for developing nations, to avoid exacerbating existing inequalities.

 LiFE Initiative: India introduces the LiFE (Lifestyle for Environment) movement, focusing on
behavioral changes rooted in sustainable traditions to encourage environmentally-conscious
practices among consumers and markets.

• Accelerated, Inclusive & Resilient Growth

 Integration of MSMEs in Global Trade: India aims to facilitate the inclusion of Micro, Small, and
Medium Enterprises (MSMEs) into global trade, recognizing their potential to drive economic
growth.
 Trade for Growth: Fostering a spirit of trade for growth, India seeks to promote international
trade that benefits all nations and contributes to sustainable development.
 Promotion of Labor Rights and Welfare: India is committed to advocating for labor rights and
ensuring the welfare of workers, fostering a fair and equitable work environment.
 Addressing Global Skills Gap: India recognizes the importance of addressing the global skills
gap by promoting education and workforce development, ensuring a skilled workforce for the
future.
 Inclusive Agricultural Value Chains: India aims to build inclusive agricultural value chains,
promoting sustainable agriculture practices and equitable access to resources.
• Accelerating progress on SDGs
 2030 Agenda Recommitment: India emphasizes the need for renewed commitment within the
G20 to achieve the targets outlined in the 2030 Agenda for Sustainable Development.
• Technological Transformation & Digital Public Infrastructure
 Human-Centric Technology: India aims to champion a human-centric approach to technology,
emphasizing ethical and inclusive digital solutions.
 Digital Public Infrastructure: Prioritizing the development of robust digital public infrastructure
to enhance public services, governance, and access to technology for all.
• Multilateral Institutions for the 21st century
 Accountable and Inclusive Multilateralism: India will advocate for a reformed multilateral
system that prioritizes accountability and inclusivity, ensuring that all nations have a meaningful
voice in global decision-making.
• Women-led development
 Inclusive Growth and Development: India will emphasize inclusive growth and development
as a central theme during its G20 Presidency, promoting policies and initiatives that benefit all
segments of society.
 Women Empowerment: India’s priority will be to empower women, recognizing their crucial
role in economic and social progress.
 Women in Leadership: India aims to promote women’s participation in leadership positions,
both in the public and private sectors, to drive socio-economic development.

32 INTERNATIONAL RELATIONS


GAINS FOR INDIA FROM THE SUMMIT
Descriptive Questions
• India was able to present a consensus text which enabled
the world to look ahead to what should be the solution for Q1. Describe the structure, composition and functioning
a just and lasting peace at the end of the war in Ukraine. of G20. Can this organisation emerge as alternative
• India’s quest to be the voice of the Global South gained to United Nations? [15 marks] [250 words]
prominence as it was able to secure membership for
Q2. 
Discuss the key takeaways from the recently
African Union and highlight the issues of climate finance,
adopted “New Delhi declaration” during the G20
upskilling of workforce which are particularly relevant summit. [15 Marks] [250 Words]
for countries of global south.
For Approach Answer, Scan:
• India made significant efforts to revive reformed
multilateralism at a particularly bleak [bad] time in the
international system.
• India was able to showcase economic growth,
technological advancement, cultural heritage and
diplomatic leadership on global stage.
• India also made the summit a ‘Jan Andolan’, which saw
participation from large sections of society and brought a
diplomatic event out from Delhi to various cities all over
the country. INDIA-MIDDLE EAST-EUROPE
• Showcased the world its comprehensive national power MEGA ECONOMIC CORRIDOR
by organising global meet like G20 successfully.
Why in news: Indian Prime Minister announced the launch
GAINS FOR THE WORLD of the India-Middle East-Europe mega economic corridor at
• Made strides in shifting the worldview from GDP the recently concluded G20 summit.
centric to human centric.
About the Project:
• Common ground found in form of New Delhi Declaration
• The initiative, jointly spearheaded by the US and India,
in this era of polarisation between the global powers.
spans connectivity and infrastructure running through
• Voice of global south gaining prominence, with the India, Saudi Arabia, the United Arab Emirates, Jordan,
inclusion of diverse issues on the high-tables of G20. This Israel and the European Union.
will make the organisation more representative and
• The rail and shipping corridor is part of the Partnership
inclusive.
for Global Infrastructure Investment (PGII).

EXPECTED QUESTIONS FOR UPSC CSE ¾ PGII is a collaborative effort by G7 nations to fund
infrastructure projects in developing nations.
Prelims MCQ ¾ PGII is considered to be the G7 bloc’s counter to
China’s Belt and Road Initiative.
Q. Consider the following:
¾ PGII is part of G7’s vision of Build Back Better
1. Global Biofuels Alliance framework.
• Aim: To enable greater trade among the involved
2. India-Middle East-Europe Economic Corridor countries, including energy products.
(IMEE EC) • The corridor will include:
3. Asia Africa Growth Corridor ¾ a rail link
¾ an electricity cable
4. 
Convention for the Safeguarding of the
Intangible Cultural Heritage ¾ a hydrogen pipeline and
¾ a high-speed data cable.
How many of the above are the outcomes of the 18th • The IMEE EC will consist of two separate corridors:
G20 Summit, New Delhi?
¾ East Corridor connecting India to West Asia/Middle
East
(a) Only one      (b) Only two
¾ Northern Corridor connecting West Asia/Middle
(c) Only three      (d) All four East to Europe
• It will include a rail line that will provide a reliable and
cost-effective cross-border ship-to-rail transit network
Answer: a (1 and 2 only)
to supplement the existing multi-modal transport routes.

INTERNATIONAL RELATIONS  33
• Expansion of G20's role from purely economic grouping
to geo-political grouping
• Geo-Economic Significance: Increased trade and
employment opportunities, facilitated by clean energy
development, can lead to economic cohesion, cost
savings, and job creation in Asia, Europe, and the Middle
East.
• Development of Infrastructure: The proposed economic
corridor project, IEC, aims to address infrastructure gaps
in lower and middle-income nations by serving as a green
and digital bridge across continents and civilizations.
• Promotion of Intra-regional Connectivity: It will
promote Intra-regional connectivity and enhance trade,
prosperity and connectivity.

Need of the project:


• It would increase prosperity in the region through an
increased flow of energy and digital communications.
• This would help deal with the lack of infrastructure
needed for growth in lower- and middle-income nations.
• It could help in reducing the turbulence and insecurity
coming out of the Middle East.
• It also provides an opportunity to counter the growing
Chinese geopolitical influence.
What is the Partnership for Global Infrastructure
Investment (PGII)?
• In 2022, during the G7 summit in Germany, the PGII Significance of the corridor for India:
was officially launched as a joint initiative to help
fund infrastructure projects in developing countries • Cooperation on value chains: Joining forces in
through public and private investments. petrochemicals manufacturing by integrating India’s
hydrocarbon value chain.
• It was presented as an alternative in response to the
infrastructure projects being undertaken and funded ¾ Creating an innovation corridor for green energy and
by China under the Belt and Road Initiative (BRI) at innovative technology manufacturing value chains.
the Global level. • Indo-US convergence in Middle East: India and US have
converged their interests in Middle East after Indo Pacific.
Significance of IMEE-EC corridor ¾ This economic and connectivity project is the second
• Geo-Political Significance: The IMEC aims to counter mega convergence between India and the United
China’s Belt and Road Initiative (BRI), which has led to States in Middle East after the I2U2 forum.
a ‘debt trap’ and reduced member country’s sovereignty. • Strategic engagement with the Arabian Peninsula:
• Geopolitical Stability in Middle East: The Corridor Through this connectivity project India gets opportunity
aims to unite Middle Eastern countries and transform to build enduring connectivity between India and
the region into a hub for economic activity, rather than a Arabia.
source of conflict or crisis. ¾ The current project will strengthen India’s role as a
• Normalisation of Diplomatic Relations: The corridor’s driver in shaping regional connectivity.
passage through Jordan and Israel could normalize • Breaking dependence on Pakistan and Iran: It breaks
diplomatic relations in the Middle East, potentially Pakistan’s monopoly over India’s overland connectivity
establishing relations between Israel and Saudi Arabia, to the West.
and support Israel’s normalization with Arab states.
• This new connectivity architecture could result in an
• Reduced geopolitical dependence on Suez Canal and alternative trans-regional commercial transportation route.
Red sea: The Suez Canal is experiencing congestion,
while China’s militarization of the Red Sea, including ¾ It also provides India an alternate transport corridor
Bab-al-Mandeb and Gulf of Aden, will create an alternate route for Europe by reducing over dependence on
transportation route. Chabahar Port and INSTC.

34 INTERNATIONAL RELATIONS


• Opportunity to join Trans-African Corridor: Trans- ¾ Important technical points have been outlined and
African corridor is an envisaged US and the EU plan preliminary alignments are being finalised, to achieve
to build a corridor connecting Angola, the Democratic seamless transportation.
Republic of Congo, and Zambia. ¾ However, it will take a lot of work to achieve this.
¾ Effective implementation of Indian Middle East • China's Presence: A challenge:
Europe Corridor project would increase India’s
chances of getting an opportunity to join Trans- ¾ While the IMEC has been proposed to counter the
African corridor. Belt and Road Initiative of China, Chinese presence
• Increase in Trade Competitiveness: According to US cannot be wished away along the IMEC route.
report, this economic corridor will make trade between ¾ This is because the Piraeus port (in Greece) is
India and Europe 40% faster. controlled by China Ocean Shipping (Group)
¾ This will increase the trade competitiveness of Indian Company which is a Chinese state-owned company.
exports.
¾ Also, Chinese companies have qualified for multiple
China’s Belt and Road Initiative packages for Phase 1 and 2 of Etihad Rail (in UAE).
• The BRI, launched in 2013, is an ambitious plan to • Objections by Egypt:
develop two new trade routes connecting China with
the rest of the world. ¾ Currently, all trade between India and Europe
happens via the sea route, that passes through Suez
• It is an effort to develop an expanded, interdependent
Canal, controlled by Egypt.
market for China, grow China’s economic and political
power, and create the right conditions for China to ¾ Egypt, which could lose revenue if the Suez Canal
build a high technology economy. is bypassed, could also raise objections to the plan.
• The BRI consists of two main components:
¾ Non-binding MoU
¾ The Silk Road Economic Belt (land-based) and the
21st Century Maritime Silk Road (sea-based). ¾ MoU of IMEC does not create any rights or obligations
¾ These components are collectively referred to as under international law.
“One Belt, One Road.” ¾ The memorandum only sets forth political
• The China – Pakistan Economic Corridor (CPEC), a commitments of its participants which are non-
collection of infrastructure projects that are currently binding.
under construction throughout Pakistan, is part of the
One Belt One Road (OBOR). • Implementation on Ground
¾ As per the MoU, the members intend to meet within
the next sixty days to develop and commit to an
action plan with relevant timetable.
¾ Early groundbreaking of the project is a major
challenge the corridor faces.
• Finance
¾ The Corridor would require massive finance for its
construction.
¾ Arrangement of such funds is a challenge considering
the recession that has been creeping in US and other
advanced economies.
¾ Mobilisation of private-sector finance also remains a
major challenge.
Conclusion

¾ Success of Indian Middle East Europe Corridor could


Challenges that the corridor can face: help diversify the options available to the countries
¾ New constructions for missing links: who have infrastructure requirements.
¾ The onward rail route connectivity from five ports ¾ The speed of implementation of the corridor and
in the UAE and Saudi Arabia stretching up to the its ability to avoid the problem of financial and
Haifa port in Israel will be a mix of already existing ecological sustainability faced by BRI will determine
brownfield projects and fresh greenfield projects to the success of IMEC.
connect missing links.

INTERNATIONAL RELATIONS  35
EXPECTED QUESTIONS FOR UPSC CSE Descriptive Question
Prelims MCQ Q. India Middle East Europe economic corridor can be
a game changer for global connectivity between east
Q. Consider the following statements with reference
and west. Critically analyse. [15 Marks] [250 words]
to the India-Middle East-Europe Mega Economic
Corridor:
1. 
It is jointly spearheaded by the European Union
(EU) and India.
20th ASEAN-INDIA SUMMIT
2. 
It is part of the Partnership for Global
Infrastructure Investment (PGII). Why in news: Indian Prime Minister attended 20th ASEAN-
India Summit and 18th East Asia Summit (EAS) in Jakarta,
3. 
Laying hydrogen pipeline and high-speed data
Indonesia
cable is part of it.
Key Highlights of the Summit:
How many of the above statements are correct?
India presented a 12-point proposal for strengthening
(a) Only one      (b) Only two India-ASEAN (Association of Southeast Asian Nations)
cooperation, which includes:
(c) All three      (d) None
1. Establishing multi-modal connectivity and economic
Answer: a (1 and 3 only) corridor linking South-East Asia-India-West Asia-
Europe.

36 INTERNATIONAL RELATIONS


2. Sharing India’s Digital Public Infrastructure Stack with About ASEAN
ASEAN Partners. • The ASEAN (Association of Southeast Asian Nations) is
3. ASEAN-India Fund for Digital Future focusing on a regional intergovernmental organization formed in
cooperation in digital transformation and financial 1967 by the Foreign Ministers of Indonesia, Malaysia, the
connectivity. Philippines, Singapore, and Thailand.
4. Invitation to ASEAN countries to join the Global Centre • It was initially created to promote regional cooperation
for Traditional Medicine and Coalition for Disaster and address political, economic, and security challenges
Resilient Infrastructure. in Southeast Asia.
5. Renewal of support to the Economic and Research • Over the years, ASEAN has expanded to include Brunei
Institute of ASEAN and East Asia (ERIA) to act as a Darussalam, Lao PDR, Cambodia, Myanmar, and
knowledge partner Vietnam, making it a ten-member association.
6. Called for enhanced cooperation on maritime safety, Organizational Structure
security and domain awareness.
• ASEAN has three major pillars of cooperation: the
7. Called for collective fight against terrorism, terror Political-Security Community (APSC), the Economic
financing and cyber-disinformation. Community (AEC), and the Socio-Cultural Community
8. Called for cooperation in disaster management. (ASCC).
9. Called for working together on Mission Lifestyle for
Regular Summits
Environment (LiFE).
• ASEAN holds biannual summits, rotating the
10. Called for collectively raising issues being faced by
chairmanship among member states.
Global South in multilateral fora.
• These summits provide a platform for leaders to discuss
11. Offered to share India’s experience in providing
regional issues, promote economic cooperation, and
affordable and quality medicines to people through Jan-
strengthen political ties.
Aushadhi Kendras.

India's connection with ASEAN:


• India has a significant interest in ASEAN and considers it a central pillar of its Act East Policy.
• The Act East Policy focuses on enhancing India's engagement with the Asia-Pacific region, emphasizing economic,
political, strategic, and cultural cooperation.

INTERNATIONAL RELATIONS  37
• India's association with ASEAN includes:
• ASEAN Plus Six: India is part of the ASEAN Plus Six grouping, which includes ASEAN member states, along with
China, Japan, South Korea, New Zealand, and Australia. This platform fosters regional cooperation and dialogue.
• Free Trade Agreement: India and ASEAN signed a Free Trade Agreement (FTA) in 2010, promoting trade and economic
integration. This FTA aims to reduce tariffs and facilitate the flow of goods and services between India and ASEAN nations.

East Asia Summit

 The EAS was founded in 2005 as an initiative of the Association of Southeast Asian Nations (ASEAN).

 The EAS is the Indo-Pacific's only leader-led forum, bringing together all-important partners to discuss political,
security, and economic problems of strategic importance.

 The EAS is guided by the ideals of openness, inclusiveness, international law respect, ASEAN centrality, and ASEAN's
position as a driving force.

 In 1991, Malaysian Prime Minister Mahathir Mohamad advocated the formation of an East Asia Grouping.

 On December 14, 2005, the first summit was held in Kuala Lumpur, Malaysia.
Six Priority Areas of Cooperation:
 Environment and energy, education, finance, global health issues and pandemic diseases, natural disaster management,
and ASEAN Connectivity.

Members:

EXPECTED QUESTION FOR UPSC CSE (c) 1, 3, 4 and 5     (d) 2, 3, 4 and 6
Answer: c
Prelims MCQ
Q. Consider the following countries: [UPSC CSE 2018] Q. There are significant areas of convergence between
India and the ASEAN grouping and it is an important
1. Australia   2. Canada pillar of India’s Act East policy. Analyse. [15 Marks]
3. China      4. India [250 Words]

5. Japan      6. USA For Approach Answer, Scan:

Which of the above are among the ‘free-trade partners’


of ASEAN?

(a) 1, 2, 4 and 5     (b) 3, 4, 5 and 6

38 INTERNATIONAL RELATIONS


INDUS WATER TREATY (IWT)
Why in News: Recently, the Permanent Court of Arbitration, or PCA, unanimously rejected India’s objections and confirmed
its competence to consider and resolve the disputes raised by Pakistan over Indus water usage.

INTERNATIONAL RELATIONS  39
More about the Recent Context: the flow in and utilization of the waters of the rivers are
• January 2023: Pakistan initiated arbitration at the PCA exchanged between the Parties every month.
to address the interpretation and application of the • Permanent Indus Commission: The Treaty has provided
IWT to certain design elements of two run-of-river for the institutional mechanism for implementing the
hydroelectric projects: Treaty through Permanent Indus Commission (PIC)
 Kishanganga Hydroelectric Project [on the comprising of one Indus Commissioner each from both
Kishanganga (a tributary of the Jhelum)] the countries.

 Ratle Hydroelectric Project [on the Chenab] • Meeting of Commissioner: The Commission is required
to meet at least once a year and is required to undertake
• India’s Objection: It argued that the Court of Arbitration once in every five years a general tour of inspection of
lacks competence to address the queries raised by rivers for ascertaining facts and also undertake the tour
Pakistan and suggests that these questions should, promptly at the request of the Commissioners.
instead, be resolved through the neutral expert process.
• Settlement of Dispute: The Treaty provides for settlement
• July 2023: The PCA unanimously passed a decision of differences/disputes as per Article IX of the Treaty.
(which is binding on both parties without appeal) The IWT also provides a three-step dispute resolution
rejecting each of India’s objections. mechanism.
 The PCA, based on its interpretation of paragraph • Exit Provision: IWT does not have a unilateral exit
1 of Annexure G and Article IX of IWT, 1960 provision, and is supposed to remain in force unless both
unanimously said that it is competent to “consider countries ratify another mutually agreed pact
and determine the disputes set forth in Pakistan’s
Request for Arbitration”. Contention by Pakistan
• However, India rejected PCA observation and said • As per provision of Treaty, information on new projects
India has been participating in another neutral is required to be sent to Pakistan at least six months prior
expert’s proceedings whose first meeting was held at to start of river works and Pakistan can raise objections
The Hague in February 2023. on the design of these projects.
Salient Provisions of The Treaty • The same are discussed in the PIC for amicable resolution.
However, whenever resolution is not achieved in PIC,
• Allocation of Water: The waters of the Eastern Rivers,
the matter can be taken up to the third forum -Neutral
about 33 Million Acre Feet (MAF), stand allocated to
Expert (NE) or Court of Arbitration (CoA) for resolution
India for unrestricted use and those of the Western
as provided in Treaty.
Rivers, about 135 MAF, to Pakistan.
 Under the treaty, India is allowed to use the western India’s Contentions:
river waters for limited irrigation use and unlimited • From the Indian point of view, the basic dissatisfaction
non-consumptive use, like power generation, with the treaty arises from the fact that it prevents the
navigation, fish culture, and others. country from building any storage systems on the
 India has been given the right to generate western rivers.
hydroelectricity through run of the river projects
on the western rivers subject to specific criteria for Evolving situation
design and operation. • India and Pakistan disagree over the technical design
 The Treaty also gives right to Pakistan to raise features of the two hydroelectric plants (Kishanganga &
objections on design of Indian hydroelectric Ratle).
projects on western rivers. • Pakistan asked the World Bank to facilitate the setting up
 India has about 20% of the total water carried by the of a Court of Arbitration to look into its concern about
Indus system, while Pakistan has 80%. At current the technical issues of the two hydroelectric projects.
usage, India utilises a little over 90% of its quota of India asked for the appointment of a Neutral Expert for
Indus waters. the same purpose.
• Storage of Water by India on Western Rivers: As per the • In 2016 the World Bank in its announcement, paused the
Treaty, the aggregate capacity of all single-purpose and appointment of a Neutral Expert, as requested by India
multipurpose reservoirs after the effective date shall not and the Court of Arbitration, as requested by Pakistan to
exceed 3.6 MAF. resolve issues.
• Supply of information: The Treaty also requires India to • At that time, the World Bank paused the works on
supply information (specified in the Treaty) on new Run- the Kishanganga and Ratle projects “to allow the two
of-River Plants to Pakistan. countries to consider alternative ways to resolve their
• Exchange of data: The Treaty stipulates the exchange disagreements”. Despite the pause, works on the
of data between the two sides. Daily data relating to Kishanganga continued.

40 INTERNATIONAL RELATIONS


Way Forward
How many of the above statements are correct?
• Revisit Indus Water Treaty: IWT can be revisited to
incorporate “equitable and reasonable utilisation” and (a) Only one     (b) Only two
the “no harm rule” in the IWT.
• Involve all stake holders: There is a need to involve local
stakeholders also in any negotiation process between (c) All three     (d) None
India and Pakistan on shared water issues. Answer: c
• Also, a joint group comprising technocrats, climate
experts, water management professionals, and scientists
from both countries can be set up to look at the core of Descriptive Question
the problem.
• Explore cooperation: To make the IWT work there is a Q. Examine the key provisions and significance of the
need to explore cooperation arrangements mentioned in Indus Water Treaty (IWT) in the context of India-
Article VII of the IWT. Pakistan relations. What are the major challenges
• Need of Amendment: As the IWT was signed more than and criticisms associated with its implementation?
60 years ago, periodic updation may be needed due to Discuss with relevant examples. [15 marks] [250
changes in the situation in the Indus River Basin region. Words]
Conclusion
¾ The recent developments around the PCA serve
as a reminder of the value of conversation and INDIA- SAUDI ARABIA STRATEGIC
collaboration between India and Pakistan. PARTNERSHIP COUNCIL
¾ The varying approaches by the two countries, would
result in two simultaneous processes and potentially
contradictory outcomes with the risk of endangering Why in News: Indian Prime Minister and Saudi Arabian
the Treaty. Crown Prince discussed expanding bilateral trade and
defence ties and described the partnership as important
¾ In this regard, both the countries should consider
alternative ways to their disagreements. The two for peace and stability of the region. The leaders also co-
countries need to find a way to resolve their differences chaired the first meeting of the India-Saudi Arabia Strategic
peacefully, in a way that is fair to both sides. Partnership Council (SPC).

Permanent Court of Arbitration (PCA)


India-Saudi Arabia Strategic Partnership Council (SPC)
 The PCA was created at the first Hague Peace
Conference of 1899. • Background: The India-Saudi Arabia Strategic
 It is a non-UN intergovernmental organization Partnership Council was set up in 2019, making India
located in The Hague, Netherlands. the fourth country with which Riyadh formed such a
 Unlike a judicial court in the traditional sense, partnership, after the UK, France, and China.
the PCA provides services of arbitral tribunal to
resolve disputes that arise out of international • India and Saudi Arabia decided to expedite
agreements between member states, international implementation of the $50-billion West Coast refinery
organizations or private parties. project, and identified energy, defence, semiconductor
 It is not a UN agency, but a UN observer. and space as areas for intensified cooperation.

EXPECTED QUESTIONS FOR UPSC CSE • Committees: The SPC consists of two main pillars, each
with its own committees:
Prelims MCQ
Q. With reference to the ‘Indus Water Treaty,’ consider ¾ Committee on Political, Security, Social, and
the following statements: Cultural Cooperation: This committee deals with
1. 
It allocates over 50% of water flow to Pakistan, political and security matters, as well as social and
exceeding India’s usage. cultural cooperation between the two nations.

2. 
This treaty grants India unrestricted use of the ¾ Committee on Economy and Investments: This
Sutlej and Beas rivers’ water. committee focuses on economic cooperation, trade,
3. 
It facilitates the establishment of the Court of and investment-related matters.
Arbitration (COA).

INTERNATIONAL RELATIONS  41
Strategic Partnership Council (SPC)

 The Strategic Partnership Council is a mechanism for enhancing and deepening bilateral relations and cooperation
between two countries, typically in the realms of politics, security, economics, and culture.

 It is a high-level platform that allows the countries involved to collaborate closely on a wide range of issues of mutual
interest.

 The exact structure and scope of a Strategic Partnership Council can vary from one country to another, depending on
the specific agreements and objectives set by the nations involved.

Prospects

The following avenues are crucial to strengthen the relations although there are many prospective areas:
• People-To-People Contact: Saudi Arabia has large number of Indian expatriates who need to further convert their
presence as an asset. The trajectory of the entertainment sector is also a potential means of tourism given the fact that
today Saudi Arabia is focusing on these sector. For example, Malayalam film industry, could prove to be an asset in a
Kerala dominated Diaspora in Saudi Arabia.
• Preparing the Next Generation: As the Fourth Industrial Revolution continues to transform life and work, both India
and Saudi Arabia may place strong emphasis on mobilising the youth as cooperation in the youth empowerment sector
plays an important role in the development of bilateral relations.

42 INTERNATIONAL RELATIONS


• Enhancing Cultural engagement: Cultural exchanges EXPECTED QUESTIONS FOR UPSC CSE
and interaction should continue to be the focus area. Prelims MCQ
Cultural engagement such as promoting yoga gives Q1. With reference to the India-Saudi Arabia Strategic
the much-needed visibility to the bilateral engagement Partnership Council (SPC), consider the following
between the two countries. statements:
1. 
It makes decisions on the basis of voting in the
Conclusion assembly.
2. 
The energy corridor between India and Europe
The establishment of a Strategic Partnership Council
via the Middle East has been launched under
demonstrates the commitment of both countries to this council.
strengthening their ties and advancing their common
Which of the statements given above is/are correct?
interests. It serves as a structured framework to promote
collaboration. These councils are important mechanisms (a) 1 only      (b) 2 only
for promoting and sustaining robust relationships between (c) Both 1 and 2   (d) Neither 1 nor 2
countries. Answer: d

INTERNATIONAL RELATIONS  43
• During the meeting, a report titled “Roadmap of Solar
Descriptive Question
Energy for Universal Energy Access”, was unveiled.
Q. Discuss the significance of India- Saudi Arabia
relations. Also highlight the prospective areas that Key Highlights of the Report
are crucial to strengthen the partnership. [15 Marks] • The report provides a strategy framework for leveraging
[250 Words] solar-powered solutions to address the problem of
global energy access in an efficient and cost-effective
For Approach Answer, Scan: manner.
• It includes case studies, actual instances, and innovative
policies designed to change the way solar mini-grids are
put into practice.
• The report’s findings are highly relevant to Africa,
INTERNATIONAL SOLAR ALLIANCE particularly to rural regions and the Sub-Saharan region.
It highlights a number of solar-powered electrification
Why in News: The International Solar Alliance (ISA) hosted techniques, concentrating especially on solar mini-grids
its 5th regional meeting in Kigali, Rwanda with participation and Decentralized Renewable Energy options.
of 36 countries and Ministers of 15 countries. • These approaches provide practical answers to various
problems with energy availability.
Highlights of the meeting
• Incentivizing the development of regional innovations
• Nine solar power demonstration projects across three
and business models might advance the adoption of solar
countries: 4 in Uganda, 2 in Comoros and 3 in Mali, were
energy generation across the nation.
virtually inaugurated in the meeting.

Renewable Energy (RE) in Africa


¾ Abundance of Renewable Energy Resources: Africa possesses a wide range of renewable energy resources.
¾ Africa is home to more than 40% of the global reserve of critical minerals essential for renewable and low-carbon
technologies.
¾ Africa has the potential to meet its own energy needs through renewable energy sources, reducing reliance on
fossil fuels and contributing to a sustainable energy future.
¾ Global Leader in Renewable Energy: Through investment, innovation, and sustainable development of these
resources, Africa can position itself as a global leader in renewable energy production.
Initiatives for Africa international trade, security, and diplomacy. Eg. African
• SolarX StartUp Challenge: Launched specifically for nations are crucial for India’s SAGAR mission.
Africa, this initiative aimed to promote entrepreneurship • Trade MoUs: India has signed MoUs with African
and advance clean energy in African ISA Member nations on the IOR, demonstrating increased defence
Countries. engagement.
• Global Solar Facility for boosting innovative solar • Showcasing our soft power: The Pan African e-Network
technologies through private investment in Africa. Project provides satellite connectivity, tele-medicine,
and education, with e-VidyaBharti and e-ArogyaBharti
Importance of Africa for India
focusing on free tele-education.
• Potential Market: Africa, with over half a dozen rapidly
• India Taking Advantage of Africa’s RE Potential:
growing countries like Rwanda, Senegal, and Tanzania,
is a global growth pole with a population of over one ¾ Investment Opportunities: India can explore
billion and a combined GDP of 2.5 trillion dollars. investment opportunities in African RE projects
while contributing to local economic development.
• Rich in Resource: India seeks diversification of oil
supplies from the Middle East, focusing on Africa’s ¾ Exporting Renewable Technology: Indian companies
resource-rich nations like Namibia, Niger, and South can export RE technologies and equipment to
Africa, which are major producers of commodities like African markets. Leveraging India's manufacturing
uranium and platinum. capabilities, this can be a win-win for both regions.

• Indian Ocean Geopolitics: Eastern African nations, Way Forward


including Somalia, Kenya, Tanzania, and Mozambique, • India can offer technical expertise and financial assistance
are key players in the Indian Ocean Region’s global to African nations in developing their RE infrastructure.
geopolitics due to their strategic location, natural resources, • India can facilitate capacity-building programs and
security concerns, and regional engagements, impacting research partnerships through collaborative projects.

44 INTERNATIONAL RELATIONS


• India can work towards regional energy partnerships ¾ This can involve the development of energy
with African countries, fostering cross-border energy corridors and transmission infrastructure to
trade. efficiently transfer RE across borders, ensuring a
stable and sustainable energy supply.

INTERNATIONAL RELATIONS  45
dotted line, or nine-dash line) officially drawn on the Chinese
EXPECTED QUESTION FOR UPSC CSE
map in 1947 by the then–Chinese Nationalist Government,
Prelims MCQ which was originally an “eleven-dotted-line”.

Q. 
Which of the following statements about the  After the Communist Party of China took over mainland
International Solar Alliance (ISA) is not true? China and formed the People’s Republic of China in 1949,
the line was adopted and revised to nine as endorsed by
(a) 
The ISA was launched at the 2015 United
Nations Climate Change Conference (COP21) Zhou Enlai.
in Paris.
 The line, which has been called a “traditional maritime
(b) ISA aims to promote solar energy deployment boundary line”, encloses the main island features of
globally and facilitate solar projects in the SCS: the Pratas Islands, the Paracel Islands, the
member countries. Macclesfield Bank, and the Spratly Islands. No country,
including Southeast Asian countries or their past rulers,
(c) The ISA has a target of achieving 1 TW of
protested or challenged the validity of the 9-dash line
solar energy by 2025.
from 1947 to 1970s.
(d) The ISA’s membership is open to all United
Nations member countries, regardless of
their geographical location.
Answer: c

CHINA’S U-SHAPED LINE


Why in News: China's U-shaped line loops as far as 1,500
km (932 miles) south of its Hainan Island and cuts into
the exclusive economic zones (EEZs) of Vietnam, the
Philippines, Malaysia, Brunei and Indonesia.

About China U-Shaped Line


• China's U-shaped line is a territorial claim that
encompasses approximately 90% of the South China Sea.
• It is a source of significant disputes in the region due to its
vast coverage, with more than $3 trillion of trade passing
through this contested waterway annually.
• In the past, China had asserted its territorial claims in
the South China Sea using a "nine-dash line," which was
depicted on maps to encircle the region.
U-Shaped Line vs. Nine-Dash Line
• Geographical Scope: The U-shaped line, as depicted in Arguments Against China’s U-shaped Line:
the latest map, covers a broader geographical area than
the previous nine-dash line. It extends as far as 1,500 km • T
 he Philippines: The Philippines has urged China to act
south of China's Hainan Island. responsibly and abide by international law. They refer
to a 2016 arbitral ruling that declared the U-shaped line
• Number of Dashes: The nine-dash line had, as the name
suggests, nine dashes. However, the U-shaped line as having no legal basis. The Philippines contends that
includes ten dashes, marking a subtle but notable change China's claim lacks legal grounds under international law.
in the territorial claim. • Malaysia: Malaysia has filed a diplomatic protest over
• Inclusion of Taiwan: Unlike the nine-dash line, the the U-shaped line, asserting that it holds no binding
U-shaped line includes Taiwan, a democratic and self- authority over Malaysia.
governed island that China considers a part of its territory.
• Taiwan: Taiwan has historically opposed China's
Background territorial claims, including the nine-dash line. Taiwan
 The prevailing basis for China’s historic claims to the SCS emphasizes its sovereignty and rejects being included in
(South China Sea) is the U-shaped line (also called nine- China's territorial assertions.

46 INTERNATIONAL RELATIONS


Conclusion
KHALISTAN MOVEMENT
China has not given up maritime claims based on the
U-shaped line and recent practice has demonstrated that Why in News: The recent developments surrounding the
China is attempting to further consolidate the claim based on Khalistan issue have significantly strained diplomatic
the line. relations between India and Canada.

The Khalistan Movement- A Historical Perspective


EXPECTED QUESTION FOR UPSC CSE
• The Khalistan movement, which seeks an independent
Descriptive Question Sikh state in India, has been a longstanding and
contentious issue in India-Canada relations.
Q. 
Examine China’s maritime claims in the South • With roots dating back over four decades, this movement
China Sea and their implications for India’s strategic has persisted in the Canadian Sikh diaspora, leading to
interests in the region. How should India navigate tensions between the two nations.
its maritime security and diplomatic strategies to
• The Khalistan movement finds its origins in the 1970s
safeguard its interests while maintaining regional
and 1980s when demands for an independent Sikh state
stability? Discuss with relevant examples. [15 Marks] gained momentum among some segments of the Sikh
[250 Words] community.

Background

 India’s Punjab state – which is about 58% Sikh and 39% Hindu – was rocked by a violent Khalistan separatist
movement in the 1980s and early 1990s, in which thousands of people died.
 
The Khalistan movement has roots in Canada, with the establishment of a ‘Khalistan government in exile’ office in
Vancouver in 1982.
 The movement gained momentum after Operation Bluestar in 1984.
 Recently, Hardeep Singh Nijjar was shot dead outside a Sikh temple in Surrey, British Columbia.
 Nijjar had campaigned for an independent Sikh nation – known as Khalistan – to be carved out of India’s Punjab state.
 He was wanted by Indian authorities and had been designated as a “terrorist” in July 2020.
 Hardeep Singh Nijjar was chief of the Khalistan Tiger Force.
 India has designated the Khalistan Tiger Force as a terrorist outfit.
 This incident led to speculation on social media about whether the Indian intelligence agencies were linked to these
deaths abroad, including on Canadian soil.

Canada as a safe haven

• Canada has earned a reputation as a safe haven for Khalistan supporters and voices accused of terrorism in India.
• This leniency is attributed to political considerations, including "vote bank politics," as Canadian Sikhs form a significant
political constituency, and their votes often influence election outcomes.
• India has repeatedly expressed concerns over Canada's perceived tolerance of Khalistani activities within its borders.
Recent Incidents

• Recent incidents in Canada have reignited [restarted] concerns about the Khalistan movement.
• For instance, a parade in Brampton, Ontario, seemingly glorified the assassination of former Indian Prime Minister Indira
Gandhi.
• India has expressed strong disapproval, urging Canada to curtail "anti-India activities" and designate individuals as
terrorists who face such charges in India.
• Canada’s accusation that the Indian Government had a part to play in the assassination of a Khalistani leader on Canadian
soil further complicated the issue.
• Canada also expelled an Indian diplomat following the incident

INTERNATIONAL RELATIONS  47
• Indian response has been a categorical rejection of the Geopolitical factors keeping Khalistan issue alive
allegations levelled by the Canadian Prime Minister.
• Beyond domestic politics, geopolitical factors play a role
• India also expelled a Canadian diplomat in the process. in sustaining the Khalistan movement.
Impact of recent developments on diplomatic relations:
• Countries like China and Pakistan have been accused of
• These have caused a dramatic escalation of tensions providing support, financial or otherwise, to Khalistani
between India and Canada. militants so as to destabilize India.
• The strained relationship had been marked with a tense
Changing Dynamics in the Diaspora
exchange between Canadian and Indian leaders during
the G20 Summit. • While the Khalistan movement has persisted in the Sikh
diaspora, support for it has dwindled over the years.
• The allegations and expulsions of diplomats further
widen this diplomatic rift, potentially affecting • Younger generations, with limited personal memory of
cooperation and dialogue between the two nations on India's tumultuous past, are less inclined to back the cause.
various international issues.
• Additionally, the movement's influence remains
• The two countries, that could agree on the outlines of a significant primarily due to political considerations,
trade deal by the end of 2023, have now frozen talks on rather than broad popular support.
the agreement. Canada gave few details while India cited
“certain political developments”.
Other Aspects of India- Canada Bilateral Relations

Relations Examples and initiatives

Diplomatic • India established diplomatic relations with Canada in 1947.


• Prime Minister of India’s visit to Canada in April 2015 elevated the bilateral relationship
to a strategic partnership.

Political • India and Canada share commonalities in Parliamentary structure and procedures.

Commercial • With more than $13.7 billion in trade, India was Canada's 10th largest two-way
merchandise trade partner in 2022.
• Canadian merchandise exports to India totaled $5.3 billion, ranking ninth.

Nuclear cooperation • Nuclear Cooperation Agreement (NCA) signed in 2010. Joint Committee on Civil
Nuclear Cooperation, restoration of nuclear cooperation.

Science and technology • IC-IMPACTS (the India-Canada Centre for Innovative Multidisciplinary Partnerships
to Accelerate Community Transformation and Sustainability) is the first, and only,
Canada-India Research Centre of Excellence.

Space cooperation • Cooperative and commercial relations in space science, satellite launch services, ground
support, and the launch of Canadian nanosatellites.

Security and defence • Collaboration in international fora, mutual ship visits. Framework for Cooperation
between India and Canada on Countering Terrorism in 2018.
• Indo-Pacific: Both countries share a suspicion of China and support free and open
navigation in international waters.

Population • Canada is home to nearly 1.4 million people of Indian ethnic or cultural origin, about
3.7% of the country's total population, according to the 2021 census.
• More than 7,70,000 people reported their religion as Sikhism, about 2% of Canada's
population, and in 2019, the government designated April as Sikh Heritage Month.

48 INTERNATIONAL RELATIONS


Cultural exchanges • Country of Focus at the International Film Festival of India.
• India-Canada Co-production Agreement.
• Diwali celebrations on Parliament Hill.
Cooperation during • Repatriation flights for stranded Canadians.
COVID-19 • Export of medicines and medical supplies to Canada.

Conclusion Way Forward


• The Khalistan movement's persistence in Canada is a • It is essential for the Government to effectively convey to
complex issue with historical, political, and geopolitical the host countries that they cannot simultaneously have
dimensions. good relations with India while allowing anti-India
• While it continues to be a source of tension in India- separatist movements on their soil.
Canada relations, its influence is gradually diminishing
within the Sikh diaspora. • However, it is essential for both nations to engage
in dialogue and cooperation to address concerns
• It is imperative for both countries to find common ground
and prevent the issue from further straining bilateral
and seek solutions that promote stability, understanding,
relations.
and cooperation on the international stage.

EXPECTED QUESTIONS FOR UPSC CSE (c) All three    (d) None


Prelims MCQ
Answer: b (1 and 2 only)

Q. Consider the following statements: Descriptive Question


1. India has signed the Nuclear Cooperation Q. India and Canada are dealing with an unprecedented
Agreement (NCA) with Canada. diplomatic crisis, which can have socio-political,
economic as well as geopolitical consequences.
2. IC-IMPACTS is the first and only Canada-India Discuss [15 Marks] [250 Words]
Research Centre of Excellence.
For Approach Answer, Scan:
3. In 2022, India ranked as Canada’s 2nd largest
trading partner.
Which of the statements given above is/are correct?
(a) Only one    (b) Only two

INTERNATIONAL RELATIONS  49
FIVE EYES ALLIANCE
Why in News: India halting visa services in Canada could impact relations with Western allies, including Five Eyes Alliance.

About Five Eyes Alliance (Founded: 1941) How many of the above are allies in the Five Eyes
• Members: Australia, Canada, New Zealand, the United Alliance?
Kingdom and the US
(a) Only one    (b) Only two
• These countries are parties to the multilateral UK-USA
(c) Only three    (d) All four
Agreement, a treaty for joint cooperation in signals Answer: c (1, 2 and 4 only)
intelligence.
• These partner nations exchange a wide spectrum of
intelligence within one of the world’s most tightly-knit INTERNATIONAL ORGANISATION
multilateral agreements as part of the collaboration.
OF LEGAL METROLOGY
• Following its origin, it was later enlarged its core group to
‘Nine Eyes’ and 14 Eyes alliances as well, encompassing
CERTIFICATES
more countries as security partners. Why in News: India has become 13th country in the world
Five Eyes’ Role in the Current India-Canada Issue that can issue an OIML (International Organization of Legal
Metrology) approval certificates.
• Countries like the US, the UK and Australia especially
are seen as close to India. They also have significant What is the OIML certificate?
Indian and Indian-origin populations within them, • The OIML-CS is a system for issuing, registering and
similar to Canada. using OIML certificates, and their associated OIML
¾ They have also seen some instances of pro-Khalistan type evaluation/test reports, for instruments like digital
activities in the last few years. But due to their historical balance, clinical thermometers, etc.
closeness to Canada and the alliance on one hand, and • With the addition of India, the number of countries
India, an emerging global power on the other, outright authorised to issue OIML certificates has increased to 13.
support for India or Canada is unlikely.
• At present, Australia, Switzerland, China, Czech Republic,
• Given the state of ties, these countries, particularly the Germany, Denmark, France, United Kingdom, Japan,
US, could play a mediating role in the issue once they Netherlands, Sweden and Slovakia can issue OIML certificates.
have clear intelligence and information on the matter.
Benefit of this development for India
EXPECTED QUESTION FOR UPSC CSE • To sell a weight or measure in the international market an
OIML Pattern Approval certificate is mandatory, which
Prelims MCQ the Department of Consumer Affairs can issue now.
Q. Consider the following countries: • The domestic manufacturers can now export their weighing
and measuring instrument worldwide without incurring
1. Australia    2. Canada additional testing fees, resulting in significant cost savings.
• It will create facility for international manufacturers to
3. Japan      4. USA
get their equipment tested in India.

50 INTERNATIONAL RELATIONS


About OIML • Though these are governed by the Convention on Biological
Diversity (CBD), benefits have been rare till date.
 
The OIML (International Organisation of Legal • The treaty would make it mandatory for patent applicants
Metrology) was established in 1955 and headquartered to declare or disclose their use of genetic resources and
in Paris. any associated traditional knowledge.
 The OIML is an international standard-setting body. Objective:

 It develops model regulations, standards and related • To enhance the efficacy, transparency and quality of
documents for use by legal metrology authorities and the patent system with regard to genetic resources and
associated traditional knowledge; and
industry.
• Prevention of patents being granted for inventions that
 It plays a crucial role in harmonising national laws are not novel or inventive.
and regulations on performance of measuring
Challenges:
instruments like:
¾ Clinical thermometers, 1. 
The draft text fails to address the problem of the
¾ Alcohol breath analysers, biopiracy of genetic resources and associated traditional
knowledge using patents.
¾ Radar speed measuring instruments,
¾ Ship tanks found at ports, and 2. The base text for this treaty categorically excludes any
¾ Petrol dispensing units. provision that is already addressed by other international
instruments. These include:
 India became a member of the OIML in 1956.
¾ Benefit-sharing and misappropriation, which are
EXPECTED QUESTION FOR UPSC CSE already dealt with in the CBD,
¾ the Nagoya Protocol on Access to Genetic Resources (GR)
Prelims MCQ
¾ the Fair and Equitable Sharing of Benefits Arising
Q. Consider the following statements with reference to from their Utilization to the Convention on Biological
the International Organisation of Legal Metrology Diversity,
(OIML): ¾ the International Treaty on Plant Genetic Resources
1. It sets international standards to be used by for Food and Agriculture and
legal metrology authorities and industries. ¾ the Pandemic Influenza Preparedness Framework.
2. 
India is one of the member countries of the 3. The scope of the proposed international instrument is very
OIML. narrow and leaves out Digital Sequence Information
from its scope.
Which of the statements given above is/are correct?
Nagoya Protocol
(a) 1 only     (b) 2 only
The Nagoya Protocol on Access to Genetic Resources
 
(c) Both 1 and 2   (d) Neither 1 nor 2 and the Fair and Equitable Sharing of Benefits Arising
Answer: (c) from their Utilization (ABS) to the Convention on
Biological Diversity is a supplementary agreement
to the Convention on Biological Diversity.
TREATY ON TRADITIONAL International Treaty on Plant Genetic Resources for
KNOWLEDGE Food and Agriculture

Why in News: Member countries of the World Intellectual Its objective is the conservation and sustainable use
 
Property Organization (WIPO) are mooting a new treaty of all plant genetic resources for food and agriculture
the fair and equitable sharing of the benefits arising
regarding Intellectual Property and Genetic Resources,
out of their use, in harmony with the Convention on
Traditional Knowledge and Folklore. Biological Diversity, for sustainable agriculture and
Key details: food security.
• The Intergovernmental Committee met in Geneva to Biopiracy
finalise the text of this legal instrument.
Is the use of bio-resources by multinational
 
• This new treaty, if agreed on, could ensure benefits to companies and other organisations without proper
indigenous communities that have protected genetic authorisation from the countries and people
resources and are knowledge holders of how these can concerned without compensatory payment.
be used.

INTERNATIONAL RELATIONS  51
India’s position
How many of the above statements are correct?
• India wants researchers to reveal the precise source of the
GR rather than simply naming the country of origin, as (a) Only one    (b) Only two
the draft treaty requires.
(c) All three    (d) None
• According to India, “Traditional Knowledge associated
with Genetic Resources” refers to any knowledge Answer: c
that is “evolving, generated in a traditional context,
whether documented or not, collectively preserved, and
transmitted from generation to generation, and including ASIA PACIFIC FORUM ON HUMAN
but not limited to know-how, skills, innovations, RIGHTS
practices, and learning, that are associated with GRs.”
• India’s proposals have not been incorporated in the text Why in News: Recently the President of India inaugurated
due to lack of consensus. the Annual General Meeting and Biennial Conference of the
Asia Pacific Forum on Human Rights in New Delhi.
About WIPO
About Asia Pacific Forum:
• The World Intellectual Property Organization (WIPO)
is a specialized agency of the United Nations. • It was founded in 1996.
• WIPO was created to promote and protect intellectual • Its Secretariat is based in Sydney, Australia.
property (IP) across the world by cooperating with • The Purpose of the Forum is to bring together National
countries as well as international organizations Human Rights Institutions (NHRIs) of the Asia Pacific
pursuant to the 1967 Convention Establishing the to address human rights challenges in the region.
World Intellectual Property Organization. • Members: The APF now includes 26 NHRIs as members,
• WIPO’s activities include: including 16 ‘A status’ members, 9 ‘B status’ members,
¾ hosting forums to discuss and shape international and one (Afghanistan) that is temporarily suspended.
IP rules and policies, The APF had five founding members.
¾ providing global services that register and protect • Membership Requirements: A NHRI must completely
IP in different countries, adhere to the Minimum International Standards
outlined in the Paris Principles in order to be admitted
¾ resolving transboundary IP disputes,
as a full member.
¾ helping connect IP systems through uniform
¾ NHRIs are allowed associate membership if they
standards and infrastructure, and
only partially adhere to the Paris Principles.
¾ serving as a general reference database on all IP
• Members – Afghanistan, Australia, India, Indonesia,
matters
Jordan, Korea, Malaysia, Mongolia, Nepal, New Zealand,
• Headquarters: Geneva, Switzerland. Palestine, Philippines, Qatar, Thailand, Timor Leste.
• Members:
¾ WIPO currently has 193 member states, including Paris Principles
190 UN member states and the Cook Islands, Holy  The Paris Principles (‘Principles Relating to the Status
See and Niue. of National Human Rights Institutions’) set out the
minimum standards that NHRIs must meet in order to
EXPECTED QUESTION FOR UPSC CSE be considered credible and to operate effectively.

Prelims MCQ  
They were endorsed by the Vienna World Conference
on Human Rights and the UN General Assembly in
Q. With reference to the ‘World Intellectual Property 1993.
Organization (WIPO)’ consider the following
statements:
APF and GANHRI
1. 
It is a global forum to promote and protect
intellectual property (IP) services across the • The Asia Pacific Forum (APF) is one of four regional
world. networks of NHRIs within the Global Alliance of National
Human Rights Institutions (GANHRI).
2. It is a self-funding agency of the United Nations.
• Other 3 Regional groupings of NHRIs are -
3. 
General Assembly is the highest decision-
making authority in WIPO. ¾ Network of African National Human Rights
Institutions (NANHRI)

52 INTERNATIONAL RELATIONS


¾ European Network of National Human Rights • GANHRI is a global network of national human rights
Institutions (ENNHRI) institutions (NHRIs). It is constituted as a non-profit
entity under Swiss law.
¾ Network of National Institutions in the Americas

• Full membership of the APF is equivalent to the GANHRI EXPECTED QUESTION FOR UPSC CSE
accreditation of “A status” and associate membership of
the APF is equivalent to the GANHRI accreditation of “B
Prelims MCQ
status”. Q. Which of the following countries is not a member of
the Asia Pacific Forum?
Global Alliance of National Human Rights Institutions
(GANHRI)
(a) Afghanistan    (b) Australia

• GANHRI was previously known as the International (c) Jordan       (d) Japan
Coordinating Committee of institutions for the Answer: d
promotion and protection of human rights (ICC).

UNITED NATIONS COMMISSION ON INTERNATIONAL TRADE LAW


(UNCITRAL)
Why in News: Recently, India hosted inaugural UNCITRAL South Asia Conference.

About UNCITRAL
• UNCITRAL (established in 1966) is a subsidiary body of General Assembly of UN with a mandate to further harmonization
and unification of law of international trade.
• Membership: UNCITRAL is composed of 70 Member States, which are elected by the United Nations General Assembly.
• Members are elected for terms of six years. (India is member till 2028).
• Legal Instruments: UNCITRAL has developed a wide range of legal instruments and model laws to address various
aspects of international trade law. These instruments cover areas such as international sales of goods, international
commercial arbitration, electronic commerce, and secured transactions, among others.
• UNCITRAL is not a part of WTO. WTO deals with trade policy issues whereas UNCITRAL deals with laws applicable to
private parties in international transactions and is not involved with “state-to-state” issues.

EXPECTED QUESTION FOR UPSC CSE

Prelims MCQ

Q. Which of the following statements about UNCITRAL (United Nations Commission on International Trade Law)
is correct?

(a) UNCITRAL is a part of the World Trade Organization (WTO) and primarily deals with trade policy issues.

(b) UNCITRAL has 50 Member States elected by the United Nations General Assembly.

(c) UNCITRAL focuses on state-to-state issues in international trade.

(d) UNCITRAL has developed legal instruments related to international trade law, but it is not a part of WTO.
Answer: (d)

INTERNATIONAL RELATIONS  53
SECTION

C ECONOMY

UNEMPLOYMENT MEASUREMENT • Unemployment rate = (Unemployed Workers / Total labour


force) x 100
Why in News: The Periodic Labour Force survey (PLFS) of
2021-22 showed unemployment reducing to 4.1%, much lower Main Types of Unemployment in India
than before, but higher than some developed economies.
• PLFS of 2017 revealed the unemployment rate of India to • Disguised Unemployment: This occurs when more
be 6.1%, the highest ever recorded in India. people are employed in a task than actually required.

¾ It’s common in India’s agricultural and unorganized


Rozgar Mela
sectors.
Context: Recently, the PM of India distributed 51,000
appointment letters to newly inducted recruits as part of • Seasonal Unemployment: This type of unemployment
the Rozgar Mela. happens during specific seasons of the year. Eg. lower
employment for agricultural labourers in some seasons.
What is Rozgar Mela?
It is an event where a number of employers and job • Structural Unemployment: This results from a
seekers come together for the purpose of applying and mismatch between available jobs and the skills of
interviewing for jobs. theworkforce.Eg.Unemployableyouthdespiteavailable
jobs in artificial intelligence sector.
Why aren’t these Rozgar Melas enough for
unemployment? • Cyclical Unemployment: It’s tied to the economic
cycle, rising during recessions and falling during
• Insufficient job opportunities: India requires 20-200 periods of economic growth. India generally experiences
million new jobs, but these rozgar melas offer very few low levels of cyclical unemployment compared to
in comparison. developed countries.
• Lack of job generation: Just pre-existing vacancies are
• Frictional Unemployment: The period of time between
filled by these melas.
the current to a new job is referred to as frictional, or
• Low Public Employment: The size of public employment temporary unemployment.
in India was already quite low.
How unemployment is measured in India?
• Because the size of public employment in India is
National Sample Survey Office (NSSO)
already quite low.
• It conducted household surveys on employment–
What do we understand by Unemployment? unemployment at regular intervals of five to six years
(quinquennial rounds) till the National Sample Survey (NSS)
• National Sample Survey Organization (NSSO) defines
68th round (2011–12) (NSSO 2013).
employment and unemployment on the following activity
status of an individual: • These are widely considered to be the most reliable source of
employment–unemployment estimates and is extensively
¾ Employed: Engaged in an economic activity
used at the national, state, and regional levels.
¾ Unemployed: Seeking or available for work
Classification of Working Status by NSSO
• Unemployment rate: It is the number of unemployed
people divided by the number of people in the labour • Usual Principal and Subsidiary Status (UPSS): It is
force. based on the activity in which an individual spent relatively
long time in the previous year.
¾ Labour force: It is the sum of employed and
unemployed people. ¾ Non-workers, as per principal status, are considered
employed if engaged in economic activity for at least 30
¾ Out of Labor Force: People neither seeking nor available
days in a subsidiary role.
for work.
• Current Weekly Status (CWS): It adopts a shorter reference
¾ Participation rate: The percentage of people in the
period of a week.
working-age population that are in the labour force.

ECONOMY  54
¾ An individual is counted as being employed if s/he has Way Forward
worked for at least one hour in the past seven days.
The Arvind Panagariya task force (2017-18) recommended
¾ UPSS unemployment rates will always be lower than using technology to speed up data collection and processing to
CWS rates. make timely data available for policy planners and researchers to
Periodic Labour Force Survey (PLFS): make informed decisions.

National Sample Survey Office (NSSO) launched Periodic Conclusion


Labour Force Survey (PLFS) in April 2017. Presence of credible and reliable data with policy makers
• ThequinquennialNSS68throunddatawererecentlyreplaced regarding unemployment will ensure that effective and
with the Periodic Labour Force Survey (PLFS), which is implementable policies are formulated which helps addressing
undertaken every year. the menace of unemployment.
• The objective of PLFS is primarily twofold: EXPECTED QUESTION FOR UPSC CSE
¾ To estimate the key employment and unemployment
indicators (viz. Worker Population Ratio, Labour Force Prelims MCQ
ParticipationRate,UnemploymentRate)intheshorttime
interval of three months for the urban areas only in Q. Consider the following statements:
the ‘Current Weekly Status’ (CWS).
1. Seasonal unemployment occurs when more people
¾ To estimate employment and unemployment indicators are employed in a job than is actually needed.
in both ‘Usual Status’ (ps+ss) and CWS in both rural
and urban areas annually. 2. Disguised unemployment results from a mismatch
• The Periodic Labour Force Survey (PLFS) gives estimates between available jobs and the skills of the workforce.
of key employment and unemployment Indicators like 3. Vulnerable unemployment is the time gap between
the Labour Force Participation Rate (LFPR), Worker leaving one job and finding another.
Population Ratio (WPR), Unemployment Rate (UR), etc.
These indicators are defined as follows: How many of the above statements is/are correct?
¾ Labour Force Participation Rate (LFPR): LFPR is (a) Only one (b) Only two
defined as the percentage of persons in labour force
(i.e., working or seeking or available for work) in the (c) All three (d) None
population. Answer: (d)
¾ Worker Population Ratio (WPR): WPR is defined as
Descriptive Question
the percentage of employed persons in the population.
¾ Unemployment Rate (UR): UR is defined as the Q. Most of the unemployment in India is structural
percentage of persons unemployed among the persons in nature. Examine the methodology adopted to
in the labour force. compute unemployment in the country and suggest
improvements. [15 Marks] [250 Words] [UPSC CSE
¾ Current Weekly Status (CWS): The activity status 2023]
determined on the basis of a reference period of last
7 days preceding the date of survey is known as the
current weekly status (CWS) of the person.
Improvement of Measures: NATIONAL LOGISTICS POLICY
• Frequent Data Collection: Continue with the periodic and (NLP)
more frequent data collection approach introduced with the
PLFS. Why in News: India marks one year of launch of National
¾ Annualormorefrequentsurveyscanprovidepolicymakers Logistics Policy on 17th September 2023.
with more up-to-date information.
About National Logistics Policy
• Incorporate Rural Areas: Expand the coverage of PLFS
to include rural areas as well. While initially focusing on • National Logistics Policy (NLP) was launched in
urban areas helps with timely data, eventually covering both September 2022, to complement PM GatiShakti National
rural and urban regions will provide a more comprehensive Master Plan (NMP).
picture.
• PM GatiShakti NMP focuses on integrated development
• Enhance Technology Adoption: Leverage technology for of fixed infrastructure and network planning, while
data collection and processing. Implement mobile apps and NLP addresses infrastructure and logistics sector
online surveys to streamline data collection, reduce errors, development.
and speed up the reporting process.
• NLP includes process reforms, logistics service
• Incorporate Informal Sector: Recognize and include the improvement, digitization, human resource development,
informal sector in employment calculations. and skilling.

55 ECONOMY
• Vision of NLP: To drive economic growth and business Targets of NLP
competitiveness of the country through cost-effective
logistics network by leveraging best in class technology • Logistics costs have to be cut by half to be near global
and skilled manpower. benchmarks by 2030 by reducing the cost of logistics from
14-16% of GDP to global best practices of 8-9%.
Logistics • India aims to be among the top 10 in the LPI (Logistics
The term “logistics” describes the total process of Performance Index) by 2030. In the year 2018, India was
controlling the acquisition, storage, and delivery of ranked 44th in the LPI.
resources to their intended location.
• The Goal to ensure that logistical issues are minimized,
It encompasses planning, coordinating, storing, and exports grow manifold, and small industries and the
moving resources from one location to another, from the people working in them benefit significantly.
production points to consumption, distribution, or other
production points. • To create data driven decision support mechanism for
an efficient logistics ecosystem.

Comprehensive Logistics Action Plan (CLAP)

Progress on Implementation of NLP & CLAP • State Logistics Policy: To bring holistic focus on ‘logistics’
• Unified Logistics Interface Platform (ULIP): For digital in public policy at State level, States/UTs are developing
integration in logistics sector and to provide single sign State Logistics Plans (SLPs) aligned with NLP. So far, 22
to users who are trading goods and using multiple modes States have notified their respective State Logistics policies.
of transport – the Unified Logistics Interface Platform • Logistics Ease Across Different States (LEADS): An
(ULIP) was launched. indigenous logistics performance index on lines of
• Comprehensive port connectivity plan developed by the World Bank’s LPI, called ‘Logistics Ease Across
Ministry of port shipping and waterways, to address Different States (LEADS)’ index for logistics performance
last and first mile infra gaps and promote seamless monitoring across states has been developed.
movement of goods to ports. Way Forward
• Logistics Data Bank (LDB) is an application that • The rail sector suffers from many structural deficiencies
tracks and traces of EXIM cargo. greater predictability,
which have to be eliminated fast if the logistics cost has to
transparency and reliability is developed.
be halved to global benchmarks.

ECONOMY  56
¾ The railways need to have a time-table based goods • Apart from improvement in key functional areas, the size
operation. of our ports have to grow manifold.
¾ It has to become an aggregator at the source of freight, • It is time to give wings to air logistics and drastically
and disaggregator at the destination, to capture the improve the transport of high-value and perishable items.
high-value small-load business.
Thus, a robust logistics sector is necessary to attain the goal
• For decades the country has talked about eco-friendly of 5 trillion Dollar economy and create employment at a
and cost-effective inland waterways freight movement. scale that country requires.
¾ There is valuable learning available from the river
ports of China, who puts key emphasis on Port EXPECTED QUESTION FOR UPSC CSE
Infrastructure.
Descriptive Question
• Road logistics is a totally fragmented sector, where a
large chunk of truck owners have a very small fleet. Q. Discuss the significance and key provisions of the
National Logistics Policy in India. How can the policy
¾ There is a clear case for the aggregation of small
contribute to the growth of the economy and address
operators with government-supported aggregation
challenges in the logistics sector? [15 Marks] [250
apps. Similarly, there is a need for large players in
Words]
the sector to drag costs down.

PEARL MILLET CULTIVATION


Why in news: A recent study suggested that due to changing climate conditions, it is essential to reconsider the cultivation of
pearl millet (bajra) in India.

International Year of Millets

Food and Agriculture Organization (FAO) and United Nations have recognized 2023 as International Year of Millets or
IYM2023 for awareness about health and nutritional benefits of millets.

57 ECONOMY
EXPECTED QUESTION FOR UPSC CSE 3. They can thrive well in rain-fed and arid climates.
Prelims MCQ How many of the above statements are correct?
Q. With reference to Millets, consider the following
statements: (a) Only one (b) Only two

1. They are nutritionally superior to wheat and rice. (c) All three (d) None
Answer: c
2. They are gluten-free and have a low glycemic index.

INDIA’S FALLING COTTON PRODUCTION


Why in News: Cotton, a versatile crop that provides food, feed, and fibre, is facing a significant threat in India due to
declining production. This decline has been primarily attributed to the infestation of the pink bollworm (PBW).

The Bt Revolution
• Between 2000-01 and 2013-14, India witnessed a significant • G
 enetically modified (GM) crops are plants that have
had their genetic material (DNA) altered through
increase in cotton production and yields, primarily due to
genetic engineering techniques. These alterations
the adoption of Bt (Bacillus thuringiensis) technology.
are made to introduce specific desirable traits into the
¾ Average per-hectare lint (fiber) yield has more than crops.
doubled from 278 kg in 2000-01 to 566 kg in 2013-14.
 t cotton varieties, genetically modified to express a
• B
• This technology introduced genetically modified (GM) protein toxic to the cotton bollworm, were officially
cotton hybrids that were resistant to the American approved for commercial cultivation in India in the
bollworm insect, a major cotton pest. early 2000s. Bt cotton adoption has been substantial in
India due to its effectiveness in reducing pest damage
• Bt cotton hybrids led to a remarkable increase in per-
and increasing yields.
hectare lint yields.

¾ India’s cotton production, in terms of lint, almost • Lint is the fluffy, white fibre of cotton that is used for
trebled from 140 lakh to 398 lakh bales of 170 kg each textiles and fabric production.
in the same time period.

ECONOMY  58
The Pink Bollworm Challenge

• While Bt cotton remained effective against the American


bollworm, PBW began to affect cotton crops extensively
from 2014 onwards.

• PBW infestations started occurring earlier in the crop’s


growth cycle, severely impacting cotton yields.

Why PBW is a Problem


• PBW is a monophagous pest, mainly feeding on cotton, making it particularly susceptible to developing resistance to
Bt proteins over time.
• This resistance was exacerbated as farmers increasingly adopted Bt cotton and stopped growing non-Bt varieties.
• The pest’s short life cycle allowed it to quickly adapt and develop resistance to Bt cotton.

59 ECONOMY
Controlling PBW
EXPECTED QUESTIONS FOR UPSC CSE
• Traditional methods of controlling PBW, such as
insecticide spraying, proved less effective as these Prelims MCQs
larvae feed on cotton bolls, buds, and flowers.
Q1. The black cotton soil of India has been formed due to
• An alternative approach called “mating disruption”
the weathering of: [UPSC CSE 2021]
involves using synthetic pheromones to prevent PBW
moths from mating. (a) Brown forest soil    (b) Fissure volcanic rock
• Two mating disruption products, PBKnot and SPLAT, (c) Granite and schist   (d) Shale and limestone
have shown promise in controlling PBW infestations. Answer: (b)

PBKnot Technology
Q2. “The crop is subtropical in nature. A hard frost is
• PBKnot involves using pheromone-laden ropes that are injurious to it. It requires at least 210 frost-free days
attached to cotton plants. and 50 to 100 centimeters of rainfall for its growth. A
• These ropes release synthetic pheromones that confuse light well-drained soil capable of retaining moisture
male PBW moths and disrupt their mating patterns. is ideally suited for the cultivation of the crop.” Which
one of the following is the crop? [UPSC CSE 2020]
• This significantly reduces PBW populations and,
consequently, crop damage. (a) Cotton        (b) Jute

(c) Sugarcane       (d) Tea


Government initiatives for the Cotton sector in India
Answer: (a)
Amended Technology Upgradation Fund Scheme
 
(ATUFS)
Q3. Consider the following statements:
 Market Access Initiative (MAI) Scheme
1. The Bt cotton can resist both American bollworm
SAMARTH (Scheme for Capacity Building in the
  and Pink bollworm (PBW).
Textile Sector)
2. Pink bollworm (PBW) does not feed on any other
 Mega Investment Textiles Parks (MITRA) plant except cotton.
 Production Linked Incentive (PLI) Scheme to promote 3. PBKnot technology can be used to prevent Pink
the production of MMF Apparel, MMF Fabrics and bollworm (PBW) infestation.
Products of Technical Textiles
How many of the above statements are correct?
 Cotton in India provides direct livelihood to 6 million
farmers and indirectly employs about 40-50 million (a) Only one    (b) Only two
people in its allied processes such as trade and
processing. (c) All three    (d) None
Way Forward Answer: (a) (Only1)
• Cotton is a crucial crop for India, especially for
smallholders, but pest infestations like PBW pose a severe
Descriptive Question
threat.

• To sustain cotton production, innovative technologies Q. Examine the challenges faced by the cotton-producing
such as GM cotton, advanced insecticides, and mating farmers in the country. What measures are required to
disruption must be embraced. address these challenges? [15 Marks] [250 Words]

• These technologies are essential to safeguarding India’s For Approach Answer, Scan:
cotton industry, which plays a pivotal role in agriculture
and the textile sector.
Conclusion
The falling production is a cause of worry for many reasons
especially the textile industry and, therefore, needs urgent
attention of the government.

ECONOMY  60
¾ Even after subtracting purchases and exports, it appears
BUMPER CROP AND RISING that supply of both rice and wheat on the domestic
CEREAL PRICES market have increased during the past two years.
Why in News: The country has no grain shortage, as per What Could Be a Better Way to Control Wheat and Rice
official production statistics, but cereal prices have been Inflation (Non-PDS)?
rising at double digits since September 2022, indicating a • Reducing Import Duty
significant inflationary trend.
¾ The government could lower the import duty on
Cereal Inflation: Present Situation wheat from 40% to, say, 10%.
• Cereals and products inflation is high at 12.71 per cent. ¾ In the case of rice, dump surplus supplies on the open
• Despite the recent prohibition on exports and stocking market at lower prices than the FCI has lately done.
constraints on dealers and processors, wheat inflation is • Food and Beverage Revision in the CPI
12.37 percent, and rice inflation is 11.78 percent. ¾ There is also a need to modify the weight of food and
• The post-Covid fiscal years saw India’s rice and wheat beverages in the CPI basket, which is out of date and
offtake exceed 62.5 million tonnes, despite significant based on a consumption study conducted in 2011.
lockdowns and employment losses. ¾ This weight is presently 45.9%, with food accounting
• The PDS proved to be an efficient social safety net, with for 39%.
no shortage for the poor and vulnerable. • Engel’s law clearly indicates that as per capita income
• However, due to high cereal prices in this FY, the Centre rises, individuals spend less on food.
discontinued releasing extra grain to NFSA recipients. ¾ According to the researchers, the weight of food and
Government Measures to Control Cereal Inflation drinks in the CPI basket will be approximately 38% in
2023, while food alone will be around 33%.
• The government has banned wheat exports since May
2022, imposing stock limits and limiting wholesale ¾ The old weights have the potential to overestimate
traders and retailers to hold no more than 3,000 tonnes CPI inflation, which requires immediate modification.
of the cereal. Conclusion
• In July 2022, all white non-parboiled non-basmati rice
The paradox of bumper crops and high cereal prices in
exports were prohibited, and a 20% duty was levied on
India raises questions about the factors contributing to the
parboiled non-basmati rice exports on August.
inflationary trend, especially given the government’s efforts
Conclusions to be Drawn from the Government’s Decision to control exports and enhance domestic grain availability.
• Grain scarcity: The Center’s activities indicate that there
National Food Security Act (NFSA) 2013
is a grain scarcity both on the open market and in public
warehouses. • The NFSA was adopted to signify a paradigm change
• Can Enhance Domestic Availability: The implementation in food security policy from welfare to rights-based
of export restrictions and stockholding restrictions is approaches.
primarily intended to enhance domestic grain availability • Under the Targeted Public Distribution System (PDS),
and avoid any “hoarding and unscrupulous speculation” the Act allows up to 75% of the rural population and
that typically results from shortages. 50% of the urban population to receive subsidized
foodgrains.
Paradox of Bumper Crop and Rising Cereal Prices
• About two thirds of the population therefore is covered
• No shortage of grain, according to government estimates
under the Act to receive highly subsidised foodgrains.
¾ The results show that India’s rice production has • As a step towards women empowerment, the Act
grown by 11.2 Mt between 2020–21 and 2022–23. mandates the eldest woman of the household of age 18
years or above to be the head of the household for the
¾ Despite indications of yield losses from the heat wave
purpose of issuing ration cards.
in March 2022 and unseasonably excessive rain in
March 2023, wheat output has also increased overall The Recent Shift in NFSA Entitlement and its Effect
by 3.2 mt. • From the new calendar year (Jan 23), the NFSA
entitlement was restored to the original 5 kg/person/
• Despite no shortage of grains, cereal prices are
month level prevailing prior to April 2020 (before
increasing
COVID 19).
¾ The procurement does not reflect the greater
• The same ration cardholders now have to purchase
production. Government grain purchases have also
rice and wheat from the open market and they are
decreased, with wheat purchases falling particularly
affected by the double-digit cereal inflation.
hard.

61 ECONOMY
How Second Green Revolution can help India tackle the
EXPECTED QUESTIONS FOR UPSC CSE
problem of Nutrition?
Prelims MCQ
• India’s Green Revolution in the 1960s successfully
Q. In India, which one of the following is responsible for addressed food shortages by providing farmers with
maintaining price stability by controlling inflation? high-yielding seeds, cheap credit, and assured prices
through procurement.
(a) Department of Consumer Affairs • However, use of chemical fertilizer, reliance on
procurement prices, and a focus on cereals rather than
(b) Expenditure Management Commission
pulses limited its impact.
(c) Financial Stability and Development Council • To address this, a second agricultural revolution is
needed.
(d) Reserve Bank of India (RBI)
• Policy interventions such as procurement prices, cash
transfers, and the Public Distribution System are
Answer: (d)
insufficient.
Descriptive Question • Yield-increasing interventions on the farm are needed to
Q. Examine the relationship between “bumper crop” and contain production costs.
“rising prices” in the context of agricultural economics • Attention should be given to
in India. Discuss the various factors that influence
this relationship and analyse the implications for both ¾ extending irrigation to 100% of the net sown area
farmers and consumers. [15 Marks] [250 Words]
¾ ending land leasing restrictions

¾ accelerating agricultural research and


FOOD INSECURITY IN INDIA
¾ re-institution of extension.
Why in news: Food prices have risen significantly since
2019, reaching a record 11% in July 2023, posing potential Way Forward
hardship for some consumers in obtaining nutritious food.
• Increase public expenditure on irrigation while ensuring
Inflation impacts Nutrition efficient management.
Inflation is the rate at which prices of goods and services • Address landholding fragmentation through solutions
increase over a period of time. Inflation and nutrition go like land leasing to enhance productivity.
hand in hand. Because of the rising costs, people are buying
less expensive food, and less expensive often means less • Revitalize public agricultural research institutes for
nutritional value. innovative farming techniques.
• The ‘State of Food Security and Nutrition in the World’ of • Strengthen extension services to assist farmers in
the Food and Agriculture Organization (FAO) estimates adopting best practices.
the proportion of the population across countries unable
to afford a healthy diet. • Implement a comprehensive program to significantly
¾ As per the report, an estimated 74% of the population increase protein production in India.
cannot afford a healthy diet in India in 2021. • Foster collaboration between state and central
• The National Family Health Survey reported an increase governments, following the spirit of cooperative
in anemia, with over 50% of adult women estimated to federalism, to enhance agricultural productivity.
be anemic.
• Encourage private enterprises to contribute to food
¾ It also reported that in children below the age of five security goals with environmentally sustainable
years, 35.5% were stunted, 19.3% showed wasting, approaches.
and 32.1% were underweight.
RBI and Inflation Targeting Conclusion

• The Reserve Bank of India has failed to control inflation, Addressing malnutrition and ensuring access to healthy
leading to higher food inflation. diets for all Indians is a prerequisite for developed nation.
• RBI’s “inflation targeting” approach does not address A second Green Revolution, aligned with ecological security,
should take centre stage in India’s economic policy over the
supply-side food inflation.
coming years. Thus, holistic approach is essential to combat
• Central banks cannot solve this issue, and intervention rising food prices, enhance agricultural productivity, and
on the supply side is needed to ensure steady food secure a healthier future for the nation.
production.

ECONOMY  62
State of Food Security and Nutrition in the World (SOFI) 2023 Report
• The ‘State of Food Security and Nutrition in the World’ (SOFI) 2023 report, published by the Food and Agriculture
Organization in collaboration with United Nations agencies, highlights several key findings regarding the cost of a
healthy diet in India and its affordability compared to other countries.
• The SOFI report calculates the cost of a healthy diet by identifying the cheapest local food items, using Purchasing
Power Parity (PPP) dollars to account for differences in living costs.
Key Findings:
• The SOFI report reveals that the cost of a healthy diet in India is 3.066 PPP dollars per person per day, making it the
lowest among BRICS nations and neighboring countries.
• The report also reveals that between 2019 and 2021, the cost of a healthy diet increased by nearly 9% in Asia, with South
Asia having the highest number and 72% share of people unable to afford a healthy diet.

EXPECTED QUESTION FOR UPSC CSE card payments, mobile payments, and electronic fund
transfers.
Prelims MCQ ¾ Such person includes his overseas principal (an
individual or a company which resides or is registered
Q. The “State of Food Security and Nutrition in the World outside India and owns/manages directly or
(SOFI) 2023” report has been published by which of the indirectly activities of the payment system in India).
following?
What is a Reporting Entity?
(a) World Food Programme • A “reporting entity” under the PMLA means a banking
company, financial institution, intermediary or a person
(b) Food and Agriculture Organization (FAO) carrying on a designated business or profession.

(c) International Food Policy Research Institute • A “person” under the PMLA is an inclusive definition
which includes an individual, a firm, a company, and an
(d) Food for the Hungry agency among others.
Answer: (b) • The PMLA lays down certain obligations of reporting
entities. Some of them include:

¾ maintaining a record of all transactions,


PAYMENT SYSTEM OPERATOR ¾ keeping them confidential, and
Why in News: Online payment platform PayPal has moved ¾ verifying the identity of their clients under the
the Delhi High Court against a single judge’s order, which Aadhar Act.
ruled that it was a “payment system operator” under the
Prevention of Money Laundering Act (PMLA) and is obliged ¾ maintain information on enhanced due diligence for
to comply with reporting entity obligations under the law a period of five years.
as a result.
Arguments in Paypal’s defence:
Difference between a payment system and a payment
system operator • PayPal argued that it only provides a technological
interface for transactions and does not handle funds
• A Payment system is a system which enables payment directly.
between a payer and a beneficiary and includes systems
which enable credit/debit card operations, money • It cited the Reserve Bank of India’s stance that it is not
transfer operations or similar operations. a reporting entity under the Payments and Settlement
¾ It is a mode by which two persons/entities can Systems (PSS) Act.
transact monetarily. • PayPal explained that it serves as a technological
• The PMLA defines a “payment system operator” as platform facilitating transactions between parties, but it
a person which includes an individual, a company, a does not directly handle funds.
firm etc., who operates a payment system. They provide
the necessary infrastructure and technology to enable • The movement of funds is managed by Authorised Dealer
various payment transactions, such as online payments, (AD) banks at different stages of the transaction.

63 ECONOMY
PMLA Act UPI QR CODE-CBDC
 
Prevention of Money Laundering Act, 2002 was enacted INTEROPERABILITY
to fight against the criminal offence of legalizing the Why in News: The fusion of UPI and CBDC is an essential
income/profits from an illegal source. component of the Reserve Bank of India’s (RBI) ongoing pilot
project aimed at propelling the retail digital rupee.
 
The Prevention of Money Laundering Act, 2002 enables
the Government or the public authority to confiscate Interoperability
the property earned from the illegally gained proceeds.
• Interoperability refers to the technical compatibility that
 Any person who directly or indirectly: allows a payment system to work with other payment
systems.
¾ Attempts to indulge. Assists the person who is • In the context of the Reserve Bank of India (RBI),
actually involved in any process is a party to the interoperability enables the use of multiple payment
activity connected with the proceeds of crime. systems for clearing and settling transactions without
 The Act was formulated for the following objectives: needing to participate in multiple systems.
• Several banks, including State Bank of India, Bank of
¾ Prevent money-laundering.
Baroda, Kotak Mahindra Bank, Yes Bank, Axis Bank,
¾ Combat/prevent channelising of money into illegal HDFC Bank, and IDFC First Bank, have introduced UPI
activities and economic crimes. interoperability on their digital rupee applications.
¾ Provide for the confiscation of property derived
from, or involved/used in, money-laundering.
¾ Provide for matters connected and incidental to the
acts of money laundering.
 Actions that can be initiated against persons involved
in ML:
¾ Seizure/freezing of property and records and
attachment of property obtained with the proceeds
of crime.
 Any person who commits the offence of money
laundering shall be punishable with –
¾ Rigorous imprisonment for a minimum term of
three years and this may extend up to seven years.
¾ Fine (without any limit).

EXPECTED QUESTION FOR UPSC CSE

Prelims MCQ

Q. 
With reference to the ‘Payment system operator,’
consider the following statements:

1. 
It is an individual who operates a payment system
in financial institutions.
2. 
It is defined under the Prevention of Money QR Code
Laundering Act (PMLA).
A Quick Response (QR) code is a square grid of black
 
3. 
It is a payment system which enables payment
squares on a white background that can be read by
between a payer and a beneficiary.
imaging devices like cameras.
How many of the above statements are correct?
It contains information about the item to which it
 
(a) Only one     (b) Only two is attached and serves as a contactless channel for
payments.
(c) All three      (d) None
Merchants can accept payments from customers directly
 
Answer: b (1 and 2 only) into their bank accounts using QR codes.

ECONOMY  64
Boosting CBDC Adoption INDIA POST PAYMENTS BANK
• UPI is a widely used payment method, and (IPPB)
interoperability with CBDC is expected to drive the
Why in News: The Indian Post Payments Bank (IPPB) was
adoption of the digital rupee and increasing the volume
launched in September 2018, and now, it has been five years
of transactions.
since its inception [beginning].
• Currently, more than 70 mobile apps and over 50 million • IPPB has played a pivotal role in enhancing financial
merchants accept UPI payments. inclusion over the past five years.
• The integration of CBDC with UPI is seen as a game- About IPPB
changer for the digital currency ecosystem. • In 2018, IPPB was established under the Department
of Posts, Ministry of Communication, with 100%
Significance of Interoperability of UPI QR Code-CBDC government equity.
• Vision: To create the most accessible, affordable, and
• Improved User Experience
trusted bank for India’s common people.
• Accelerated Adoption of the Digital Rupee • IPPB operates as a Scheduled Payment Bank under the
regulatory framework of the Banking Regulation Act,
• Merchant-Friendly 1949 and with RBI’s licensing.
• IPPB offers a suite of financial products, including savings
• Increasing Financial Inclusion and current accounts, remittances, money transfers, and
the Aadhar Enabled Payment System.
• Efficiency and Cost Savings • It caters to various customer needs through three account
types:
• Promotes Innovation
¾ Safal (Regular account)
• Fosters Competition ¾ Sugam [(Basic Savings Bank Deposit Account
(BSBDA)]
Conclusion ¾ Saral (BSBDA-Small)

The compatibility of UPI and CBDC signifies a watershed Performance of IPPB


moment in the history of digital payments. The retail digital • IPPB’s accomplishments are noteworthy, with over 6
rupee is poised to acquire widespread adoption as a result of crore accounts opened nationwide, including 96 lakh
the merger of two formidable platforms, revolutionising the accounts in aspirational districts.
landscape of digital transactions in India. • To facilitate seamless banking services, 1.90 lakh postmen
and Gramin Dak Sevaks have been equipped with
EXPECTED QUESTION FOR UPSC CSE smartphones and biometric devices.
• The Post Offices, totalling 1.37 lakh banking access points
Prelims MCQ across India, have become key conduits for financial
services.
Q. Consider the following statements:
Benefits of IPPB
1. Central Bank Digital Currency (CBDC) uses
Distributed Ledger Technology as its underlying  
Expansion of banking in rural areas through its wide
technology. network of branches across India.
 IPPB acts as a catalyst for social and financial inclusion
2 Unified Payments Interface (UPI) QR codes can be through the vast network of post offices throughout the
used with Central Bank Digital Currency (CBDC) nation.
applications.
 
Last mile delivery of services through the postman -
Which of the statements given above is/are correct? and ‘Grameen DakSewaks’ acting as Mobile bankers –
providing “banking at doorstep”.
(a) 1 only      (b) 2 only  
Push to rural MSMEs as they benefit from financial
services offered by IPPB.
(c) Both 1 and 2    (d) Neither 1 nor 2
 
Effective DBT: IPPB would give a boost to Government’s
Answer: c initiatives like promoting digital transactions and Direct
Benefits Transfer (DBT).

65 ECONOMY
 
Credibility: It has credibility as the local postman is Conclusion
still an integral part of the day-to-day lives of the rural
In order to make IPPB a game changer and to sustain
populace.
competition it requires efficient infrastructure, autonomy
Challenges in funds management, training and skilling of the postal
• Despite its successes, IPPB and other Payment Banks face employees and promoting use of cashless transactions
competition from private financial entities. among rural masses.
• They grapple with the challenges of low financial literacy
EXPECTED QUESTIONS FOR UPSC CSE
and inadequate infrastructure in rural areas.
• Strict Regulation: Given the severe restrictions imposed Prelims MCQ
by the RBI on how it can employ its funds, the odds seem
to be stacked against the IPPB at the moment. Q. 
With reference to Payment Banks (PBs) in India,
consider the following statements:
• User Charges: To generate revenues, it plans to charge
fees on money transfers and other financial services 1. 
These are based on the recommendations of
which may act as a disincentive for rural customers. Nachiket Mor committee.
• IPPB will also require constant usage and upgradation of 2. They can accept fixed deposits and issue credit
technology which remains a challenge. cards.
Which of the statements given above is/are correct?
Payment Banks (PBs) in India
¾ PBs were conceived based on recommendations from (a) 1 only       (b) 2 only
the Nachiket Mor committee (2013), with the aim of
(c) Both 1 and 2     (d) Neither 1 nor 2
serving lower-income groups and small businesses.
¾ PBs are authorized to accept demand deposits Answer: a
up to Rs. 1 lakh, facilitate mobile payments/
purchases, and offer services like ATM/debit cards,
Descriptive Question
net banking, and third-party fund transfers.
¾ However, they are restricted from accepting fixed Q. Can India Post Payment Bank really improve financial
deposits, term deposits, recurring deposits, any NRI inclusion in our country? Analyse. [10 marks] [150
deposits, advancing loans, or issuing credit cards. words]

SELF-REGULATORY ORGANISATION (SRO) FOR FINTECHS


Why in News: The Reserve Bank of India (RBI) has called upon fintech entities to establish a Self-Regulatory Organisation
(SRO) to oversee the industry.

Need for Regulation


• Lack of International Standards: There are no clear-cut international regulatory standards or organizations for FinTech.
• Challenges of Rapid Tech Evolution: Regulating FinTech is challenging due to the rapidly changing technology
landscape, including cryptocurrencies, machine learning, and AI.
• Data Security and Privacy: Risks include platform downtimes and information theft impacting financial services.

ECONOMY  66
• Concerns About Finance Products: Some FinTech • Ethical Business Practices: SROs encourage member
platforms charge exorbitant interest rates, raising ethical organizations to adhere to a standard code of conduct,
concerns. promoting ethical business practices and enhancing
• Urban-Rural Divide: FinTech platforms are concentrated confidence in the industry.
in urban areas, leaving rural regions with financial • Watchdog Role: SROs act as watchdogs, guarding
literacy challenges. against unprofessional practices within the industry or
• Regulation is necessary to address arbitrage, ensure profession.
compliance with existing laws, and adapt to technological • Customising standards and promoting responsible
advancements. innovation: It will dynamically adapt to industry
developments to foster a competitive, diverse ecosystem
What is an SRO?
while emphasizing compliance in areas such as consumer
• A Self-Regulatory Organisation (SRO) is a non- protection, cybersecurity, etc.
governmental entity responsible for setting and enforcing
rules and standards governing the conduct of industry Key Concerns
members.  One of the primary challenges in self-regulating the
• Objectives: Promoting ethics, Equality, professionalism, fintech sector lies in the complex task of striking a
Responsible practices, Protection against systemic risks, careful balance between profitability and adhering to a
and Global competitiveness, enhance transparency, fair customer-centric approach.
competition, and consumer protection in the FinTech
sector.  
In addition, they also seek further clarity on the
impartiality of the organisation and its assurance on
• Functions addressing potential compliance gaps.
¾ Communication Channel: Serve as intermediaries
between their members and RBI.  Fintech companies may lack the incentive or resources
to enforce regulations rigorously, leading to compliance
¾ Standard Establishment: Establishing minimum
gaps.
benchmarks and standards while promoting
professional market behavior among members.  The rapidly evolving nature of fintech can make it difficult
¾ Training and Awareness: Offer training to its for self-regulatory bodies to keep up and establish
members and conduct awareness programs on appropriate guidelines. Smaller fintech startups may
industry-related matters. struggle with compliance costs associated with self-
¾ Grievance Redressal: They establish uniform regulation.
grievance redressal and dispute management Conclusion
frameworks across their members.
This highlights the urgent need to ensure that innovations
• Importance are accompanied by prudential safeguards and responsible
¾ Financial Inclusion conduct. It is also imperative that regulated entities operate
¾ Innovation and Disruption within the perimeter set by the licensing conditions and
only undertake activities which are permitted under the
¾ Efficiency and Cost Reduction
regulations. It will enhance the industry standards, protect
¾ Improved Customer Experience consumers, and promote responsible practices.
¾ Innovative Payment Solutions
¾ Access to Capital for Small Businesses EXPECTED QUESTION FOR UPSC CSE
¾ Data-Driven Insights and Personalization Prelims MCQ
RBI’s Expectations from Fintech Players
Q. Consider the following statements with reference to
• Develop industry best practices, privacy and data the Financial Technology (Fintech) Sector in India:
protection norms in compliance with local laws
1. It contributes to 30% of the GDP of India.
• Establish standards to prevent mis-spelling and promote
ethical business practices and pricing transparency. 2. It is contributing more than 4o% to the total
employment in India.
• It could play a vital role in facilitating collaboration
among companies, regulators, and stakeholders. Which of the statements given above is/are correct?
• Establish an SRO to ensure responsible self-regulation.
(a) 1 only      (b) 2 only
Benefits of an SRO
• Industry Expertise: SROs possess in-depth knowledge (c) Both 1 and 2    (d) Neither 1 nor 2
of their respective industries, making them valuable Answer: (c)
participants in deliberations and policy formulation.

67 ECONOMY
RBI REMOVES INCREMENTAL CRR Impact of Introduction of ICRR

Why in News: The Reserve Bank of India (RBI) recently • It will reduce the amount of money that banks have to
announced its decision to phase out the incremental cash lend, which could lead to higher interest rates.
reserve ratio (I-CRR) in response to evolving liquidity • Higher interest rates could make it more expensive for
conditions. businesses to borrow money, which could slow down
growth.
RBI’s Announcement
• The RBI, after reviewing the liquidity situation, has opted • It could probably make it more difficult for banks to
to discontinue the I-CRR gradually. This decision aims raise capital, and it could impact their profitability.
to prevent sudden shocks to the system and ensure the
• ICRR is also expected to help to stabilise the rupee,
orderly functioning of money markets.
which has been under pressure in recent months.
• The central bank will release the funds maintained by
• It could also help to reduce inflation, which has been
banks under the I-CRR in three stages.
rising recently.
• The I-CRR measure was expected to absorb over Rs 1
• ICRR could also strengthen the rupee making imports
lakh crore of excess liquidity from the banking system,
cheaper, which help to reduce inflation.
contributing to a temporary liquidity deficit.
• However, a stronger rupee also makes exports more
Reasons for Introduction: expensive, hitting export sectors and growth.
• The I-CRR was introduced in August, 2023, banks need
to maintain an I-CRR of 10% on the increase in their net Impact of Removing ICRR on Banks
demand and time liabilities (NDTL).
 Scheduled Commercial Banks
• The I-CRR was introduced as a temporary measure to
absorb excess liquidity in the banking system, driven For commercial banks, the phased reduction in I-CRR
by factors such as the return of Rs 2,000 banknotes, would likely mean more funds available for lending,
RBI’s surplus transfers to the government, increased thereby potentially increasing their net interest
government spending, and capital inflows. margin.

• The central bank recognized that excessive liquidity  Regional Rural Banks
could pose risks to price stability and financial stability.
This group may benefit from increased liquidity, which
Consequently, the I-CRR aimed to manage this liquidity
they can then invest in rural development schemes.
efficiently and take corrective measures when necessary.
 Co-operative Banks
Net Demand and Time liabilities (NDTL)

Like the others, co-operative banks will also have
NDTL is the difference between the sum of demand and increased liquidity which they can use for providing
time liabilities (deposits) of a bank (with the public or the more loans to their community-based customer base.
other bank) and the deposits in the form of assets held by
the other banks.

CRR and I-CRR

• Cash reserve Ratio (CRR) - Banks are required to


maintain liquid cash amounting to a certain proportion
of their deposits and certain other liabilities with the RBI.
This is a tool at the disposal of the RBI to control the
liquidity in the economy and can also act as a buffer
in periods of bank stress. Banks are currently required
to maintain 4.5 per cent of their Net Demand and Time
Liabilities as CRR with the RBI. Conclusion
• Incremental CRR is an additional CRR that banks have The RBI’s decision to discontinue I-CRR reflects its efforts to
to maintain over and above the standard CRR. It was manage liquidity conditions in a way that avoids sudden
designed as a monetary policy tool to manage excess shocks to the banking system. This move is expected to
liquidity in the financial system. What this means is that provide banks with sufficient liquidity to accommodate
banks will now have to park more liquid cash with the increased credit demand during the upcoming festival
RBI. season and maintain the stability of the Indian financial
system.

ECONOMY  68
labour force participation has not been fully realized,
EXPECTED QUESTIONS FOR UPSC CSE
suggesting room for growth and improvement.
Prelims MCQ • Global Demand Shift: The demand for online gig
workers is growing at a faster rate in developing nations
Q1. Which of the following is the temporary measure compared to industrialized ones, indicating shifting
introduced by the Reserve Bank of India (RBI) to dynamics in the gig economy.
absorb excess liquidity in the banking system?
Benefits of online gig work
(a) Incremental Cash Reserve Ratio (I-CRR)
¾ A flexible workforce help businesses and startups to
(b) Repo Rate remain active and agile and competitive in dynamic
market
(c) Marginal Standing Facility (MSF) Rate
¾ Create new jobs and tackle the menace of
(d) Cash Reserve Ratio (CRR) underemployment and unemployment
Answer: a ¾ Will promote skilling for youth as they are attracted
gig work due to its flexibility to combine gig work
with school or another job
Descriptive Question
¾ Promote women’s participation in online gig
workforce
Q. Describe qualitative and quantitative tools available
with RBI to manage liquidity in the Indian economy. ¾ It can help expand opportunities in rural areas as
[10 marks] [150 words] online work does not have restrictions of location.
E.g., 6 out of 10 online gig workers live in small cities
Challenges of online gig work
WORLD BANK’S ONLINE GIG • Workers are outside the social security net and lack
WORK REPORT workplace entitlements etc.
• Many workers might face unclear career pathway which
Why in News: The World Bank has released a report titled
can create challenges for them in long run
“Working Without Borders: The Promise and Peril of
Online Gig Work.” This report explores the landscape of • A considerable wage gap also exists, where women are
online gig work. paid less than their male counterparts
• Lack of job security, irregular wages, and uncertain
What is Online gig work?
employment status
 It is a work that is performed and delivered online • Rising stress due to uncertainty associated with
independent of location. irregularity in available work and income
• Limited access to the internet and digital technology
 These jobs now account for as much as 12% of global
labour force. What can government and private sector do?

 
It involves individuals working as independent • Expand social protection coverage for all online gig
contractors or freelancers on digital platforms for workers. E.g. India implementing Code on social security
short-term assignments. 2020

Key Findings of the World Bank report • Some private firms are developing solutions for tax
planning, savings, financial access for their gig workers
• Global Presence: Online gig workers make up a
significant portion of the global workforce, ranging from • Enhance investment in digital infrastructure to create
4.4% to 12.5%. opportunities in rural and far-flung areas for online work
• Regional Platforms: Most online gig work platforms
• Government can collect data and monitor labour market
tend to be regional or local in nature. This regional focus
trends to formulate effective policies
suggests that gig work often aligns with local needs and
preferences. • Supporting small businesses & entrepreneurs associated
• Gender Inclusivity: Globally, women are actively with platforms
participating in online gig work to a greater extent than
• Replicating the best practices such as Rajasthan Gig
in the general labour market. The report points to the
workers (Registration and Welfare) Act, 2023 which
potential of online gig work to empower female workers.
provides social security, grievance redressal and
• Untapped Potential: Despite women’s participation, the provision for welfare cess for gig workers.
full potential of online gig work in supporting female

69 ECONOMY
Conclusion EXPECTED QUESTIONS FOR UPSC CSE

Developing countries must leverage this growing part Prelims MCQ


of labour market to create more opportunity and expand
Q. Consider the following statements with reference to
economic inclusivity and increase prosperity for their people. the “Gig Economy”:
The report’s recommendations provided aim to address these
1. 
Gig workers include self-employed, freelancers,
challenges and maximize the positive impact of online gig independent contributors and part-time workers.
work on workers and economies.
2. It is tech-enabled platforms connect the consumer to
the gig worker to hire services on a short-term basis.

ECONOMY  70
international cooperation in the investigation and prosecution
Which of the statements given above is/are correct?
of economic offences. For instance, fugitive Nirav Modi was
(a) 1 only       (b) 2 only arrested in the UK and is currently facing extradition to India
for his alleged involvement in the PNB scam.
(c) Both 1 and 2     (d) Neither 1 nor 2 4. 
Transparency and Accountability: The act has helped
Answer: c in bringing transparency and accountability as the act
requires economic offenders to disclose all their assets,
Descriptive Question which helps in tracking and confiscating their properties.
Q. Discuss the impact of the gig economy on labor market
Criticisms of the Act
dynamics, social security, and economic growth in India.
[10 Marks] [150 Marks] 1. Under it, any court or tribunal may prohibit FEO from
initiating or defending civil cases in its court or tribunal.
¾ It would violate Article 21 of the Constitution, which
FUGITIVE ECONOMIC OFFENDER protects the right to life. The interpretation of Article
Why in News: Recently, the Bombay high court rejected a 21 includes the right to obtain justice.
plea filed by an absconding diamond trader seeking dismissal 2. Before conducting a search, the Act does not require
of the Enforcement Directorate (ED) proceedings against him authorities to acquire a search warrant or verify the
under the Fugitive Economic Offenders (FEO) Act, 2018. presence of witnesses. So, there is no protection against
harassment and abuses such as evidence planting.
Fugitive Economic Offender (FEO)
A Fugitive Economic Offender (FEO) is an individual
  3. It permits the forfeiture of an FEO’s property. So, no
against whom an arrest warrant has been issued for time period to defend oneself is given, which leads to
their involvement in economic offenses. jeopardizing the defendant’s legal interests and is against
due process.
 These economic offenses must involve at least Rs. 100
crores in value (as per FEO Act, 2018). These criticisms can hinder the process of bringing FEOs
 The person becomes an FEO when they leave India to to justice and recovering their assets. So, these must be
evade criminal prosecution. effectively tackled with cooperation with other international
partners and updated law.
About FEO Act, 2018:
EXPECTED QUESTION FOR UPSC CSE
• Act outlines the legal framework for dealing with FEO cases.
• Scheduled Offences under this act include counterfeiting Prelims MCQ
government stamps or currency, cheque dishonour, Q. 
With reference to the ‘Fugitive Economic Offender
transactions defrauding creditors, money laundering, Act, 2018, consider the following statements:
and similar economic offenses. 1. Financial offenses worth at least Rs. 100 crore or
• To declare an individual as an FEO, an application must be more come under the purview of this Act.
filed in a Special Court designated under the Prevention
2. 
It designates a person as a Fugitive Economic
of Money-Laundering Act, 2002. Offender when they leave India to evade criminal
• The Enforcement Directorate (ED) enforces this law. prosecution.
• Authorities are empowered to confiscate assets even 3. 
It only authorizes Supreme Court to declare a
without obtaining a conviction, and the rights and titles of person as a Fugitive Economic Offender.
the confiscated property belong to the central government.
How many of the above statements are correct?
• Appeals against the orders of the special court can be
made to the High Court. (a) Only one    (b) Only two
• The act aims to tackle economic offenses and prevent (c) All three     (d) None
financial fraud.
Answer: b (1 and 2 only)

Gains from the Act


1. 
Recovering Loan Defaults: The key provision of CRYPTO REGULATION DISCUSSION
confiscation of properties has helped in recovering
loan defaults. For instance, in 2019, the Enforcement
AT G20
Directorate (ED) confiscated assets worth over Rs 18,000 Why in News: India is working with the Financial
crore belonging to fugitive economic offender Vijay Stability Board (FSB) to reach a consensus on regulating
Mallya. cryptocurrencies during the country’s G20 presidency itself.
2. 
A deterrence for Economic Offenders: The act has acted as a Crypto-Asset Regulation
strong deterrence for economic offenders from absconding.
• At the request of the Indian G20 Presidency, the
3. 
International Cooperation: The act has mechanism for International Monetary Fund (IMF) and the Financial

71 ECONOMY
Stability Board (FSB) released a policy paper • The market exchange rate, which represents the
recommending against an outright ban on crypto-assets. equilibrium price where currency supply equals
• Instead, the paper proposes the implementation of a demand, is crucial to understanding this concept.
licensing regime for crypto-asset platforms, subjecting • When a government sets a fixed exchange rate below
them to anti-money laundering and counter-terrorist the market rate, the currency’s supply decreases, and
financing standards. demand for it increases.
• Consequently, this can lead to a shortage of currency,
India’s stance
with demand exceeding supply.
In 2018, the Central Board of Direct Taxes proposed banning
virtual currencies, and the Reserve Bank of India (RBI) later Currency Exchange Rates
barred banks from dealing in cryptocurrencies.  An exchange rate is a rate at which one currency will be
• However, this banking restriction was overturned by the exchanged for another currency and affects trade and
Supreme Court in 2020. the movement of money between countries.
• Despite this, the RBI has expressed concerns about  Exchange rates can be either fixed or floating.
crypto-assets, labelling them a “macro-economic risk.”
 Fixed exchange rates are decided by central banks of
• There is a need for international collaboration due to the a country whereas floating exchange rates are decided
borderless nature of digital currencies. by the mechanism of market demand and supply.
Way Forward Gresham’s Law in Sri Lanka
• India’s approach during its G20 presidency reflects its • The Sri Lankan economic crisis of last year provides a
commitment to finding common ground on global issues. practical example of Gresham’s law in action.
• Whether it’s shaping a global regulatory structure for • During this crisis, the Central Bank of Sri Lanka fixed
crypto-assets or navigating climate change discussions, the exchange rate between the Sri Lankan rupee and the
India’s willingness to adapt and facilitate dialogue U.S. dollar.
highlights its role in seeking consensus and cooperation • The government mandated that the price of the U.S.
on the international stage. dollar in terms of the Sri Lankan rupee should not
exceed 200 rupees per dollar, even though the black
EXPECTED QUESTION FOR UPSC CSE market indicated a much higher exchange rate.
Prelims MCQ • This effectively restricted people from paying more than
200 Sri Lankan rupees for a U.S. dollar, causing the rupee
Q. Consider the following statements: to be overvalued and the U.S. dollar to be undervalued
1. 
Cryptocurrencies are subjected to income tax in compared to the market rate.
India. • This policy led to a decrease in the supply of dollars in
2. 
Financial institutions that are regulated by the the market and resulted in the gradual exclusion of the
Reserve Bank of India (RBI) are banned from U.S. dollar from the formal foreign exchange market.
dealing in cryptocurrencies. • Individuals seeking U.S. dollars for foreign transactions had to
resort to the black market, where they had to pay significantly
3. 
Central Bank Digital Currency (CBDC) is a more than 200 Sri Lankan rupees for each U.S. dollar.
cryptocurrency issued by RBI.
Conclusion
How many of the above statements given above is/are
correct? Gresham’s law states that “bad money drives out good” and is
a monetary principle that can be applied to the currency markets.
(a) Only one     (b) Only two During the historical use of precious metals to manufacture
(c) All three      (d) None coins, Gresham’s law applied to the changing value of coins and
Answer: a (Only 1) their contents. Since the global financial system has transitioned
to fiat currencies, examples of Gresham’s law are rare.
EXPECTED QUESTION FOR UPSC CSE
GRESHAM’S LAW Prelims MCQ
Why in News: The recent economic crisis in Sri Lanka last Q. The phenomenon where government-fixed low
year brought Gresham’s law into the spotlight. commodity prices lead to their disappearance from the
About Gresham’s Law and it’s applicability formal market is associated with which of the following?
(a) Principles of Comparative Advantage
• Gresham’s law comes into play when a government fixes
Answer: b

the exchange rate of a currency below its market value. (b) Gresham’s Law
• This causes the undervalued currency to disappear from (c) Law of Competition
circulation, while the overvalued currency remains in
(d) Law of Supply and Demand 
circulation but struggles to find buyers.

ECONOMY  72
SECTION

D ENVIRONMENT & GEOGRAPHY

GLOBAL BIOFUELS ALLIANCE About Global Biofuel Alliance (GBA)


¾ An impressive total of 19 nations and 12 international
Why in News: Under the leadership of India as the G20 organizations have agreed to join GBA.
Chair, a groundbreaking initiative known as the Global
Biofuel Alliance (GBA) has been launched. ¾ The alliance will also act as a central repository of
knowledge and an expert hub.
• It would attempt to bring countries together to co-
develop, accelerate technological advances in production ¾ GBA aims to serve as a catalytic platform, fostering
processes, intensifying utilization of sustainable biofuels, global collaboration for the advancement and
shaping robust standard setting and certification through widespread adoption of biofuels.
the participation of a wide spectrum of stakeholders.

What are Biofuels? • Environmental Benefits: Biofuels release fewer carbon


Any hydrocarbon fuel that is produced from an organic emissions compared to fossil fuels.
matter (living or once living material) in a short period of • Waste Management: Biofuels can help manage municipal
time (days, weeks, or even months) is considered a biofuel. solid waste by converting waste into fuel.
Biofuels may be solid, liquid or gaseous in nature.
• Energy Security: Biofuels can be produced locally,
• Solid: Wood, dried plant material, and manure reducing a nation’s dependence on foreign energy
sources.
• Liquid: Bioethanol and Biodiesel
• Economic Growth: Biofuel manufacturing plants create
• Gaseous: Biogas jobs, particularly in rural areas, employing hundreds or
thousands of workers.
Biofuels: Significance
• Addressing Key Concerns: Rising oil prices, greenhouse
• Renewable Availability: Biofuels offer a sustainable gas emissions from fossil fuels, and supporting
alternative to finite fossil fuels, addressing concerns agricultural communities are significant motivations for
about resource depletion. transitioning to biofuels.
• Diverse Source Material: Biofuels can be produced from Recent Initiatives to Promote Biofuels in India
a wide range of materials, including crop waste and
• Initiatives by Department of Biotechnology, Ministry of
manure.
Science and Technology:

ENVIRONMENT & GEOGRAPHY  73


¾ The department successfully developed 2G Ethanol
indicates a viability gap funding scheme for 2G
and transferred the technology to Oil Marketing
ethanol Bio refineries of Rs.5000 crore in 6 years
Companies (OMCs). in addition to additional tax incentives, higher
¾ Developed Indigenous Cellulolytic Enzyme for the purchase price as compared to 1G biofuels. The
production of biofuels. Policy encourages setting up of supply chain
¾ It has strengthened the international collaboration to mechanisms for biodiesel production from non-
accelerate innovation in Sustainable Biofuel through edible oilseeds, Used Cooking Oil, short gestation
crops.
multilateral programs like Mission Innovation and
Biofuture Platform.
 oles and responsibilities of all the concerned
R
• Pradhan Mantri JI-VAN Yojana, 2019: The scheme is to Ministries/Departments with respect to biofuels
create an ecosystem for setting up commercial projects have also been captured in the Policy.
and to boost Research and Development in 2G Ethanol
sector.
Conclusion
• Ethanol blending: The 2018 Biofuel Policy has the • The launch of the Global Biofuels Alliance marks a
objective of reaching 20% ethanol-blending and 5% watershed moment in our quest for sustainability and
biodiesel-blending by the year 2030. clean energy.
¾ Recently, instead of 2030, the Centre plans to move
ahead with its ethanol blending target of 20% of • The G20 Leaders’ Declaration said that the member
petrol containing ethanol by 2025-26. countries “recognize the importance of sustainable
biofuels in our zero and low- emission development
• GOBAR (Galvanizing Organic Bio-Agro Resources) strategies, and note the setting up of a Global Biofuels
DHAN scheme, 2018: Focuses on managing and Alliance”.
converting cattle dung and solid waste in farms to useful
compost, biogas and bio-CNG, thus keeping villages
EXPECTED QUESTIONS FOR UPSC CSE
clean and increasing the income of rural households. It
was launched under Swachh Bharat Mission (Gramin).
Prelims MCQ
• Repurpose Used Cooking Oil (RUCO): It was launched
by Food Safety and Standards Authority of India (FSSAI) Q. Consider the following countries:
and aims for an ecosystem that will enable the collection
and conversion of used cooking oil to biodiesel. 1. India                   2.USA

3. Brazil                   4.Argentina
India’s National Policy on Biofuels, 2018
5. South Africa
 he Policy categorises biofuels as “Basic Biofuels”
T
viz. First Generation (1G) bioethanol & biodiesel
Which of the above are the members of the Global
and “Advanced Biofuels” - Second Generation (2G)
Biofuels Alliance (GBA)?
ethanol, Municipal Solid Waste (MSW) to drop-in
fuels, Third Generation (3G) biofuels, bio-CNG etc.
to enable extension of appropriate financial and (a) 1, 2 and 3 only           (b) 2, 4 and 5 only
fiscal incentives under each category.
(c) 1, 2 and 5 only             (d) 1, 2, 3, 4 and 5
 It expands the scope of raw material for ethanol Answer: d
production by allowing use of sugarcane juice,
sugar containing materials like sugar beet, sweet
sorghum, starch containing materials like corn, Descriptive Questions
cassava, damaged food grains like wheat, broken
rice, rotten potatoes, unfit for human consumption Q1. Enumerate the categories of Biofuels as per National
for ethanol production. Policy on Biofuels. Also list the other measures
by government of India to promote biofuels. [10
 he Policy allows use of surplus food grains for
T Marks] [150 Words]
production of ethanol for blending with petrol
with the approval of National Biofuel Coordination Q2. Critically analyse the relevance of Global Biofuel
Committee. Alliance for tackling emission of greenhouse gases
and promoting sustainable development. [15
 With a thrust on Advanced Biofuels, the Policy
 Marks] [250 words]

74 ENVIRONMENT & GEOGRAPHY


ONE YEAR OF PROJECT CHEETAH

ENVIRONMENT & GEOGRAPHY  75


EXPECTED QUESTION FOR UPSC CSE • 
The government committed to meet 50% of its
electric power needs from renewable, non-fossil
Prelims MCQ fuel energy sources — up from 40% committed at
the Paris agreement.
Q. With reference to Cheetahs, consider the following
statements: • It promised to create an additional carbon sink of
2.5 to 3 billion tonnes of Co2-equivalent [GtCO2e]
1. Asiatic Cheetah is slightly smaller than African through additional forest and tree cover by 2030.
Cheetah.
• India pledged to achieve for net zero emissions by
2. Asiatic Cheetah only occurs in India, Pakistan and 2070.
Bangladesh.
3. 
African Cheetah is declared as Critically EXPECTED QUESTION FOR UPSC CSE
Endangered on IUCN Red list.
How many of the above statements are correct? Prelims MCQ
(a) Only one    (b) Only two Q. 
With reference to the Climate Ambition Summit
(c) All three    (d) None (CAS), consider the following statements:
Answer: a (1 only)
1. It promotes new commitments by the countries in
line with the Paris Agreement.
CLIMATE AMBITION SUMMIT 2. It is convened time to time by the United Nations
General Assembly (UNGA).
Why in News: Recently, the UN Climate Ambition Summit
(CAS) was held at United Nations Headquarters, New York. Which of the statements given above is/are correct?
• China, the U.S. and India — which collectively account
(a) 1 Only     (b) 2 Only
for about 42% of global greenhouse gas emissions and
are the top three emitters in that order — were all absent (c) Both 1 and 2    (d) Neither 1 nor 2
from the summit.
About Climate Ambition Summit (CAS) Answer: c
• It is a prominent international event aimed at addressing
the pressing issue of climate change.
PLANETARY BOUNDARIES
• Aim: To accelerate climate action as a prelude to the 28th
Conference of Parties (COP28) to the United Nations Why in News: As per a study, six out of nine planetary
Framework Convention on Climate Change (UNFCCC) boundaries, that make Earth healthy and habitable have
and to uphold the Paris Agreement’s 1.5°C temperature been transgressed because of human-induced pollution and
limit. destruction of the natural world.

Paris Agreement Key details:


The broken boundaries show Earth’s life-support systems
It was adopted at the COP 21 in Paris in December
 
2015. have been driven far away from the safe operating space for
humanity that existed during the Holocene period.
It is a legally binding international treaty on climate
 
change. ¾ The period started with the end of the last ice age
Target: To limit global warming to well below 2°
  and the start of the Industrial Revolution.
Celsius, and preferably limit it to 1.5° Celsius, ¾ It’s characterised by relatively stable and warm
compared to pre-industrial levels. planetary conditions.
What are the planetary boundaries?
India’s Updated Intended Nationally Determined
Contributions (INDCs) under Paris Agreement • Planetary boundaries environmental limits within
which humanity can safely operate to maintain Earth’s
• India updated its climate pledges in 2022 of stability and biodiversity.
reducing emissions intensity (the volume of • There are 9 such boundaries.
emissions per unit of gross domestic product
• For each of the boundaries, control variables are chosen
(GDP)) by 45% from 2005 levels by 2030, a 10%
to capture the most important anthropogenic influence
increase from what it agreed to in 2015.
at the planetary level of the boundary in focus.

76 ENVIRONMENT & GEOGRAPHY


Boundaries and their control variables: reservoirs) and green water (available in the soil for
• Biosphere integrity: plants and soil microorganisms)
¾ The health of ecosystems and rate of extinction of • Atmospheric aerosol loading:
species.
• Climate change: ¾ Tracking various particles from anthropogenic
¾ Atmospheric CO2 concentration and the change emissions that affect cloud formation as well as
in radiative forcing — a measure of the balance of global and regional atmospheric circulation.
energy from sunlight that hits Earth, in comparison • Ocean acidification:
with thermal energy the planet loses.
• Novel entities: ¾ Reduction in the pH of the ocean over an extended
period of time.
¾ Levels of plastic, concrete, synthetic chemicals, gene-
modified organisms, etc. that would not be found on • Land system change:
Earth if we humans were not here.
¾ Changes in land use, especially the conversion of
• Stratospheric ozone depletion: tropical forests to farmland.
¾ The anthropogenic release of manufactured
chemicals that destroy ozone molecules. • Biogeochemical flow:
• Freshwater change: ¾ Alteration in the natural flows and the forms of
¾ It includes an examination of the human-induced nitrogen and phosphorus cycles, which are essential
impact on blue water (found in lakes, rivers, and elements for plant growth.

ENVIRONMENT & GEOGRAPHY  77


Findings of the study Way Forward
• Out of the nine planetary boundaries, humans have • Take conservation actions to protect biodiversity, restore
breached six: ecosystems, and safeguard endangered species and
¾ climate change, genetic variability.
¾ biosphere integrity, • Enable communities to engage in sustainable practices,
¾ freshwater change, instilling a sense of collective responsibility for
¾ land system change, environmental protection.
¾ biogeochemical flows and • Prioritize climate mitigation strategies to limit
¾ novel entities. temperature rise and prevent further breaches of the
• While atmospheric aerosol loading and ozone depletion planetary boundary related to climate change.
remain within the constraints, ocean acidification is • Encourage zero-emission technology and reduce carbon
close to being breached. emissions through incentives to adopt clean energy and
• Overflow of nitrogen and phosphorus in the sustainable transport.
environment:
EXPECTED QUESTIONS FOR UPSC CSE
¾ Although nitrogen and phosphorus are essential
for life, their widespread use as crop fertilisers is Prelims MCQ
wreaking havoc by, for instance, triggering algal
blooms (it can cause entire fish populations to leave Q. Consider the following pairs:
an area or even die) and ocean dead zones (a reduced
level of oxygen in the water).
Planetary Boundary Their control variables
• Biosphere integrity boundary:
1. Biosphere integrity Rate of extinction of species
¾ Researchers have also noted that the biosphere
integrity boundary was violated in the last 19th 2. Climate change Change in radiative forcing
century when acceleration in land use caused a
strong impact on numerous species. 3. Biogeochemical flow Alteration in the natural
flows of water cycle
• Freshwater:
¾ In the case of freshwater, the boundary was breached How many of the above pairs are correctly matched?
last century, between 1905 and 1929.
¾ Rising levels of novel entities in the environment: (a) Only one    (b) Only two
¾ Humans have been releasing unprecedented (c) All three    (d) None
amounts of synthetic chemicals like pesticides and Answer: b (1 and 2 only)
plastic, without adequate safety testing for decades.
• Land system change:
Descriptive Questions
¾ Land-use conversion and fires are causing rapid
change in forest area, and deforestation of the
Q1. What are planetary boundaries? Discuss the
Amazon tropical forest has increased such that it has
now transgressed the planetary boundary. significance of planetary boundaries in the context
of an increasingly warming world? [15 Marks] [250
• Climate change:
Words]
¾ For climate change, the analysis notes that both
the atmospheric CO2 concentration and radiative
Q2. Discuss global warming and mention its effects on
forcing are steadily increasing.
the global climate. Explain the control measures to
Can earth still recover? bring down the level of greenhouse gases that cause
• Planetary boundaries aren’t like tipping points. global warming, in the light of the Kyoto Protocol,
• Therefore, if humans do away with fossil fuel burning 1997. [10 Marks] [150 Words] [UPSC CSE 2022]
and end destructive farming, the transgressed boundaries
can be brought back into space operating space. For Approach Answer, Scan:
• Notably, the one improving boundary, the stratospheric
ozone depletion boundary, is recovering owing to the
combined international efforts initiated by the Montreal
Protocol in 1987.
• This level of coordinated change may help in dealing
with other boundaries.

78 ENVIRONMENT & GEOGRAPHY


40% of India’s petroleum product consumption, mainly
POLLUTION TAX due to its use in the transport sector.
Why in News: Recent discussions around the imposition • Geographic Concentration- Three states—Uttar Pradesh,
of a ‘pollution tax’ on diesel vehicles in India have raised Maharashtra, and Haryana—account for nearly 40% of
questions about the future of diesel fuel. India’s diesel consumption

The Proposed Pollution Tax Impact on Diesel-Run Vehicles


• Pollution tax is a tax levied on person one who pollutes • Automaker Decisions- Major car manufacturers,
or damages the environment. The main aim of this tax is including India’s largest carmaker, have ceased
to reduce environmental harm. production of diesel vehicles or scaled back their diesel
• In a recent statement, the Road Transport Minister offerings due to the challenges of meeting stringent
suggested the possibility of introducing a 10% GST hike emission norms and the associated costs.
on diesel vehicles as a ‘pollution tax.’ • Emission Concerns- Diesel engines tend to produce
• Although this proposal is not currently under active higher emissions of oxides of nitrogen (NOx) compared
consideration by the government. to petrol engines. The Volkswagen emissions scandal in
2015 further tarnished diesel’s image globally.
Challenges Faced by Diesel
• Emission Norms- The transition to BS-VI emission
• Environmental Goals- The Indian government has set norms in 2020 presented additional challenges and costs
ambitious targets to reduce greenhouse gas emissions
for automakers, making diesel vehicles less costly.
under Paris Climate Agreement.
• Diesel Consumption- Diesel accounts for approximately

ENVIRONMENT & GEOGRAPHY  79


Bharat Stage Emissions Standards
Descriptive Question
• Bharat Stage (BS) emissions standards are India’s
equivalent of the European Union’s Euro norms. Q. Discuss the potential implications of implementing
• These standards define the acceptable levels of various a pollution tax in India on both the environment and
pollutants that vehicles can emit, especially those the economy. [15 Marks] [250 Words]
equipped with internal combustion engines.

LAWS GOVERNING FORESTS OF


THE NORTHEAST
Why in News: Mizoram Assembly passed a resolution
opposing the Forest (Conservation) Amendment Act, 2023,
aimed at safeguarding the rights and interests of the people
of Mizoram.

Forest (Conservation) Amendment Act, 2023


• This Act amend the Forest Conservation Act, 1980,
Way Forward which regulates the extraction of forest resources
including timber and other minerals by industries and
• The proposed pollution tax on diesel vehicles highlights
local communities.
the government’s commitment to cleaner and greener
fuels. • This amendment allows forest land diversion for
• While it may not be immediately implemented, it various projects near India’s international borders
underscores the evolving landscape for diesel in India. without requiring forest clearance under the Forest
• The automotive industry faces challenges in adapting (Conservation) Act (FCA) of 1980.
to stricter emissions norms, while consumers are • It provides that the de-reservation of reserved forests,
reevaluating the economic viability of diesel vehicles. use of forest land for non-forest purpose, assigning
• Ultimately, the future of diesel in India may hinge on its forest land by way of lease or otherwise to private entity
ability to align with the nation’s environmental and and clearing of naturally grown trees for the purpose
economic goals. of reafforestation requires prior permission of the
Conclusion: Central Government.

Pollution taxes can be an effective tool for addressing [For full analysis of Forest (Conservation) Amendment Act,
environmental externalities and encouraging pollution 2023, refer August 2023 Magazine]
reduction. The design and implementation of pollution taxes
Is FCA Applicable to the Northeast?
should consider related challenges of pollution tax, so as to
• The FCA is a significant piece of legislation concerning
make them most effective.
forests, but its applicability in the northeastern states is
EXPECTED QUESTION FOR UPSC CSE influenced by special constitutional provisions.
• Article 371A for Nagaland and Article 371G for Mizoram
Prelims MCQ
prohibit the application of any law enacted by Parliament
Q. Consider the following taxes: that interferes with the customary law and procedure,
land ownership, and resource transfer in these states.
1. Carbon Tax        2. Angel Tax • To apply such laws, their Legislative Assemblies must
pass resolutions.
3. Windfall tax         4. Fat Tax
• In 1986, Nagaland extended the FCA’s application to
How many of the above are the pollution taxes? government forests and certain wildlife sanctuaries
under state control.
(a) Only one    (b) Only two • In contrast, Mizoram, after becoming a state in 1986,
adopted the FCA under the 53rd amendment to the
(c) Only three    (d) All four
Constitution (Article 371G).
Answer: a (1 only) • The FCA is also applicable in the rest of the Northeast,

80 ENVIRONMENT & GEOGRAPHY


including Meghalaya, Tripura, the Sixth Schedule reasons for not implementing it, including irrelevance,
Areas in these states, Arunachal Pradesh, Sikkim, and predominant ownership by communities, clans, chiefs,
Manipur, each with varying degrees of FCA clearances. and individuals, and a lack of forest-dwellers who are
entirely dependent on forests.
Understanding RFA (Recorded Forest Area) and FRA • For instance, Mizoram initially extended the FRA in 2009
(Forest Rights Act) but later revoked it in 2019, citing the absence of claims
• Recorded Forest Area (RFA) constitutes a significant for rights and the Ministry of Tribal Affairs’ refusal to
portion of northeastern states’ forests, ranging from sanction funds for implementation.
34.21% in Assam to 82.31% in Sikkim. Within RFA, • Nagaland is yet to decide on the FRA, with a committee
unclassed forests, governed by customary law and examining the matter for years.
procedures, account for substantial portions, such as
97.29% in Nagaland, 88.15% in Meghalaya, and others in Protecting Forests through Legal Measures:
Manipur and Tripura. ¾ To protect forests and uphold the Forest Rights Act
• The Forest (Conservation) Act (FCA) primarily aimed (FRA), states can adopt legal measures to ensure
to facilitate deforestation. However, the 1996 Supreme mandatory FRA fulfillment before recommending
Court’s Godavarman case expanded the FCA’s definition forest diversion proposals.
of “forest land” to include any area recorded as forest in ¾ They can also ensure Gram Sabha consent before
government records, including unclassed forests, thereby transferring forest land.
extending the FCA’s purview. ¾ The Ministry of Tribal Affairs holds the power to
• Additionally, areas outside the Recorded Forest Area issue legally enforceable directions under the FRA
(RFA) exist, with 38.5% in Assam, 29% in Nagaland, and or enact a separate law to recognize and settle forest
1.5% in Mizoram, neither recorded nor surveyed. rights when forests are diverted for other purposes,
• The Scheduled Tribes and Other Traditional Forest especially when forest dwellers are relocated.
Dwellers (Recognition of Forest Rights) Act (FRA) Conclusion
of 2006 considers “forest land” to include unclassified
Effective implementation of the FRA and proactive legal
forests, among other types.
measures can contribute to the conservation of these vital
• While this benefits most northeastern states due
natural resources while respecting the rights of indigenous
to their substantial unclassed forests, the FRA also
includes provisions recognizing rights under state communities.
laws, autonomous district councils, or traditional tribal
EXPECTED QUESTIONS FOR UPSC CSE
customary laws.
Prelims MCQs
Scheduled Tribes and Other Traditional Forest
Dwellers (Recognition of Forest Rights) Act, 2006 Q1. Consider the following statements:
It has been enacted to recognize and vest the
 1. The Constitution of India prohibits the
forest rights and occupation of forest land in forest enforcement of the parliamentary laws affecting
dwelling Scheduled Tribes and other traditional land ownership rights in Nagaland.
forest dwellers, who have been residing in such 2. Article 371A has been incorporated into the
forests for generations, but whose rights could not Constitution of India by the 13th Constitutional
be recorded. Amendment Act, 1962.
 This Act not only recognizes the rights to hold and Which of the statements given above is/are correct?
live in the forest land under the individual or common
(a) 1 Only     (b) 2 Only
occupation for habitation or for self-cultivation for
livelihood, but also grants several other rights to (c) Both 1 and 2    (d) Neither 1 nor 2
ensure their control over forest resources. Answer: c

 The Act also provides for diversion of forest land for Q2. 
Which government body is responsible for the
public utility facilities managed by the Government, implementation and administration of the Forest
such as schools, dispensaries, fair price shops, Rights Act (FRA) at the state level in India?
electricity and telecommunication lines, water tanks,
etc. with the recommendation of Gram Sabhas. (a) Ministry of Environment, Forest, and Climate
Change
Challenges in FRA Implementation: (b) Ministry of Tribal Affairs
• Despite the FRA’s potential to secure forest rights, its (c) National Tiger Conservation Authority (NTCA)
implementation remains limited in the northeastern states.
(d) Ministry of Panchayati Raj
• Assam and Tripura are exceptions, while other states cite Answer: d

ENVIRONMENT & GEOGRAPHY  81


¾ State Disaster Management Authorities (SDMAs),
DESIGNATION OF NATIONAL led by the Chief Ministers.
DISASTERS TAG ¾ Together, these bodies, in conjunction with district-
level organizations, oversee the coordination of
Why in News: Recently, Himachal Pradesh’s Chief Minister disaster management across the country.
has requested the Indian Prime Minister to declare the
destruction caused by heavy rains in the state a National Funding Mechanisms
Disaster. • The primary financial support mechanisms for
responding to disasters in India are the National Disaster
What is ‘Natural Disaster’ Tag? Relief Fund (NDRF) and the State Disaster Response
• In India, there is no official designation of “national Funds (SDRFs).
disasters.”
• Instead, the governance of natural disasters falls under • These funds are governed by the 2005 Disaster
the purview of the 2005 Disaster Management Act. Management Act.
• This legislation defines a disaster as an event that results • The Central Government typically contributes 75% of
in significant loss of life, human suffering, damage the funding to SDRFs in most states, with this percentage
to property, or environmental harm that exceeds the rising to 90% in northeastern and Himalayan states.
community’s capacity to cope.
• The primary purpose of these funds is to provide
• Consequently, the Act established two key entities:
immediate relief to victims affected by various types of
¾ the National Disaster Management Authority
calamities.
(NDMA), presided over by the Prime Minister

82 ENVIRONMENT & GEOGRAPHY


Determining Severe Calamities
Descriptive Question
• The categorization of a calamity as “severe” is based
on a specific set of criteria, which includes factors such Q. With reference to the National Disaster Management
as the calamity’s intensity, magnitude, and the level of Authority (NDMA) guidelines, discuss the
assistance required. measures to be adopted to mitigate the impact of
• When a calamity attains the designation of “severe,” a recent incidents of cloudbursts in many places of
special fund known as the Calamity Relief Fund (CRF) Uttarakhand. [UPSC CSE 2016]
is established.
• The funding for this relief fund is shared between the
Central Government and the state government.
ATLANTIFICATION OF THE
• In cases where the resources in the CRF are insufficient,
additional financial assistance can be sought from the
ARCTIC
National Calamity Contingency Fund (NCCF), which is Why in News: New research shows that the Arctic Ocean is
fully funded by the Central Government. undergoing significant changes, driven by two interconnected
Allocation of Funds phenomena: Atlantification and the Arctic Dipole. These
• The allocation of funds for various disaster relief purposes factors are reshaping the Arctic’s environment and climate,
is an integral part of India’s budgetary process. impacting sea ice loss and the broader ecosystem.
• The Finance Commission, a constitutional body
Atlantification: A Warm Influx
responsible for recommending the distribution of
• Atlantification refers to the increasing flow of warm
financial resources among states and the Centre, plays a
Atlantic Ocean water into the Arctic Ocean.
pivotal role in this process.
• This process is making some parts of the Arctic,
• When determining funds for immediate relief, the Finance
particularly the Barents Sea, more closely resemble the
Commission takes into account various factors, including
Atlantic.
past expenditures, the risk exposure of specific regions,
and the vulnerability of states to disasters. • The Arctic Ocean’s natural structure involves layers of
• These funds are then allocated to the National Disaster sea ice on top, followed by cooler freshwater, and then
Relief Fund (NDRF) and State Disaster Response Funds deeper, warmer, saltier water from the Atlantic.
(SDRFs). • These layers are held in place by differences in water
• The release of these funds by the Central Government is salinity, protecting the sea ice from melting due to warm
done in two equal instalments, although exceptions can Atlantic water.
be made during urgent situations, as was seen recently
• However, the rapid decline in Arctic ice cover in recent
when certain requirements for fund release were waived
decades is promoting Atlantification.
due to extreme rainfall in various states.
• As sea ice melts, it disrupts the layering, allowing warm
EXPECTED QUESTIONS FOR UPSC CSE
Atlantic water to penetrate deeper into the Arctic Ocean.
Prelims MCQ
The Role of the Arctic Dipole
Q1. With reference to the ‘Prime Minister’s National • The Arctic Dipole is an atmospheric feature that plays a
Relief Fund (PMNRF),’ consider the following crucial role in Atlantification.
statements:
• It operates in an approximately 15-year cycle, influencing
1. Disbursement of funds from it is made at the the Arctic’s wind and pressure patterns.
discretion of the Prime Minister.
• In the present “positive” Arctic Dipole regime, high
2. It has compulsory funds allotment in the Annual pressure is centred over the Canadian sector of the
Financial Statement. Arctic, causing clockwise winds.
3. It helps needy patients to meet treatment • Simultaneously, low pressure is centred over the
expenses in government hospitals. Siberian Arctic, resulting in counterclockwise winds.
How many of the above statements is/are correct?
• This wind pattern has far-reaching effects on the Arctic
(a) Only one     (b) Only two environment.
(c) Only three    (d) None • It affects upper ocean currents, air temperatures, heat
Answer: b (1 and 3 only) exchanges between the atmosphere, ice, and ocean, sea-
ice drift, and ecological systems.

ENVIRONMENT & GEOGRAPHY  83


The Arctic dipole

 he Arctic dipole anomaly is a pressure pattern


T
characterized by high pressure on the arctic
regions of North America and low pressure on
those of Eurasia. This pattern sometimes replaces
the Arctic oscillation and the North Atlantic
oscillation.
 I t was observed for the first time in the first decade
of 2000s and is perhaps linked to recent climate
change.
 he Arctic dipole lets more southern winds into
T
the Arctic Ocean resulting in more ice melting.
 he summer 2007 event played an important role
T
in the record low sea ice extent which was recorded
in September.
  he Arctic dipole has also been linked to changes in
T The Future and Climate Implications
arctic circulation patterns that cause drier winters • Researchers emphasize that the positive Arctic Dipole
in Northern Europe, but much wetter winters in regime may not persist indefinitely.
Southern Europe and colder winters in East Asia, • A reversal could lead to significant climatological
Europe and the eastern half of North America. repercussions, potentially accelerating sea-ice loss across
the entire Arctic region.
Impacts and Feedback Loops • These changes impact not only sea ice but also the broader
Arctic and sub-Arctic climate systems.
• They influence water exchanges between the Nordic seas
• The dynamics of the Arctic Ocean are closely
and the Arctic Ocean, with sea ice decline serving as a
interconnected with the Arctic’s climate, ecosystems, and
significant indicator of climate change.
the global climate system.
• Recent research indicates that under the positive Arctic
Dipole regime, there has been a decrease in the flow of Conclusion
Atlantic Ocean water into the Arctic Ocean through the Atlantification and the Arctic Dipole are driving
Fram Strait east of Greenland. Simultaneously, there has complex changes in the Arctic Ocean, affecting sea ice,
been an increase in Atlantic flow into the Barents Sea, regional climates, and ecosystems. Understanding these
north of Norway and western Russia. interconnected processes is crucial for comprehending the
• These alternating changes, described as a “switchgear broader implications of a changing Arctic and global climate.
mechanism,” are caused by the Arctic Dipole regimes.
• They have contributed to slowing the overall loss of sea EXPECTED QUESTION FOR UPSC CSE
ice in the Arctic when compared to previous years.
Prelims MCQ
• Importantly, the counterclockwise winds from the low-
pressure region in the current positive Arctic Dipole Q. Consider the following statements:
regime have driven freshwater from Siberian rivers into Statement I: The Arctic Dipole is an atmospheric
the Canadian sector of the Arctic Ocean. phenomenon that affects the heat exchanges between
• This freshwater movement has acted as a barrier, the atmosphere, ice, and ocean.
preventing the warmer saltwater from melting sea ice Statement II: Atlantification refers to the increasing
from below. flow of warm Atlantic Ocean water into the Arctic
Ocean.
Which one of the following is correct in respect of the
above statements?
(a) 
Both Statement-I and Statement-II are correct
and Statement-II is the correct explanation for
Statement-I
(b) 
Both Statement-I and Statement-II are correct and
Statement-II is not the correct explanation for
Statement-I

84 ENVIRONMENT & GEOGRAPHY


• It is particularly important during hot and humid weather
(c) 
Statement-I is correct but Statement-II is incorrect
conditions because high humidity can make the air
(d) Statement-I is incorrect but Statement-II is correct temperature feel much hotter than it actually is. The heat
Answer: a index is commonly used in weather forecasts to provide a
better understanding of the perceived temperature.

• A heat index value of 67°C or above can be extremely


HEAT INDEX dangerous for people and animals who have direct and
Why in News: Recently, Iran faced an unprecedented prolonged exposure.
challenge with a scorching heat index of 70 degrees Celsius • The temperature is 31 °C (88 degrees Fahrenheit), a
(°C) along its coastal region, raising concerns about the relative humidity of 40% only means that people need
survivability of such extreme conditions. to be cautious of fatigue with prolonged exposure and/
Heat Index or physical activity. But, if the relative humidity at the
• The heat index, also known as the “apparent temperature” same temperature is 95%, heat cramps or heat exhaustion
or “feels-like temperature,” is a measure of how hot it is likely, whereas prolonged exposure and/or physical
feels when relative humidity is factored in with the actual activity can also lead to a heat stroke.
air temperature.

ENVIRONMENT & GEOGRAPHY  85


2009 under the provisions of the EPA, 1986.
Adaptation Measures
• It is being implemented by the National Council for
 T
 o tackle these challenges, adaptation efforts Rejuvenation, Protection and Management of River
must include investments in early warning Ganga (National Ganga Council).
systems, adjustments to work schedules, and the • Objectives: To ensure effective abatement of pollution
development of sustainable cooling solutions. and rejuvenation of the river Ganga by adopting a river
 T
 hese measures are essential to mitigate the adverse basin approach to promote inter-sectoral co-ordination
effects of rising temperatures and humidity levels for comprehensive planning and management and
on both human health and overall well-being. ¾ To maintain minimum ecological flows in the river
Stay well-hydrated by drinking plenty of water.
  Ganga with the aim of ensuring water quality and
environmentally sustainable development.
Limit outdoor activities during the hottest and most
 
humid parts of the day. About Namami Gange Mission
 U
 se fans or air conditioning to create a cooler indoor • The Namami Gange Programme was launched by the
environment. government in June 2014 with the aim of rejuvenating
 Wear lightweight and breathable clothing. the Ganga River and its tributaries.
Take breaks in shaded or air-conditioned areas
  • Initially, the program was intended to run until March
when working or exercising outdoors. 31, 2021, but it was subsequently extended until March
31, 2026.
 B
 e aware of the signs of heat-related illnesses and
seek medical attention if symptoms arise. • The program is being implemented by the National
Mission for Clean Ganga (NMCG), along with its state
and district counterparts under the Ministry of Jal
EXPECTED QUESTION FOR UPSC CSE Shakti.

Prelims MCQ What is the Ganga River System?

Q1. The term, ‘Heat Index,’ recently mentioned in the   he headwaters of the Ganga called the ‘Bhagirathi’
T
news, refers to are fed by the Gangotri Glacier and joined by the
Alaknanda at Devprayag in Uttarakhand.
(a) It is the atmospheric Heat Budget of the specific
area on the Earth.   t Haridwar, Ganga emerges from the mountains to
A
the plains.
(b) It is a measure to assess how hot it feels to
the human body when relative humidity is  he Ganga is joined by many tributaries from the
T
combined with the air temperature. Himalayas, a few of them being major rivers such
as the Yamuna, the Ghaghara, the Gandak, and the
(c) I t is a measure of the excess temperature Kosi.
responsible for the melting of the polar ice caps.

(d) A
 measure to assess the total carbon credits Key Aspects:
earned by industry through emission controls. • Objectives: The primary goal of the Namami Gange
Answer: b Programme is to clean and rejuvenate the Ganga
River and its tributaries. It focuses on various aspects,
including sewage treatment, river-front development,
river surface cleaning, afforestation, biodiversity
MISSION TO CLEAN GANGA conservation, public awareness, effluent management,
and the development of Ganga Grams (villages located
Why in News: Seven years after the launch of the ambitious along the Ganga).
National Mission for Clean Ganga (NMCG), the mission’s
• State-wise Allocation: Notably, there is no state-
goal of achieving a clean Ganga river remains a task that is wise allocation of funds under the Namami Gange
still underway, with significant work yet to be completed. Programme, indicating a centralized approach to river
National Mission for Clean Ganga (NMCG) (2011) conservation and cleaning.
• It was registered as a society under the Societies Current Status:
Registration Act 1860. • Operational Projects: Out of the 409 projects for
• It acted as implementation arm of National Ganga River addressing river pollution planned, only 232 projects
Basin Authority (NGRBA) which was constituted in were made operational as of December 2022.

86 ENVIRONMENT & GEOGRAPHY


• Focus on Sewage Infrastructure: The majority of these projects pertain to the creation of sewage infrastructure. The
untreated discharge of domestic and industrial wastewater is identified as a major contributor to pollution in the Ganga.
• Sewage Treatment: In the five major basin states (Uttarakhand, Uttar Pradesh, Bihar, Jharkhand, and West Bengal), a total
of 11,765 million liters per day (MLD) of sewage is generated. The central government plans to set up Sewage Treatment
Plants (STPs) to treat only 7000 MLD of sewage by 2026.

National Ganga Council (NGC) (Formed: 2016) • There is a conspicuous sign of the improvement in
water quality along the Ganga was a rise in the dolphin
 It was formed under the River Ganga (Rejuvenation, population — both adult and juvenile — from 2,000 to
Protection and Management) Authorities Order, about 4,000.
2016.
• Fishermen are also reporting the increased presence
  � This order dissolved the National Ganga River of Indian carp (a fish species) that only thrives in clean
Basin (NGRBA) and replaced it with the NGC. water.

 
It was established under the Environment • The typical parameters used by the Central Pollution
(Protection) Act (EPA) 1986. Control Board (such as the levels of dissolved oxygen,
biochemical oxygen demand, and faecal coliform) vary
 Function: Preventing pollution and rejuvenating widely along various stretches of the river.
the Ganga River Basin, including the River Ganga
and its tributaries. • The NMCG is now working to develop a water quality
index, on the lines of the air quality index, to be able to
 Chairperson: Prime Minister of India better communicate about river-water quality.
 Vice-Chairperson: Union Minister for Jal Shakti
 It is chiefly responsible for the implementation of EXPECTED QUESTION FOR UPSC CSE
the National Mission for Clean Ganga.
Prelims MCQ
National Mission for Clean Ganga (NMCG)
Q. Consider the following statements with reference to
 It is a statutory authority established under the
National Council for River Ganga (Rejuvenation, the Namami Gange Mission programme:
Protection and Management) Act, 2016. 1. It is being implemented by the National Ganga
 In 2016, the Government authorised the NMCG to River Basin Authority (NGRBA).
exercise powers under the Environment (Protection)
2. It facilitates afforestation and biodiversity
Act, 1986. conservation.

Way Forward 3. It allocates more funds to the states with larger
Ganga basin than the states which have smaller
• The river’s water quality is now within prescribed limits spans of basin.
of notified primary bathing water quality.

ENVIRONMENT & GEOGRAPHY  87


• Averting, minimising and addressing ‘loss and damage,’
How many of the above statements is/are correct?
requires urgent action across climate and development
policies to manage risks comprehensively and provide
(a) Only one     (b) Only two
support to impacted communities.
(c) All three    (d) None • Financial flows needed to be made consistent with
Answer: a (2 only) climate-resilient development to meet urgent and
increasing needs.
• Access to climate finance in developing countries needed
to be enhanced.
GLOBAL STOCKTAKE REPORT Importance of the ‘Global Stocktake’ report
Why in News: The United Nations climate secretariat made • It illuminates a path forward that governments will need
public a ‘synthesis report’ on the results of three meetings to follow to combat the climate crisis.
held so far to discuss progress achieved by countries in • It prompts specific actions to accelerate transformative
achieving the goals of the Paris Agreement of 2015. climate action and support in high-impact areas.

Why is ‘Global Stocktake’ report? Which Aspects of Climate Action Does the Global Stocktake
Assess?
• The Global Stocktake is a periodic review mechanism
established under the Paris Agreement (at the UNFCCC 1. Mitigation: Evaluating global efforts to reduce
greenhouse gas emissions and keep global
COP 21) in Paris in 2015.
temperature rise below 2 degrees C (3.6 degrees F) and
¾ The first Global Stocktake will take place in 2023 ideally 1.5 degrees C (2.7 degrees F), and identifying
(COP28, Dubai). opportunities for additional emissions cuts.
• The synthesis report ties into a larger exercise called the 2. Adaptation: Measuring progress in countries’
‘global stocktake,’ that is expected to take place once in abilities to enhance their resilience and reduce
five years. vulnerability to climate impacts.
• In 2015, countries committed in Paris to keep global 3. Means of implementation, including finance, tech-
temperatures from rising beyond 2 degrees Celsius by nology transfer and capacity building: Assessing
the end of the century and “as far as possible” below 1.5 progress on aligning financial flows with emis-
degrees Celsius. sions-reduction goals and climate-resilient develop-
ment, and providing support to developing nations.
¾ Here they also agreed to periodically review, or take
stock of efforts, made by individual countries in
Conclusion
containing greenhouse gases and transitioning their
fossil-fuel dependent energy systems to renewable T transition to a renewable energy economy. The Declaration
sources. noted the need for USD 5.8-5.9 trillion in the pre-2030 period
required for developing countries as well as USD 4 trillion
Key findings
per year for clean energy technologies by 2030 to reach net
• The report states that the Paris Agreement has galvanised zero by 2050.
countries into setting goals and signaling the urgency of
the climate crisis.
EXPECTED QUESTION FOR UPSC CSE
• Governments need to support ways to transition their
economies away from fossil fuel, countries should work Prelims MCQ
on ensuring that the economic transition be equitable
and inclusive. Q. The “Global Stocktake Report” has been published
• It stated that much more ambition was needed to reduce by which of the following?
global greenhouse gas emissions by 43% by 2030 and
(a) International Union for Conservation of Nature
further by 60% in 2035 and reach net zero CO2 emissions
by 2050 globally. (IUCN)
• Deforestation and land-degradation have to be halted (b) United Nations Environment Programme
and reversed. (UNEP)
• Agricultural practices critical to reducing emissions (c) U
 nited Nations Framework Convention on
and conserving and enhancing carbon sinks have to be Climate Change (UNFCCC)
encouraged.
• Transparent reporting on adaptation could facilitate (d) International Monetary Fund (IMF)
and enhance understanding, implementation and Answer: c
international cooperation.

88 ENVIRONMENT & GEOGRAPHY


UNEP’S REPORT ON WASTEWATER • Resource Recovery: Wastewater contains valuable
resources. It can be a raw material for producing
Why in News: Recently, a report prepared by Global various products such as paper, polymers, pesticides,
Wastewater Initiative (GWWI), a global platform bringing rubber, paint, biodiesel, food preservatives, flavours,
fireproofing materials, waterproof fabrics, jewellery,
together UN agencies, NGOs, the private sector etc. to tackle
and more.
wastewater pollution, “Wastewater – Turning Problem to
Solution” was released. • Improper wastewater management: It is a global challenge,
exacerbated by population growth, urbanization, and
GWWI climate change. By 2030, domestic and municipal
wastewater production is estimated to rise by 24-38%
It is the only global partnership that facilitates
  • Vulnerable groups, especially women and children,
cooperation and coordination to better understand
are disproportionately affected by poor wastewater
and address wastewater challenges and
management.
opportunities worldwide.
Linked SDGs
UNEP acts as the Secretariat for GWWI and the
 
Initiative is currently chaired by the Turkish Water
Institute (SUEN).

United Nations Environment Programme


 It is a leading global environmental authority
which sets the global environmental agenda,
promotes the sustainable development within What is Wastewater?
the United Nations system, and serves as an
Wastewater comprises various components, including:
authoritative advocate for global environment
protection. • Domestic effluent, which includes black water (excreta,
 Established on: 5th June 1972 urine, and faecal sludge) and grey water (from kitchens
and bathing).
 HQs: Nairobi, Kenya
• Water from commercial establishments, including
 Its main reports: hospitals.
 Emission Gap Report • Industrial effluent, stormwater, and other urban run-off.
 Adaptation Gap Report • Agricultural effluent or run-off.
 Global Environment Outlook
 overning body: United Nations Environment
G
Assembly (UNEA)

About the Report


• This new report, “Wastewater – Turning problem to
solution” challenges the view that wastewater is an end-
of-pipe problem to be disposed of.
• It repositions wastewater as a circular economy
opportunity: a renewable and valuable resource to be
conserved and sustainably managed with the potential to
drive new jobs and revenue streams.
Key Highlights of the Report

• Low Wastewater Reuse: Only 11% of the world’s treated


wastewater is currently being reused. Approximately What are the Challenges to Wastewater Reuse in India?
50% of untreated wastewater still enters the environment.
• Unclear responsibilities among central, state, and local
• Untapped Potential: Wastewater has significant untapped government bodies.
potential. It could serve as an alternative energy source • Inadequate technological designs.
for around half a billion people, offset more than 10%
of global fertilizer use, and irrigate approximately 40 • Delays in project execution.
million hectares of land. • Lack of adequate financing mechanisms.

ENVIRONMENT & GEOGRAPHY  89


Recommendations for way ahead Conclusion
• Implementation of target-based regulations with The transformation of wastewater from a pollution problem
defined national reuse standards. to a valuable resource is essential for addressing water
• Detailed guidance on sewage treatment and reuse scarcity, pollution, and climate change, and ensuring a
technologies. sustainable
• Establishment of effective financing mechanisms that
allow for sufficient cost recovery.
• It calls for three actions: EXPECTED QUESTION FOR UPSC CSE
¾ reducing how much wastewater we produce,
¾ being more careful about what goes into the water Prelims MCQ
we use, and
¾ considering how to better collect and treat wastewater Q. The “Wastewater - Turning Problem to Solution”
so that we can recover and safely use its valuable report has been published by which of the following?
resources.
(a) World Union for Protection of Life
• The building blocks focus on the social, cultural and
behavioural changes that will need to happen in order (b) Wetlands International
for actions to succeed:
(c) Wildlife Conservation Society
¾ ensuring an enabling, coherent governance and
legislative framework (d) United Nations Environment Programme (UNEP)

¾ mobilizing investment in infrastructure Answer: d

¾ encouraging technical and social innovation


Descriptive Question
¾ improving data feedback for iterative adaptation
and Q. Discuss the challenges and potential solutions for
¾ strengthening communication and awareness to sustainable wastewater management in India. [10
help change our behaviours and attitudes. Marks] [150 Words]

90 ENVIRONMENT & GEOGRAPHY


cooling demand, transition to environmentally friendly
WORLD OZONE DAY refrigerants, enhance energy efficiency, and adopt
advanced technology options by the year 2037-38.
Why in News: The Ministry of Environment, Forest and
Climate Change celebrates World Ozone Day on 16th EXPECTED QUESTION FOR UPSC CSE
September. It is celebrated each year to commemorate
signing of Montreal Protocol. Prelims MCQ
¾ Theme 2023: "Montreal Protocol: Fixing the Ozone
Q. 
With reference to the Montreal Protocol, 1987,
Layer and Reducing Climate Change"
consider the following statements:
Montreal Protocol
1. The Kigali Agreement (2016) is an amendment to
• The Montreal Protocol, established in 1987, is a crucial
the Montreal Protocol.
international environmental treaty aimed at eliminating
the production and use of Ozone Depleting Substances
2. It was treaty under the framework of the Vienna
(ODS) to protect the Earth’s ozone layer. Convention (1985).
¾ India became a party to the Montreal Protocol in June
1992. 3. It is international trademark registration treaty.
• This treaty operates under the framework of the
Vienna Convention, which was adopted in 1985 as the How many of the above statements are correct?
foundation for global efforts to safeguard the ozone layer.
Top of Form (a) Only one     (b) Only two

• Later, Kigali Amendment to the Montreal Protocol (c) All three    (d) None
was adopted in 2016; to phase down the production and
Answer: b (1 and 2 only)
consumption of Hydrofluorocarbons (HFCs).
• HFCs were introduced as a non-ozone-depleting
alternative to CFCs and HCFCs, but it has huge global
warming potential thousands of times that of carbon
dioxide.
GREEN NUDGES
Ozone Depleting Substances Why in News: A recent study with an online food delivery
platform in China found that making “no disposable cutlery”
• ODS are Chlorofluorocarbons (CFCs),
the default choice and rewarding customers with green
Hydrochlorofluorocarbons (HCFCs), Carbon
points increased no-cutlery orders by 648%.
tetrachloride, Halons, etc.
• ODS contains Chlorine and bromine atoms reach the What does this study show?
stratosphere and repeatedly break apart ozone molecules
• The study revealed a substantial reduction in the
in the stratosphere, a process known as catalytic cycles
consumption of single-use cutlery thereby reducing
resulting in destruction of ozone.
waste generation.
India’s achievements in the implementation of the Montreal
Protocol What are Green Nudges?
• Phasing out Ozone-Depleting Substances (ODS): India • Green nudges are gentle encouragements designed to
has successfully phased out the production and controlled influence environmentally responsible behaviour.
use of ODS such as CFCs (Chlorofluorocarbons), carbon
tetrachloride, halons. • These nudges, rooted in behavioural economics, aim
• HFCs Phase-out Management Plan (HPMP): India has to guide people towards making sustainable decisions
been actively implementing HPMP Stage-II since 2017, while preserving their freedom of choice.
which is expected to continue until 2024. This plan focuses
on the phased reduction of Hydrofluorocarbons (HFCs), • Nudge techniques: Public awareness campaigns, Social
which are potent greenhouse gases and replacements for Norms Marketing, and Community-based Initiatives.
ODS. Criticism
• Phasing out HCFC-141b: India has eliminated the use of
• Historically, nudges have been criticized for their short-
HCFC-141b in the manufacturing of rigid foam.
term effects.
• India Cooling Action Plan (ICAP): ICAP is a strategic
• Critics of nudges often cite concerns about transparency
initiative that outlines a comprehensive approach to
and manipulative tactics.
cooling systems and refrigeration. It aims to reduce

ENVIRONMENT & GEOGRAPHY  91


Nudges Are Being Used in India (b) 
Deep-ocean polymetallic nodules which can be
Zomato, an online food delivery platform in India
  mined as resource of minerals.
has similar nudges on its app. It makes default (c) Gentle encouragements designed to influence
selection to “no-cutlery” since August 2021. environmentally responsible behaviour.
 “Zomato’s no-cutlery initiative was designed (d) Artificially created green patches in desert areas.
to reduce not just plastic but overall material
waste.”
Answer: c

Nudge in Governance: Nudge policies are becoming


 
increasingly popular in India as policymakers look
for ways to promote positive social and economic RAFFLESIA
change without resorting to traditional command-
and-control regulations. Why in News: The Rafflesia flower, renowned as the world’s
largest flower, is facing an escalating threat of extinction, as
or example, nudge policies used in programs
 F
highlighted in recent research.
such as Swachh Bharat Mission, GiveItUp
campaign and Beti Bachao Beti Padhao (BBBP),
etc.

Nudges Are Being Used Globally


Green nudges to tackle climate change involves
 
encouraging people to make choices that are good
for themselves, their communities, and the planet.
 UK: “The Big Switch Off” campaign to
encourage people to turn off lights and appliances
when not in use.
 Netherlands: “Pay as you throw” policy for
garbage collection.’
 US: San Francisco plastic bag ban.

Green Nudge Underlies the LiFE (Lifestyle for


 
Environment) mission: The idea promotes ‘mindful
and deliberate utilization’ instead of ‘mindless and
wasteful consumption’.

 The LiFE Movement aims to utilize the power of


collective action and nudge individuals across
the world to undertake simple climate-friendly
actions in their daily lives.

Conclusion
Thus, green nudges represent a promising approach to About Rafflesia flower:
addressing plastic pollution. These must be implemented • Rafflesia is a genus of parasitic flowering plants native
in transparent manner while increasing awareness about to Southeast Asia.
the causes to have a persistent effect. As environmental
consciousness continues to grow, green nudges may play a • These plants are renowned for producing the largest
individual flowers in the world, both in terms of size
pivotal role in shaping sustainable consumer behavior.
and weight.
EXPECTED QUESTION FOR UPSC CSE • Due of their unpleasant smell, they are frequently called
“corpse flowers”.
Prelims MCQ • Found in: Sumatra, Java, Borneo, Peninsular Malaysia,
Q. The term, ‘Green Nudges,’ recently mentioned in the southern Thailand, and the Philippines’ tropical
news, refers to rainforests
(a) 
Seeds scattered from airborne vehicles to boost • There are 42 known species of Rafflesia, and the most
greenery in arid mountain areas.
famous species within this genus is Rafflesia arnoldii.

92 ENVIRONMENT & GEOGRAPHY


EXPECTED QUESTION FOR UPSC CSE 2. They found in Western Ghats in India.
Prelims MCQ How many of the above statements are correct?
Q. Consider the following statements with reference to (a) 1 Only     (b) 2 Only
the ‘Rafflesia flower’: (c) Both 1 and 2    (d) Neither 1 nor 2
1. They are the largest individual flowers in the Answer: a
world, both in terms of size and weight.

SPECIES IN NEWS
Pterygotrigla Intermedica Tharosaurus Indicus
Why in News: Scientists have discovered a new species of a Why in News: Recently, scientists have characterised
vibrant orange coloured deep water marine fish from Digha dinosaur fossils from the Middle Jurassic period, found in
Mohana in West Bengal. the Thar desert near the Jaisalmer Basin by the Geological
Survey of India.

About Tharosaurus Indicus


• Belonging to the family Dicraeosauridae and from the
superfamily Diplodocoidea, these fossils are the first
dicraeosaurid sauropods to have been found in India.
• At 167 million years old, they are the oldest known
diplodocoid fossils in the world.
• The scientists named the dinosaur Tharosaurus
indicus, with:
About Pterygotrigla intermedica: ¾ Tharo deriving from the Thar desert,
¾ saurus from the Greek ‘sauros’, or lizard, and
• The new species, commonly known as gurnards or sea-
¾ indicus from its Indian origin.
robins, belongs to the family Triglidae.
Saurpods
• Named Pterygotrigla intermedica, it has characters quite
 Sauropods first appeared on the earth during the
similar to species like Pterygotrigla hemisticta. Jurassic period, about 200 million years ago.
• It is the fourth species of Pterygotrigla genus reported
 They were one of the most dominant clades of dino-
in India so far and there are a total 178 species of the saurs, surviving until the late Cretaceous period 65
Triglidae family worldwide. million years ago, when dinosaurs went extinct.

EXPECTED QUESTIONS FOR UPSC CSE WOLLEMI PINE


Prelims MCQs
Why in News: Scientists have decoded the genome of the
Q1. The term, ‘Gurnards,’ recently mentioned in the Wollemi pine, which was thought extinct 2 million years ago,
news is associated with which of the following?
to understand its almost unchanged survival since dinosaur
(a) Antibiotics times.
(b) Augmented reality
About Wollemi pine:
(c) Sahara Desert
• The Wollemi pine (Wollemia
(d) Species of marine fish Answer: d nobilis) is nearly identical to
preserved remains dating to
Q2.The term, ‘Tharosaurus Indicus,’ recently mentioned the Cretaceous period (145
in the ne ws, is associated with: million to 66 million years
(a) Aerobic bacterium ago).
(b) Dinosaur species • There are now just 60 of these
(c) Medicinal plant trees in the wild — and these tenacious survivors are
(d) African apes threatened by bushfires in the region. It was thought to
Answer: b have gone extinct around 2 million years ago.

ENVIRONMENT & GEOGRAPHY  93


• Wollemia is a genus of coniferous tree in the family • The Lower Lake was created in 1794 by Nawab Chhote
Araucariaceae. Khan.
• Wollemi National Park is the only place in the world • They have been designated a wetland of international
where these trees are found in the wild. importance under the international Ramsar Convention
• The tree has been almost commonly referred to as the since 2002.
Wollemi pine, though it is not a true pine (genus Pinus)
or a member of the pine family (Pinaceae) EXPECTED QUESTION FOR UPSC CSE
• The Wollemi Pine is one of the world’s oldest and rarest
plants dating back to the time of the dinosaurs. The oldest Prelims MCQ
fossil of the Wollemi tree has been dated to 90-200 million
years ago. Q. Consider the following statements with reference to
the ‘Bhoj Wetland’:
• The Wollemi pine is categorized as Critically Endangered
(CR) on the IUCN’s Red List. Prior to 1994 it was thought
1. Bhojtal lake was created in 1794 by Nawab Chhote
to be extinct.
Khan.
EXPECTED QUESTION FOR UPSC CSE
2. 
They have been designated a wetland of
Prelims MCQ international importance under the international
Ramsar Convention since 2002.
Q. Consider the following statements with reference to
the ‘Wollemi pine’:
How many of the above statements are correct?
1. 
It can only be found in the wild in Wollemi
National Park.
(a) 1 Only     (b) 2 Only
2. It is categorized as Vulnerable on the IUCN’s Red
List. (c) Both 1 and 2    (d) Neither 1 nor 2
3. It is world’s oldest and rarest plants dating back Answer: b
to the time of the Dinosaurs.
How many of the above statements are correct?

(a) Only one     (b) Only two CAMPBELL BAY NATIONAL PARK
Why in News: A lot of areas in the Nicobar Islands are not
(c) All three    (d) None
accessible to outsiders, but entry is allowed for Campbell Bay
Answer: b (1 and 3 only)
National Park.

About the National Park:


BHOJ WETLAND • Campbell Bay National Park is located in the Great
Nicobar Island.
Why in News: The National Green Tribunal ((NGT) central • Campbell Bay National Park became a national park in
bench has issued a ban on the operation of cruise boats 1992.
in the Upper Lake (Bhoj Wetland) and other wetland • Campbell Bay National Park is a part of the Great Nicobar
sites in Madhya Pradesh. Biosphere Reserve.
• These vessels have the potential to discharge sewage and • Campbell Bay National Park is closer to Sumatra Island
trade effluents into the water. in Indonesia (190 km) than Port Blair in India (533 km).

About the Wetland • The national park lies very close to Galathea National
• The Bhoj Wetland consists of two lakes located in Bhopal, Park and a mere 12 km wide forested buffer zone
Madhya Pradesh. separates the two.
• The park is made up of a diverse range of habitats like
• The two lakes are the Bhojtal (Upper Lake) & the Lower
evergreen rainforests, mangroves, coastal areas, and
Lake.
marine ecosystems.
• The Upper Lake is surrounded by Van Vihar National • The park is home to some of the endangered and endemic
Park on the south. species like the Nicobar megapode, Nicobar pigeon, and
• The Bhojtal was created by Paramara Raja Bhoj (1005- the Nicobar long-tailed macaque.
1055), ruler of Malwa.

94 ENVIRONMENT & GEOGRAPHY


EXPECTED QUESTION FOR UPSC CSE

Prelims MCQ

Q. With reference to the ‘Campbell Bay National Park,’ consider the following statements:

1. It is part of the Great Nicobar Biosphere Reserve.

2. It contains mangrove forests.

3. It is located in the vicinity of the Saddle Peak National Park.

4. It contains thick tropical evergreen forests and abundant tree ferns.

How many of the above statements are correct?

(a) Only one     (b) Only two

(c) Only three    (d). All four

Answer: c (1, 2 and 4)

ENVIRONMENT & GEOGRAPHY  95


SECTION

E SCIENCE & TECHNOLOGY

21st century so there is need for the nation’s focus on


DIGITAL PERSONAL DATA building a strong data privacy regime.
PROTECTION ACT, 2023 • Digital Personal Data Protection Act, 2023, is a momentous
stride in safeguarding individual privacy rights and
Why in News: Recently, the Digital Personal Data Protection promoting responsible data management practices.
Bill has received the Presidential assent followed by official
• This groundbreaking legislation aims to strike a delicate
gazette notification and has become law of the land. balance between rights and duties of the citizen (Digital
Nagrik) on one hand and the obligations to use collected
Need of Digital Personal Data Protection (DPDP) Act, 2023
data lawfully by the Data Fiduciary on the other hand.
• Technology has become the defining paradigm of the

Key stakeholders defined in DPDP Act


 Data Principals – Individuals within the territory of India whose personal data is being processed.
Data Fiduciary – Data fiduciaries are organizations deciding what data is to be collected, how data is to be collected and
 
the purposes for which it is to be used.
 Significant Data fiduciary – Organisation that processes large volumes of sensitive data sets.
 Data Processors – Organisations that process data on behalf of data fiduciary based on instructions.
 Consent manager – These assist data principals and data fiduciaries to give, manage, review, and withdraw consent.

96 SCIENCE & TECHNOLOGY


Key Features of Digital Personal Data Protection Act, 2023 ¾ The central government may, by notification,
exempt certain activities from the application of
• Applicability – the Act. These include processing by government
¾ The Act will apply to the processing of digital entities in the interest of the security of the state and
personal data within India. public order, and research, archiving, or statistical
purposes.
¾ It will also apply to the processing of personal data
outside India, if it is for offering goods or services or • Data Protection Board of India –
profiling individuals in India.

• Consent – ¾ The central government will establish the Data


Protection Board of India.
¾ Personal data may be processed only for a lawful
purpose for which an individual has given consent. • Key functions of the Board include monitoring
compliance and imposing penalties,
¾ A notice must be given before seeking consent.
¾ Consent may be withdrawn at any point in time. ¾ directing data fiduciaries to take necessary measures
in the event of a data breach, and hearing grievances
¾ For individuals below 18 years of age, consent will be made by affected persons.
provided by the legal guardian.

• Transfer of Personal Data outside India – • Penalties –

¾ The central government will notify countries where a • The schedule to the Act specifies penalties for various
data fiduciary may transfer personal data. offences such as up to
¾ Transfers will be subject to prescribed terms and
Rs 200 crore for non-fulfilment of obligations for children,
 
conditions.
and
• Exemptions –

¾ Rights of the data principal and obligations of data Rs 250 crore for failure to take security measures to
 
fiduciaries (except data security) will not apply in prevent data breaches.
specified cases.
Rs. 10,000 for breach in observance of duty of data
 
¾ These include prevention and investigation of principal.
offences, and enforcement of legal rights or claims.

SCIENCE & TECHNOLOGY  97


Rights of Data Principals
 Right to access information about personal data
¾ Data Principal has the right to obtain information on processing, the categories of personal data shared and identities
of all the Data Processors with whom the personal data has been shared.
 Right to correction, erasure and withdrawing consent for use of personal data
¾ The data principal can reach out to Data Fiduciary in order to exercise their right to correct, complete, update and
erasure of personal data.
 Right to Grievance Redressal
¾ Data Fiduciary and Consent Manager is required to respond to the grievance of Data principal within the prescribed
time period.
 Right to Nominate
¾ This is a unique right which enables Data Principals to nominate any other individual to exercise the rights of Data
Principal in event of death or incapacity of Data Principal.
¾ From an ethical and moral standpoint, this is a milestone in recognising the human rights of an individual’s
identifiable property after death rather than it being in the public domain and used by anyone and everyone.

Positive aspects of the DPDPA 2. Empowers Data Principals


1. Boosts growth and innovation
• Individuals now have the right over their personal data to
• The DPDPA has a significant effect on the business sector withdraw their consent, seek grievance redressal, correct,
and was inevitable considering the speed of digitisation access, and update their personal data and appoint a
in India and volumes of personal data being processed. nominee for the same.
• Implementation of the DPDPA will boost consumer • This empowers Data Principals to have more control over
trust, cross-border trade, lawful processing and will thus their personal data along with ensuring transparency.
enhance India’s economy and digital innovation.

98 SCIENCE & TECHNOLOGY


3. Provides alternative to consent-based processing
• Information Technology (Intermediary Guidelines
• The legitimate uses involve different purposes for and Digital Media Ethics Code) Rules 2021
processing such as complying with laws, and court
orders, responding to medical emergencies, ensuring � IT Rules (2021) mandate social media platforms
safety, and defending rights. to exercise greater diligence with respect to the
• These permitted purposes can enable businesses to content on their platforms.
reduce their burden by relying on grounds of processing,
Potential concerns in application of the Act
other than consent. It will save costs in implementing
additional mechanisms for consent management. 1. Data Processing under contractual obligations
4. Effective Data Processor governance • Moving away from deemed consent, the law now seeks
to require Data Fiduciaries to process personal data on
• The Data Processor can now be held accountable under
consent and certain legitimate uses.
contract for failing to assist a Data Fiduciary in obligations.
• It is seen that contractual obligation has not been directly
• This can help Data Fiduciaries plan for their risk appetite
categorised under legitimate uses.
and set off risk with obligations which are shared
responsibilities with the Data Processor. 2. No obligation for data mapping and data inventorisation
5. Ease in implementation • The law does not mandate maintaining records of
processing activities or data maps to identify how
• A phased implementation proposed in the DPDPA
the personal data is flowing within the organisation
gives organisations the time to plan their changes for
infrastructure and where it is being stored.
complying with provisions of the DPDPA and reduces
resources required for compliance with the DPDPA. 3. Processing of children’s personal data
6. Relaxed cross-border transfer • If the central government is satisfied that a Data Fiduciary
processes children’s personal data in a manner that is
• The DPDPA has provided a minimally restrictive
“verifiably safe”, the Central Government can exempt
approach to crossborder data transfers.
such Data Fiduciary from any or all obligations of
7. Increased accountability processing children’s data above a certain age.
• Significant Data Fiduciaries have an additional obligation • However, what would constitute as verifiably safe
to conduct Data Protection Impact Assessments. is unclear and thus can be detrimental to the rights
• The requirement to assess the risk to the rights of protection of children.
individuals as a part of the Data Protection Impact 4. Limitation on Data Principal rights
Assessment will bring a balance between the risks of
• The rights of Data Principals with respect to access,
processing personal data and business interests.
correction, and erasure do not apply when data is
8. Distributed liability for organisation processed for legitimate uses except when the Data
• The DPDPA allows both, Data Fiduciary and Consent Principal voluntarily provides their personal data.
Managers to be held liable before the Board when they • While the rights are now made available, it discriminates
fail in their carrying out their respective responsibilities. against individuals whose personal data has been
• This eases consent-related compliance and liability for collected for legitimate uses and not with their consent.
Data Fiduciaries. 5. Exemption on classes of Data Fiduciary
• The Central Government can exempt a Data Fiduciary or
India’s efforts to strengthen data protection regime
a class of Data Fiduciaries from certain obligations under
• B.N. Srikrishna Committee 2017 the DPDPA.
• However, the rationale behind such exemptions is
� Government appointed a committee of experts unclear.
for Data protection under the chairmanship
of Justice B N Srikrishna in August 2017, that Global scenario for Data protection
submitted its report in July 2018 along with a
draft Data Protection Bill. The EU’s General Data Protection Regulation,
 
which came into effect in 2018, is claimed to be the
The Report has a wide range of
�  “toughest privacy and security law in the world,”
recommendations to strengthen privacy law in and seen as the global benchmark.
India including restrictions on processing and Several nations including China and Vietnam have
 
collection of data, Data Protection Authority, recently tightened laws governing the transfer of
right to be forgotten, data localisation etc. personal data overseas.

SCIENCE & TECHNOLOGY  99


Australia in 2018 passed a bill that gave police
  Descriptive Question
access to encrypted data.
Q. 
“Data is the new oil.” In light of this statement
About 70% of countries worldwide have some form
 
critically analyse the recent Digital Personal Data
of legislation for data protection, according to the
Protection Act, 2023. [15 marks, 250 words]
United Nations trade agency (UNCTAD).
U.S. model has no comprehensive set of privacy
 
rights or principles but there are limited sector GM CROPS IN INDIA
specific regulations.
Why in News: The Centre has moved the Supreme Court
Way Ahead: seeking to withdraw its oral undertaking that it will maintain
The Act will be able to keep the personal data of a user
  a status quo and will not go ahead with the commercial
safe, and give them more liberty on how to port their cultivation of genetically modified (GM) mustard in India.
personal data.
The Need for GM Crops
The Act will ensure that entities like internet companies,
 
mobile apps, and business houses are more accountable • The adoption of GM crops has become crucial in the
and answerable about collection, storage and processing global quest for food and nutritional security, especially
in the face of climate change, population pressure, and
of the data of citizens as part of “Right to Privacy”.
stressed agricultural lands.
The Act also provides for a legislative backing to the
 
Supreme Court’s landmark judgement in Justice K. S. • The Sustainable Development Goal of ‘Zero Hunger’ by
Puttaswamy (Retd) Vs Union of India Case (2017). 2030 necessitates innovative approaches to agriculture.

¾ The judgement of nine-judge bench of the Supreme Court • GM crops offer the potential to significantly contribute
unanimously held that Indians have a constitutionally to this goal.
protected fundamental right to privacy that is an India’s Unique Requirements
intrinsic part of life and liberty under Article 21.
• India faces specific challenges, including a deficit in
Conclusion
edible oils, reliance on imports, low per-hectare crop
With implementation of the Act, India has taken huge step yields, and the need for effective weed control.
towards facilitating personal data privacy and protection. • GM crops can play a pivotal role in addressing these
This is evident from few of the Data Principal’s rights such issues, saving soil moisture and nutrients, and
as right to erase and restrict processing of personal data, enhancing farm income.
which are in line with global best practices.
Understanding DMH-11 and the Barnase-Barstar Technology
By adhering to the essential compliances outlined in the Act,
companies can build a foundation of trust and loyalty with
their customers in this data-driven era.

EXPECTED QUESTIONS FOR UPSC CSE


Prelims MCQ
Q. Consider the following statements with reference
to the ‘Digital Personal Data Protection Act, 2023’:
1. 
It enables the aggrieved party to file an appeal
before the Telecom Disputes Settlement and
Appellate Tribunal (TDSAT).
2. 
Appellate Tribunal under this act shall have all
the powers of a Civil Court.
3. 
It facilitates the establishment of the Data
Protection Board of India.
How many of the above statements are correct?
(a) Only one   (b) Only two
(c) All three   (d) None
Answer: c

100 SCIENCE & TECHNOLOGY


• The DMH-11 GM mustard variety carries an herbicide- amyloliquefaciens, specifically Bargene, Barnase, and
resistant gene, making it herbicide-tolerant. Barstar.

• It has been developed by scientists at the Centre for • This innovative use of the barnase-barstar technology
Genetic Manipulation of Crop Plants (CGMCP), Delhi has the potential to revolutionize mustard cultivation in
University, using the barnase-barstar GM technology. India, offering high yields and herbicide resistance.
• However, its adoption also raises important concerns and
• This technology involves the incorporation of considerations in the context of GM crop development
genes isolated from the soil bacterium Bacillus and regulation.

SCIENCE & TECHNOLOGY  101


Conclusion
How many of the statements given above are correct?
Cultivation of these GM mustard hybrids developed (a) Only one      (b) Only two
indigenously could help enhance farmers’ income, reduce
(c) All three       (d) None
the oil-import burden and help achieve much-needed self-
Answer: c
reliance in edible oil production. The environmental release
of DMH-11 marks the beginning of a new era in self-reliance
Descriptive Question
and sustainability in agriculture.
Q. 
Discuss the role that GM crops can play in
EXPECTED QUESTIONS FOR UPSC CSE
developing economies across the globe. [10 Marks]
[150 words]
Prelims MCQ

Q. With reference to Genetic Modification (GM) of


crops, consider the following statements:
NIPAH VIRUS
Why in News: Recent reports from Kerala highlight the
1. 
GM involves inserting DNA into the genome of resurgence of Nipah virus infections, with two casualties
an organism.
confirmed and two others currently under treatment.
2. 
GM plants can be developed through changes in What is the Nipah Virus?
Mitochondrial DNA.
• Nipah is a zoonotic virus, usually transmitted to humans
through infected animals like fruit bats or contaminated
3. 
G M of plants can result in new or different
food.
characteristics as compared to parent
species. • Person-to-person transmission through close contact is
also possible.

102 SCIENCE & TECHNOLOGY


• This highly lethal virus has mortality rates varying from
3. 
It can spread to humans through consumption
40% to 75%.
of contaminated food.

How many of the above statements are correct?

(a) Only one     (b) Only two

(c) All three     (d) None


Answer: c

Descriptive Question

Q. Discuss the Nipah virus, its transmission, and the


factors contributing to its high fatality rate. Analyze
the measures taken for containment during past
outbreaks and highlight the challenges in managing
this infectious disease. [15 Marks] [250 Words]

DUCHENNE MUSCULAR
DYSTROPHY
Why in News: Recently, World Duchenne Muscular
Dystrophy Day was observed by the Department of
Empowerment of Persons with Disabilities (DEPwD) on 7th
September.
• The purpose of the day is to encourage initiatives that
will raise awareness, advocate for change, and promote
social inclusion for people with dystrophinopathies.
About Duchenne muscular dystrophy:
Containment and Transmission Rate • Duchenne muscular dystrophy (DMD) is a severe type of
muscular dystrophy that primarily affects boys.
• Despite its high fatality rate, Nipah virus outbreaks have • Muscle weakness usually begins around the age of four,
typically been localized and contained relatively quickly. and worsens quickly.
¾ The 2018 outbreak in Kerala resulted in the death of • Most are unable to walk by the age of 12.
17 out of 18 confirmed cases.
• Some may have intellectual disability.
• One reason for this is the virus’s relatively low
• About two thirds of cases are inherited from a person’s
infectiousness.
mother, while one third of cases are due to a new
• The reproductive number (R0), a measure of how quickly mutation.
a virus spreads, has been estimated at around 0.48 in
• It is caused by a mutation in the gene for the protein
previous Nipah outbreaks.
dystrophin.
• An R0 value below one indicates that an infected person
¾ Dystrophin is important to maintain the muscle
infects less than one other person, leading to a gradual
fiber’s cell membrane.
decline in the outbreak.
• Treatment:
EXPECTED QUESTIONS FOR UPSC CSE ¾ Although there is no known cure, physical therapy,
Prelims MCQ braces, and corrective surgery may help with some
symptoms.
Q. 
With reference to ‘Nipah virus,’ consider the ¾ Assisted ventilation may be required in those with
following statements: weakness of breathing muscles.
1. It is a zoonotic disease. ¾ Medications used include:
 steroids to slow muscle degeneration,
2. 
Flying Foxes are the animal reservoir of this
 anticonvulsants to control seizures and some
virus.
muscle activity, and

SCIENCE & TECHNOLOGY  103


 immunosuppressants to delay damage to
dying muscle cells. MONOCLONAL ANTIBODY
¾ Gene therapy, as a treatment, is in the early stages of Why in News: Monoclonal antibodies, such as REGEN-
study in humans.
COV2, have proven to be life-saving treatments for severely
EXPECTED QUESTION FOR UPSC CSE affected Covid-19 patients in a UK clinical trial.

Prelims MCQ What are Monoclonal antibodies?


• They are artificially created antibodies that aim to aid
Q. Consider the following statements with reference to the body’s natural immune system.
the ‘Duchenne Muscular Dystrophy’:
• They target a specific antigen — a protein from the
1. It primarily affects males as compared to females. pathogen that induces immune response.
2. It causes infants to be born with microcephaly How are they created?
and other congenital malformations.
• Monoclonal antibodies can be created in the lab by
3. It is a genetic disease mostly inherited from a exposing white blood cells to a particular antigen.
person’s mother. • To increase the quantity of antibodies produced, a
single white blood cell is cloned, which in turn is used to
How many of the above statements are correct? create identical copies of the antibodies.
(a) Only one     (b) Only two • In the case of Covid-19, scientists usually work with the
spike protein of the SARS-CoV-2 virus, which facilitates
(c) All three     (d) None the entry of the virus into the host cell.
Answer: b (1 and 3 only)
EXPECTED QUESTION FOR UPSC CSE
Prelims MCQ

PREECLAMPSIA Q. With reference to the REGEN-COV2, consider the


following statements:
Why in News: As per a study, a liquid-biopsy approach 1. These are Monoclonal antibodies.
that measures DNA-methylation levels in the blood may 2. These are created by artificial cloning of the stem
improve the detection of pregnancies at risk of developing cells.
preeclampsia at early stages.
Which of the statements given above is/are correct?

About Preeclampsia: (a) 1 only       (b) 2 only


(c) Both 1 and 2     (d) Neither 1 nor 2
• Preeclampsia is a complication of pregnancy.
Answer: a
• It is a serious condition that can happen after the
20th week of pregnancy or after giving birth (called
postpartum preeclampsia). GLOBAL IMPACT OF HIGH BP

EXPECTED QUESTION FOR UPSC CSE Why in News: The World Health Organization (WHO) has
released its first-ever report on the global impact of high
Prelims MCQ blood pressure (hypertension).
Q. The term, ‘Preeclampsia,’ recently mentioned in What is Hypertension?
the news, refers to  Blood pressure is the force exerted by circulating
blood against the walls of the body’s arteries, the major
(a) Ukraine’s Black Sea port
blood vessels in the body.
(b) Vector-borne diseases  When the blood pressure is excessively high,
hypertension develops.
(c) Pregnancy-related medical complications  
Having systolic blood pressure more than or equal
to 140 mmHg, diastolic blood pressure greater than
(d) Non-vascular plant species or equal to 90 mmHg, or taking anti-hypertensive
Answer: c medication to reduce blood pressure are all considered
to be signs of hypertension.

104 SCIENCE & TECHNOLOGY


Hypertension in India
• Most significant risk factor for death and disability in India.
• A paper published in The Lancet, last year revealed that less than one-fourth of hypertensive patients in India had
their blood pressure under control during 2016-2020.
• The National Family Health Survey (NFHS-5) reported a hypertension prevalence of 24% in men and 21% in women,
indicating an increase from the previous round of the survey.

SCIENCE & TECHNOLOGY  105


Best practice to tackle Hypertension in India

 India has set a target of 25% relative reduction in the prevalence of hypertension (raised blood pressure) by 2025.
 To achieve this, the Government of India launched the Indian Hypertension Control Initiative (IHCI) to fast-track
access to treatment services for over 220 million people in India who have hypertension.

EXPECTED QUESTION FOR UPSC CSE • Efficiency Challenge: While the model was successful,
only around 1% of the stem cell mixture assembled
Prelims MCQ spontaneously. This low-efficiency rate highlights a
limitation in the process.
Q. The first-ever report on the global impact of high Importance of Embryo Models and Research
blood pressure (Hypertension) is published by
• Ethical Constraints: Early-stage embryo development
which of the following?
is challenging to study ethically, particularly once it
(a) United Nations Programme on HIV and AIDS implants in the uterus. As a result, researchers rely on
lab models or donated embryos to explore these initial
(b) Global Health Council changes.
• Reducing Miscarriages and Birth Defects: The initial
(c) World Health Organization (WHO) days of embryo development are crucial, as most
miscarriages and birth defects occur during this period.
(d) International Medical Corps Studying these stages can provide insights into genetic
and inherited diseases, potentially leading to improved
Answer: c
outcomes.
• Enhancing IVF Success: Understanding why some
embryos develop normally and implant successfully
HUMAN EMBRYO MODEL while others do not can enhance the success rates of in
WITHOUT SPERM OR EGG vitro fertilization, a critical fertility treatment.

Why in News: Scientists have achieved a remarkable feat Can Lab-Grown Embryos Lead to Pregnancy?
by creating a model of a human embryo in a laboratory No, lab-grown embryo models are not intended for
environment, completely bypassing the need for sperm and pregnancy. These models are designed solely for studying the
egg cells. early stages of fetal development and are generally destroyed
after 14 days of research. Attempts to implant these models
About Experiment
are not allowed, and they are far from becoming actual
• This innovative research involved the use of stem cells, embryos capable of implantation.
which have the ability to differentiate into various cell
types, allowing them to spontaneously assemble into an
embryo-like structure.
• This breakthrough opens up new avenues for studying
the early stages of embryo development, which have
significant implications for understanding genetic and
inherited diseases, as well as improving the success rates
of in vitro fertilization (IVF).
The Creation of the Embryo Model
• Stem Cells and Chemicals
¾ Scientists in Israel developed a human embryo-
like model using a combination of stem cells and
chemicals.
¾ These stem cells had the potential to differentiate
into different types of cells required for fetal
development, including those responsible for
forming the fetus itself, providing nutrients, creating
the body’s developmental plan, and constructing
structures like the placenta and umbilical cord.

106 SCIENCE & TECHNOLOGY


Insights Gained from Embryo Models Conclusion
• Understanding errors in DNA duplication and The creation of a human embryo model in the laboratory
chromosome distribution. without using sperm or egg cells represents a significant
• Identifying when DNA duplication abnormalities advancement in scientific research. These models offer
occur, shedding light on the origins of developmental insights into early embryo development, genetic diseases,
issues. Investigating the roles of various genes in fetal and fertility treatments while adhering to ethical boundaries.
development.
What are stem cells? EXPECTED QUESTION FOR UPSC CSE

Stem cells are the body’s raw materials — cells from which Prelims MCQ
all other cells with specialized functions are generated.
Q1. 
In the context of recent advances in human
Under the right conditions in the body or a laboratory, reproductive technology, ‘In Vitro Fertilization
stem cells divide to form more cells called daughter cells. (IVF)” is used for
Types of stem cells (a) Genetic modification of sperm-producing cells
Stem cells are divided into 2 main forms. They are
(b) Fertilization of egg in the laboratory by using
embryonic stem cells and adult stem cells. the donor sperm
1. Embryonic stem cells. The embryonic stem cells used
(c) Development of embryos using recombinant
in research today come from unused embryos. These DNA technology
result from an in vitro fertilization procedure. They
are donated to science. These embryonic stem cells are (d) 
Prevention of mitochondrial diseases in
pluripotent. This means that they can turn into more offspring
than one type of cell. Answer: b

2. Adult stem cells. There are 2 types of adult stem cells. One
type comes from fully developed tissues such as the brain,
skin, and bone marrow. There are only small numbers of FIRST CLONED MAMMAL
stem cells in these tissues. They are more likely to generate
only certain types of cells. For example, a stem cell that Why in News: Sir Ian Wilmut, the man who led the team that
comes from the liver will only make more liver cells. created Dolly the sheep, the first mammal to be cloned from
an adult cell, has died, aged 79.

Animal Cloning
Animal cloning refers to the process of creating genetically
identical copies of an animal through a variety of scientific
techniques. This process results in offspring that are genetically
identical to the donor animal, essentially making them clones.

Types of Animal Cloning


1. Reproductive Cloning: This form of cloning aims to
create an identical copy of an existing animal. It involves
transferring DNA from a donor cell into an egg cell without
its nucleus. The resulting embryo is then implanted into
Challenges and ethical considerations a surrogate mother, ultimately leading to the birth of an
 
Despite the promising results, the current 99 percent organism genetically identical to the donor.
failure rate in model development poses a challenge. 2. Therapeutic Cloning: Therapeutic cloning is specifically
 Understanding the causes of miscarriages or infertility may employed for medical purposes. It focuses on creating
be hindered if the model cannot reliably assemble itself. embryonic stem cells, which can be instrumental in
regenerating tissues and potentially treating a wide range
 The possibility of extending embryo development beyond of diseases.
the 14-day stage also raises ethical questions.
3. DNA Cloning: DNA cloning is a molecular biology
 In some countries, this would not be illegal, as these technique used to make multiple copies of a specific
models are legally distinct from actual embryos. DNA segment, such as a gene or a particular sequence.

SCIENCE & TECHNOLOGY  107


The Process of Animal Cloning • Medical Advancements: In the field of medicine, cloning
• Isolation: In this initial step, scientists extract the donor holds the potential for regenerative medicine, including
DNA containing the desired genes. Careful selection the development of personalized therapies and the
ensures that the chosen DNA possesses the required production of organs for transplantation.
traits for replication. • Repopulation of Endangered Species: Cloning presents a
• Insertion: The isolated DNA is then inserted into a vector, potential solution for restoring dwindling populations of
often a plasmid, which serves as a carrier molecule to endangered species, thereby contributing to biodiversity
introduce the donor DNA into a host organism. conservation efforts.
• Transformation: The vector containing the donor DNA is • Research and Drug Development: Cloning provides
introduced into the host organism, where it incorporates researchers with essential tools to explore the underlying
the DNA fragment into its own genome. mechanisms of diseases and develop new drugs.

• Replication and Expression: Within the host organism, Dolly the Sheep
the donor DNA replicates along with the host’s DNA.
This process allows for the expression of the desired  Dolly the Sheep was a historic and iconic scientific
genes and the subsequent production of proteins. achievement in the field of cloning. Dolly was the first
mammal to be cloned from an adult somatic cell using
• Selection: The transformed host cells exhibiting the
desired traits are identified and isolated for further a technique known as somatic cell nuclear transfer
applications. (SCNT).

Advantages of Animal Cloning


 In the SCNT technique the nucleus of an adult somatic
cell (in this case, a mammary gland cell) was transferred
• Replication of Desirable Traits: Cloning enables the
into an enucleated egg cell. This reconstructed egg
replication and preservation of desirable traits, making
it a valuable tool in agricultural advancements and was then stimulated to develop into an embryo and
breeding strategies. implanted into a surrogate mother sheep for gestation.

108 SCIENCE & TECHNOLOGY


Challenges in Animal Cloning impact on personal identity, family dynamics, and social
• Ethical Considerations: Despite its significant norms. These aspects raise questions about the broader
opportunities, animal cloning raises ethical questions consequences of cloning on human society.
regarding the manipulation of life. Looking to the Future
• Genetic Diversity: Animal cloning can reduce genetic
As we move forward, the field of animal cloning holds vast
diversity within populations, potentially making them
potential to revolutionize various sectors, from medicine
more vulnerable to diseases and environmental changes.
and agriculture to conservation and research. However, its
• High Failure Rates: One of the challenges of animal
ethical, scientific, and social dimensions demand careful
cloning is its relatively low success rates. High failure
consideration and continuous evaluation. Striking a
rates result in significant costs and resource requirements,
often making cloning a resource-intensive process. balance between scientific advancements and responsible
implementation is essential to harness the benefits while
• Psychological and Social Implications: Animal cloning
has complex societal implications, including its potential mitigating the risks associated with animal cloning.

EXPECTED QUESTION FOR UPSC CSE

Prelims MCQ

Q. In somatic cell nuclear transfer, what is the role of the somatic cell?

(a) To provide the egg with nutrients.

(b) To develop into an embryo.

(c) To contribute to the placenta.

(d) To provide the genetic material for the clone.


Answer: d

SCIENCE & TECHNOLOGY  109


advertisers. This broad scope ensures that a wide range
DARK PATTERNS of digital entities is covered under these regulations.
Why in News: The Department of Consumer Affairs,
• Objective: The primary objective of these guidelines is
Government of India, has released Draft Guidelines for
to identify and regulate practices that manipulate or
Prevention and Regulation of Dark Patterns and is seeking alter consumer choices through deceptive or misleading
public input and feedback on these guidelines. techniques, as well as manipulated web designs and
interfaces. The guidelines aim to protect consumer
Key points of draft guidelines:
interests and create a fair and transparent digital
• Definition of Dark Patterns: The guidelines define dark marketplace.
patterns as deceptive design practices in user interfaces
(UI) and user experiences (UX) on any platform. These • Industry Commitment: These guidelines are seen as a
practices are designed to mislead or trick users into means to strengthen the industry and protect consumer
taking actions they did not intend to take, thereby rights.
subverting consumer autonomy and violating consumer
rights. • Public Consultation: The department is actively seeking
• Applicability: The guidelines will apply to all public comments, suggestions, and feedback on the draft
individuals and online platforms, including sellers and guidelines.

110 SCIENCE & TECHNOLOGY


Specified Dark Patterns as per guidelines Conclusion

• False Urgency: This involves falsely creating a sense Addressing the issue of dark patterns is essential for
of urgency or scarcity to mislead users into making promoting trust, transparency, and fairness in the digital
immediate purchases or taking immediate actions. world. Regulatory bodies and consumer advocates are
increasingly scrutinizing dark patterns and advocating for
• Basket Sneaking: This includes adding additional items stronger measures against their use.
(products, services, donations) to a user’s shopping
basket during checkout without their consent, resulting EXPECTED QUESTION FOR UPSC CSE
in higher costs than intended.
Prelims MCQ
• Confirm Shaming: This practice uses fear, shame,
ridicule, or guilt to nudge users into specific actions, such Q. The term, ‘Dark Patterns,’ recently mentioned in
as making a purchase or continuing a subscription. the news, was used in the context of:

• Forced Action: It entails forcing users to take actions (a) Biotechnology     (b) Cyber Security
that require them to buy additional goods or subscribe
to unrelated services to access the originally intended (c) Black holes       (d) Corona of the Sun
product or service. Answer: b
• Subscription Trap: This involves making it difficult or
complex for users to cancel a paid subscription, often
through deceptive practices. INDIA’S FIRST LARGEST
• Interface Interference: Design elements that manipulate INDIGENOUSLY DEVELOPED
the user interface to highlight certain information while NUCLEAR POWER PLANT
obscuring other relevant information, misdirecting users
from their intended actions. Why in News: India’s first indigenous 700 MWe N-plant
starts working at full capacity in Gujarat.
• Bait and Switch: This practice advertises one outcome
based on the user’s action but deceptively delivers a About KAPP- 3
different result.
• India’s KAPP-3 reactor is a significant milestone as the
• Drip Pricing: Elements of prices are not revealed upfront country’s first indigenous 700 MWe Pressurized Heavy
or are hidden within the user experience, potentially Water Reactor (PHWR), representing a substantial
leading to unexpected costs. advancement from the previous 540 MWe PHWR design
• Disguised Advertisement: It involves posing at Tarapur.
advertisements as different types of content, such as user- • Four 700 MWe reactors are currently under construction
generated content or news articles. at Kakrapar (KAPP-3 and 4) and Rawatbhata (RAPS-7
• Nagging: Users experience an overload of requests, and 8).
information, options, or interruptions unrelated to their • They will serve as the foundation for a new fleet of 12
intended purchase, disrupting the user’s transaction. reactors approved in 2017.
These types of dark patterns are deceptive design strategies • PHWRs use natural uranium as fuel and heavy water as a
used in online interfaces to manipulate user behaviour, moderator, forming the core of India’s nuclear reactor fleet.
often to the detriment [hinder] of consumer choice and About PHWR
rights. The guidelines aim to prevent and regulate such • The development of PHWR technology in India began
practices. in the late 1960s with the construction of the 220 MWe
Rajasthan Atomic Power Station, with full support from
Dark patterns Canada.
• Over time, India progressively reduced import content
These patterns exploit cognitive biases and behavioral
 
and increased indigenization.
psychology to trick users into making unintended
decisions, often for the benefit of the company or Safety Features of PHWR
organization behind the design. • Use of thin-walled pressure tubes: instead of large
pressure vessels, which disperses pressure boundaries
Dark patterns are unethical and can harm user trust and
  among numerous small-diameter pressure tubes,
experience. reducing the severity of potential accidents involving
pressure boundary rupture.

SCIENCE & TECHNOLOGY  111


• Dedicated Passive Decay Heat Removal System: Current methods for diagnosis and treatment of cancer
ensuring the removal of decay heat from the reactor core
without requiring operator intervention. Lung cancer is typically detected by any abnormality in
the chest radiograph or the presence of systemic or local
• This safety measure aligns with international practices
for mitigating accidents like the Fukushima incident in symptoms like cough, hard breathing, wheezing, blood in
Japan in 2011. sputum, weight loss, and hard swallowing.

EXPECTED QUESTION FOR UPSC CSE The diagnosis of lung cancer is related to the type of lung
cancer (SCLCs, NSCLCs), position of primary tumor, size of
Prelims MCQ tumor, metastasis, and clinical grade of the individuals.
Q. The terms, ‘KAPP-3 and RAPS-7,’ recently
mentioned in the news, are associated with Advantages of Using Hybrid Nanoparticles for cancer
detection and treatment
(a) Exoplanets
(b) New varieties of Rice • Hybrid systems have exhibited superiority in loading
(c) Nuclear energy efficiency, release kinetics, cellular uptake, and
(d) Varients of the Dengue viruses cytotoxicity in in-vitro and in-vivo compared with non-
hybrid systems.
Answer: c
• Hybridization of metal nanocarriers with endogenous
substances/lipids causes a diminution in toxicity
by reducing the association of metals with cells.
HYBRID NANOPARTICLES TO This hybridization is often helpful in stabilizing
CURE CANCER nanocarriers.
Why in News: The development of hybrid nanoparticles • Hybridization of inorganic nanocarriers with polymers
made of gold and copper sulphide by scientists at the Indian having anti-oxidant properties causes the reduction of
Institute of Science (IISc) represents a promising advancement ROS generated by inorganic nanocarriers
in the field of cancer detection and treatment.
What are Hybrid Nanoparticles?
¾ It refers to nanoparticles composed of two or more
different materials, often combined to take advantage
of the unique properties of each component.
¾ These nanoparticles can be engineered to have a
wide range of functionalities and find applications in
various fields, including medicine, materials science,
electronics, and environmental science.
About New Development
• These nanoparticles have unique properties that make
them potentially useful for both detecting and killing
cancer cells.
• The hybrid nanoparticles also possess photoacoustic Nanoparticles
properties. They can absorb light and generate ultrasound
waves, which can be used to detect cancer cells with high Nanoparticles are tiny particles or structures with
 
contrast once the nanoparticles reach them. dimensions typically in the nanometre range, which
• This property can aid in more accurate imaging and is between 1 and 100 nanometres (nm). They can be
measurement of oxygen saturation in tumours, composed of various materials, including metals,
improving cancer detection. metal oxides, semiconductors, polymers, and
• The researchers highlight the potential for integrating biological substances.
these nanoparticles with existing cancer detection and
treatment systems. Nanoparticles exhibit unique and often enhanced
 
properties compared to their bulk counterparts due
• For example, the nanoparticles can be activated to
to their small size and high surface area-to-volume
produce heat by shining light on them using an endoscope
ratio.
typically used for cancer screening.

112 SCIENCE & TECHNOLOGY


Conclusion Which one of the following is correct in respect of the
The hybrid NPs applicable for lung cancer are categorized above statements?
into diagnostic, therapeutic, and theragnostic multifunctional
systems. By using hybrid NPs, the drawbacks of the (a) 
Both Statement-I and Statement-II are correct
conventional nanoparticulate delivery systems in terms and Statement-II is the correct explanation for
Statement-I
of low water solubility, non-specific targeting, and poor
therapeutic outcomes could be solved. (b) Both Statement-I and Statement-II are correct and
Statement-II is not the correct explanation for
EXPECTED QUESTIONS FOR UPSC CSE Statement-I
Prelims MCQs
(c) Statement-I is correct but Statement-II is incorrect
Q1. With reference to the Nanoparticles, consider the
following statements: (d) Statement-I is incorrect but Statement-II is correct
1. 
Nanoparticles have reduced surface area to Answer: a
volume compared to larger particles.
2. 
Nanoparticles can be used for targeted drug
Descriptive Question
delivery and medical imaging.
Q. 
Discuss the unique properties of nanoparticles
Select the correct answer using the code given below:
and their significance in various scientific and
(a) 1 only        (b) 2 only technological domains. Highlight their advantages
and potential challenges in healthcare applications.
(c) Both 1 and 2     (d) Neither 1 nor 2 [15 Marks] [250 Words]
Answer: b

Q2. Consider the following statements:


NEW SCIENCE AWARDS
Statement I: Hybrid Nanoparticles are useful in more Why in News: Recently, the Central government has come
accurate medical imaging of cancer tumours. out with a new set of National Awards in the field of Science,
Technology and Innovation known as “Rashtriya Vigyan
Statement II: Hybrid Nanoparticles can absorb light
Puraskar (RVP).
and generate ultrasound waves.

SCIENCE & TECHNOLOGY  113


Key Details: Objective
On the lines of civilian awards, Government will now The objective of RVP is to recognize the notable and
recognise scientists with Rashtriya Vigyan Puruskar under inspiring contribution made by the scientists, technologists,
four categories — and innovators individually or in teams in various fields of
1. 
Vigyan Ratna - Vigyan Ratna award will recognize science, technology and technology­led innovation.
lifetime achievements & contributions made in any field
of science and technology. Eligibility
2. 
Vigyan Shri - Vigyan Shri award will recognize
A. Scientists, technologists, and innovators in government
distinguished contributions in any field of science and
or private organizations who have made significant
technology.
societal impact through groundbreaking research,
3. 
Vigyan Yuva–Shanti Swarup Bhatnagar - Vigyan
technology-led innovation, or significant technological
Yuva-Shanti Swarup Bhatnagar award will recognize &
development are eligible for the award.
encourage young scientists up to the age of 45 years who
made an exceptional contribution in any field of science B. People of Indian Origin staying abroad with exceptional
and technology. contributions benefiting the Indian communities or
4. 
Vigyan Team - Vigyan Team award to be given to a team society shall also be eligible for the awards.
comprising of three or more scientists.

Domains

Physics Chemistry Biological Sciences Mathematics & Earth Medicine


Computer Science Science

Engineering Sciences Agricultural Science Environmental Science Technology & Atomic Space
Innovation Energy Science and
Technology

Others.

The representation from each domain or field, including • These awards shall be announced on 11th May (National
gender parity will be ensured. Technology Day) every year.

Number of awards • The Award Ceremony for all categories of awards will be
held on 23rd August (National Space Day).
• There will be 3 Vigyan Ratnas awarded yearly, 25 Vigyan • All Awards will have a Sanad & a medal.
Shri, 25 Vigyan Yuva, and 3 Vigyan Team.

• Only Vigyan Yuva will have an age limit of 45 years. Nomination of Scientists

• The awards can also be given to Persons of Indian Origin • Each year, a committee will be established to supervise
(PIOs). the selection procedure.

¾ However, only one PIO can receive the Vigyan • Six Secretaries of Science Ministries, up to four Presidents
Ratna award. of Science and Engineering Academies, and six eminent
scientists and technicians from diverse sectors will be
¾ Three PIOs can be selected for both the Vigyan Shri among the committee’s prominent members.
and the Vigyan Yuva awards.
• The awards program will initially be managed by
• PIOs are not eligible for the Vigyan Team awards. the Council of Scientific and Industrial Research
Nomination and Ceremony (CSIR).
• The nominations for this bouquet of awards will be
• The proposed National Research Foundation will
invited every year on 14th January which would remain
assume control of the administration after two years.
open till 28th February (National Science Day) every
year.

114 SCIENCE & TECHNOLOGY


• Purpose: To recognize outstanding Indian contributions
EXPECTED QUESTION FOR UPSC CSE
to science and technology.
Prelims MCQ • The Shanti Swarup Bhatnagar Awards, each of the value
of Rs 5,00,000 (Rupees five lakh only), are awarded
Q. Consider the following pairs: annually by the CSIR for notable and outstanding
research, applied or fundamental, in
Award Honors for
(a) Biological Sciences
1. Vigyan Ratna Distinguished contributions in the
field of science and technology (b) Chemical Sciences
(c) Earth, Atmosphere, Ocean, and Planetary Sciences

2. Vigyan Shri Lifetime contributions in the field (d) Engineering Sciences


of science and technology
(e) Mathematical Sciences
3. Vigyan Yuva– Exceptional contribution by young
Shanti Swarup scientists in the field of science and (f) Medical Sciences, and
Bhatnagar technology (g) Physical Sciences.

How many of the above pairs are correctly matched? • Eligibility for the award
¾ Any citizen of India engaged in research in any field
(a) Only one     (b) Only two of science and technology up to the age of 45 years.
¾ Overseas citizen of India (OCI) and Persons of
(c) All three      (d) None Indian Origin (PIO) working in India are also
Answer: a (3 only) eligible.
¾ The Prize is awarded on the basis of contributions
made through work done primarily in India during
SHANTI SWARUP the five years preceding the year of the Prize.
BHATNAGAR AWARDS EXPECTED QUESTION FOR UPSC CSE

Why in news: After nearly a year’s delay, the Council of Prelims MCQ
Scientific and Industrial Research (CSIR) has publicised the
list of winners of the Shanti Swarup Bhatnagar (SSB) awards Q. For outstanding contribution to which one of the
for 2022. following fields is Shanti Swarup Bhatnagar Prize
given? [UPSC CSE 2009]
Shanti Swarup Bhatnagar Awards:
• They are the highest multidisciplinary science awards in
(a) Literature   (b) Performing Arts
India.
(c) Science    (d) Social Service
• The Shanti Swarup Bhatnagar (SSB) Prize is instituted
in the memory of the founder and the first Director- Answer: (c)
General (D-G) of the Council of Scientific and Industrial
Research (CSIR).

SCIENCE & TECHNOLOGY  115


SECTION

F DEFENCE & SECURITY

Key details:
EXERCISE SIMBEX 23 • The first phase of ‘Varuna-2023’ was conducted off
Why in News: Indian Naval Ships Ranvijay and Kavaratti India’s Western Seaboard.
and submarine INS Sindhukesari arrived in Singapore • Indian & French Navy bilateral naval exercise was
to participate in the 30th edition of the Singapore India initiated in 1993.
Maritime Bilateral Exercise (SIMBEX). • The exercise was later christened as ‘Varuna’ in 2001
and has since become a hallmark of robust India-France
Key details: strategic bilateral relationship.
• It is an annual bilateral Naval exercise between the India • It has promoted fortified common certainty between
and Singapore. operability and sharing of best practices among the two
Navies.
• It is being conducted since 1994.
• Foster mutual cooperation for rules-based order at sea.
• SIMBEX holds the distinction of being the longest
continuous naval exercise that Indian Navy has with any EXPECTED QUESTION FOR UPSC CSE
other country.
Prelims MCQ
• Other Exercises between India and Singapore include
exercise Bold Kurukshetra, Trilateral Maritime Exercise Q. 
The Exercise Varuna 2023, recently mentioned
SIMTEX (With Thailand) and exercise Agni Warrior in the news was conducted between which of the
(Army). following countries?
• SIMBEX between the Indian Navy and the Republic
of Singapore Navy (RSN), is recognized as the Indian (a) USA and India   (b) India and Bangladesh
Navy’s longest continuous naval exercise with any
foreign nation. (c) India and UAE   (d) India and France

• Both navies’ units will collaborate to enhance their Answer: d


combat capabilities and their ability to carry out multi-
discipline operations in the maritime environment.

EXPECTED QUESTION FOR UPSC CSE


INDIA-INDONESIA-AUSTRALIA
TRILATERAL MARITIME EXERCISE
Prelims MCQ
Why in News: Indian Naval Ship Sahyadri Participates in
Q. ‘SIMBEX’ is the naval exercise, recently seen in the the Maiden India-Indonesia-Australia Trilateral Maritime
news is conducted among which of the following?
Exercise
(a) India - Australia    (b) India - Japan
Key details:
(c) India - Russia    (d) India - Singapore • Indian Navy’s indigenously built warship INS
Answer: d Sahyadri, participated in the maiden trilateral
Maritime Partnership Exercise with the ships and
aircraft from the Royal Australian Navy (RAN) and
Indonesian Navy.
EXERCISE VARUNA 2023
• The trilateral exercise provided an opportunity for the
Why in News: Phase Il of the 21st edition of Varuna three maritime nations to strengthen their partnership
(Varuna-23) bilateral exercise between Indian and French and improve their collective capability to support a
Navy was conducted in the Arabian Sea. stable, peaceful and secure Indo-Pacific region.

116 DEFENCE & SECURITY


• About INS Sahyadri: Indian Coast Guard
¾ INS Sahyadri, the third ship of the indigenously
designed and built Project-17 class multirole stealth It is a maritime law enforcement and search and rescue
 
frigates was built at Mazagon Dock Ltd. agency of India with jurisdiction over its territorial
• It offered an invaluable opportunity for the participating waters including its contiguous zone and exclusive
navies to share their knowledge and expertise. economic zone.
 ICG was established in August 1978 by the Coast Guard
OPERATION SAJAG Act, 1978 as an independent armed force of India.
 As the fourth largest Coast Guard in the world, it has
Why in News: The Indian Coast Guard recently conducted
played a significant role in securing the Indian coasts
‘Operation Sajag,’ a Coastal Security Drill along the Western
and enforcing regulations in the maritime zones of India.
Coast.
 Parent Ministry: Ministry of Defence
Key details
• The drill facilitates revalidation of the coastal security  The organization is headed by the Director General
mechanism and bring awareness among the fishermen Indian Coast Guard (DGICG).
at sea.  HQ: New Delhi
• The drill is conducted every month.
• The drill enables to verify implementation of various
EXPECTED QUESTION FOR UPSC CSE
coastal security measures besides bringing out important
lessons and highlight trends in coastal security. Prelims MCQ
• During the drill, extensive checking and verification of
documents and crew passes of all fishing boats, barges Q. 
The ‘Operation Sajag,’ recently mentioned in the
and craft at sea was undertaken. news, is used in the context of:
• Measures taken for coastal security: (a) Coastal security drill conducted by the Indian
Coast Guard.
¾ Issuance of Biometric cards for the fishermen,
(b) Rescue of stranded Indian citizens in conflict-hit
¾ colour coding of fishing boats as per each state,
Sudan.
¾ manning of fish landing centres
(c) Military exercise between India and Mangolia.
¾ access control at entry/exit check points,
(d) 
Nationwide mock drill in hospitals to test
¾ coastal mapping,
emergency preparedness.
¾ designating specific marine band frequency for Answer: a
security agencies,
¾ training of marine police personnel by Indian Coast
Guard

DEFENCE & SECURITY  117


SECTION

G SOCIAL ISSUES

From when do we celebrate the International Literacy Day?


INTERNATIONAL LITERACY DAY • Its roots trace back to the World Conference of Ministers of
Education on the Eradication of Illiteracy, held in Iran, in 1965.
Why in News: Since 1967, the UNESCO
• This conference sparked the idea of a day dedicated to
celebrating International Literacy Day (ILD) every year on
promoting literacy on a global scale.
8 September. It serves as a global reminder of the profound
significance of literacy in human rights and societal dignity. • Subsequently, UNESCO officially declared September
8 as International Literacy Day during its 14th General
Conference in 1966.

What is the Significance of International Literacy Day? What is the Theme for the year 2023?
• According to UNESCO, approximately 775 million people • “Promoting Literacy for a World in Transition” - The
around the world lack basic literacy skills. Shockingly, theme emphasizes literacy’s pivotal role in navigating
one in five adults remains illiterate, with a significant global transitions and fostering sustainable and peaceful
gender gap as two-thirds of these individuals are women. solutions.
• Moreover, millions of children are absent from school • Literacy empowers individuals to adapt to changes,
or attend irregularly, and the consequences of the participate in sustainable development, and promote
COVID-19 pandemic have exacerbated these challenges. peace and understanding.
• International Literacy Day puts spotlight on progress • Literacy is not merely a skill; it is a transformative force
made in literacy rates and to develop strategies that that empowers individuals and societies to thrive in a
ensure literacy becomes accessible to all. rapidly changing world.

118 SOCIAL ISSUES


What is Literacy and how it is different from Education? Continuing Challenge of Low Literacy Rates in India
• At its core, literacy embodies the ability to read and write. • India faces a significant issue of illiteracy, with a literacy
• However, its broader interpretation encompasses the rate of 74.04%, but gender disparities persist at 82.14%
capacity to comprehend, interpret, and critically engage for males and 65.46% for females.
with written communication in various forms. • Despite 75 years of independence, 313 million illiterate
• While a person is considered literate if they can read, individuals remain, with 59% of them being women.
write, and communicate through written language, this • Factors contributing to illiteracy include linguistic
definition is further expanded in the Census of India to diversity, socio-economic factors, and the quality of
include those aged seven and above who can both read education.
and write with understanding in any language. • Illiteracy impacts economic opportunities, poverty, access
to essential services, and perpetuates social inequality.
Education Literacy
¾ It encompasses a ¾ It centres on Addressing illiteracy
wider spectrum of reading, writing, • Addressing illiteracy requires a multifaceted approach,
cognitive and personal and conveying including equitable access to education, quality
development, extending ideas through enhancement, and engagement with marginalized
beyond literacy to written language, communities.
include holistic learning enabling access to • A holistic effort is essential to break the vicious cycle of
experiences. information and illiteracy and unleash India’s potential for inclusive and
communication sustainable development.

SOCIAL ISSUES  119


Conclusion

While India has made significant progress in improving


literacy rates over the years, there is still work to be done,
particularly in rural and marginalized communities. By
implementing New Education Policy 2020 in letter and
spirit, India can continue to make progress in increasing
literacy rates and ensuring that education is accessible to all
its citizens.

EXPECTED QUESTION FOR UPSC CSE

Prelims MCQ

Q. Consider the following statements with reference to


International Literacy Day:

1. 
It is outcome of the World Conference of
Ministers of Education on the Eradication of
Illiteracy, 1965.

2. 
It was announced by the United Nations
Educational, Scientific and Cultural Organization
Advantages
(UNESCO) during 14th UNESCO General
Conference in 1966. 1. 
Individualized Learning Experience: PAL’s software-
based approach will enable each student to have a
Which of the statements given above is/are correct? personalized learning experience.

(a) 1 only      (b) 2 only 2. 


Customized Curriculum: With PAL, the curriculum can
be customized to cater to the strengths and weaknesses of
(c) Both 1 and 2    (d) Neither 1 nor 2 each student.
Answer: b
3. 
Increased Engagement: Personalized learning is more
engaging for students because it aligns with their interests
and abilities.
DIKSHA E-EDUCATION PLATFORM
TO OFFER AI HELP 4. 
Improved Learning Outcomes: When students receive
tailored instruction and support, they are more likely to
Why in News: The National e-Governance Division (NeGD) achieve better learning outcomes.
of the Ministry of Electronics and Information Technology
5. 
Adaptive Assessment: PAL can offer adaptive
(MeitY) is set to integrate Personalised Adaptive Learning
assessments that adjust the difficulty level of questions
(PAL) into its existing Digital Infrastructure for Knowledge
based on a student’s performance.
Sharing (DIKSHA) platform.
6. 
Real-time Feedback: Students receive immediate
What is DIKSHA Platform? feedback on their progress, allowing them to correct
• DIKSHA is a national online platform for school mistakes and reinforce their understanding of topics as
education in India, initiated by the National Council for they go along.
Educational Research and Training (NCERT) under the
7. 
Teacher Support: While PAL is student-centric, it can
Ministry of Education (MoE), Government of India.
also provide valuable insights to teachers. Teachers can
• It was officially launched in 2017 and has gained track the progress of individual students and use this
widespread adoption, by most states, Union Territories, data to tailor their teaching strategies accordingly.
central autonomous bodies/boards, and the Central
Board of Secondary Education (CBSE). 8. 
Improved Gross Enrolment Ratio (GER): When
students perceive that their education is tailored to
• DIKSHA is designed to enhance the quality of education their needs and interests, they may be more inclined
in India and promote digital learning. to enrol and remain in school, thus contributing to a
higher GER.

120 SOCIAL ISSUES


EXPECTED QUESTION FOR UPSC CSE
VIDYA SAMEEKSHA KENDRAS
Prelims MCQ
Why in News: The Ministry of Education has asked States to
Q. 
With reference to the ‘Digital Infrastructure for open Vidya Sameeksha Kendras under the National Digital
Knowledge Sharing (DIKSHA),’ consider the
Education Architecture (NDEAR).
following statements:
1. 
It provides Personalised Adaptive Learning Decoding Vidya Sameeksha Kendras (VSK):
(PAL) based on student priorities. • Vidya Samiksha Kendra (VSK) is aimed at leveraging
2. It is based on the Closed Source Software (CSS) data and technology to bring a bid leap in learning
technology. outcomes.
3. It will serve as National Digital Infrastructure • This will cover data of more than 15 Lakh schools, 96
for teachers to learn and train themselves. Lakh teachers and 26 Crore students and analyse them
using big data analysis, artificial intelligence and machine
How many of the above statements are correct? learning.
(a) Only one       (b) Only two • These centres will serve as comprehensive data repositories
containing information from all educational schemes
(c) All three        (d) None administered by the Ministry of Education (MoE).
• Vidya Samiksha Kendra will be a “Force Multiplier” that
Answer: b (1 and 3 only)
will amplify the existing capabilities of people or systems,
enabling transformative impact towards outcomes.

SOCIAL ISSUES  121


NEP 2020 caters to a multitude of requirements along with Coordination in driving improvements - 'Ability to
3. 
many schemes and programs run by the Ministry of Education Amplify Actions' through timely, coordinated efforts
that benefit different facets of education & stakeholders. But based on data & insights
these schemes face challenges in effective implementation
due to lack of: Keeping in view the challenges faced, a comprehensive
1. 
Timely Visibility to Progress and relevant metrics - system is required through which the relevant stakeholders
'Ability to See' by different stakeholders on what's going on will be able to track progress of various education initiatives.
This is being done under the Vidya Sameeksha Kendras.
Insights about what’s happening - 'Ability to Make
2. 
Sense' of what's working/what's not

122 SOCIAL ISSUES


Conclusion: Q2. Consider the following statements with reference
In summary, VSKs are envisioned as data-centric centres that to the Vidya Sameeksha Kendras (VSK)?
will play a pivotal role in improving education outcomes by 1. 
It aligns with the National Education Policy
monitoring projects and student performance and facilitating 2020.
grievance redressal. These efforts align with the broader 2. 
It is a comprehensive data repository for all
framework of the National Digital Education Architecture Ministry of Education schemes.
(NDEAR) in advancing the objectives of the National
Which of the statements given above is/are correct?
Education Policy 2020.
(a) 1 only        (b) 2 only
EXPECTED QUESTIONS FOR UPSC CSE (c) Both 1 and 2      (d) Neither 1 nor 2
Answer: c
Prelims MCQs

Q1. With reference to the Vidya Samiksha Kendras


(VSKs), consider the following statements: BIMA SUGAM
1. 
It is established under the National Digital Why in News: The Insurance Regulatory and Development
Education Infrastructure (NDEAR).
Authority of India (IRDAI) claims that the proposed Bima
2. 
Its primary objective is to leverage data and Sugam is a game changer and a UPI moment for the
technology to enhance learning outcomes in insurance segment.
education.
3. 
It facilitates real-time monitoring of the various About Bima Sugam
activities under Samagra Shiksha programme. • It’s an online platform where customers can choose a
suitable scheme from multiple options given by various
How many of the above statements is/are correct? companies.
• All insurance requirements, including those for life,
(a) Only one       (b) Only two
health, and general insurance (including motor and
travel) will be met by Bima Sugam.
(c) All three       (d) None
• This platform will help in the settlement of claims,
Answer: c whether it’s health coverage or death claims, in a
paperless manner on the basis of policy numbers.

SOCIAL ISSUES  123


• The overall budget for Bima Sugam has been hiked to Rs
which is responsible for managing and regulating
200 crore from around Rs 85 crore.
insurance and re-insurance industry in India.
Its role and utility for customers IRDAI is a 10-member body-
• The platform would act as a single window for the � a chairman,
policyholder to manage his/her insurance coverage. � five full-time members and
• It will provide end-to-end solutions for customers’
� four part-time members.
insurance needs i.e., purchase, service, and settlement in
a seamless manner. • It was constituted under an Act of Parliament in
1999.
• It will facilitate insurance companies to access the
validated and authentic data from various touch points • Headquarters: Hyderabad.
on a real-time basis.
• The platform will interface for the intermediaries EXPECTED QUESTION FOR UPSC CSE
and agents to sell policies and provide services to
policyholders and reduce paperwork. Prelims MCQ

Need of the scheme: Q. Consider the following:

• Currently, there are hundreds of insurance schemes in 1. Life Insurance Companies


the life and non-life sectors.
2. General Insurance Companies
• Customers have no idea who is offering the best deal and
the pros and cons of different schemes. 3. Microfinance Institutions
• Bima Sugam will enable them to identify a suitable
scheme for the customers in a single platform. 4. Re-Insurance Companies
• This will help in increasing the penetration of insurance How many of the above are regulated by the Insurance
in the country which is 4.2%, which is much less than Regulatory and Development Authority of India
global average of 7%. (IRDAI)?

About IRDAI (a) Only one    (b) Only two


The Insurance Regulatory and Development Authority (c) Only three    (d) All four
of India (IRDAI) is an autonomous and statutory body Answer: c (1, 2 and 4 only)

124 SOCIAL ISSUES


SECTION

H HISTORY & CULTURE

NEW PARLIAMENT BUILDING

EXPECTED QUESTION FOR UPSC CSE 3. The design of the Rajya Sabha chamber is based
on India’s national bird Peacock.
Prelims MCQ
How many of the above statements are correct?
Q. 
With reference to ‘Central Vista,’ consider the
following statements:
(a) Only one     (b) Only two
1. The design of the Lok Sabha chamber is inspired
from the Lotus flower.
(c) All three      (d) None
2. ‘Simhah’ is the name of one of the entrances of
the Parliament building. Answer: d

HISTORY & CULTURE  125


OLD PARLIAMENT BUILDING

126 HISTORY & CULTURE


Features:
EXPECTED QUESTION FOR UPSC CSE
• Circular Design: Unlike conventional Hindu temples
Prelims MCQ dedicated to a single deity, the Chausath Yogini Temple
boasts a unique circular design. Its circular structure
Q. 
Consider the following statements regarding old encompasses 64 chambers, each dedicated to one of the
Parliament Building of India: 64 yoginis. In the centre lies a shrine dedicated to Lord
Shiva.
1. It was designed by British architect Sir Edwin
Lutyens and German Architect Herbert Baker. ¾ The Indian parliament on the other hand has 144
columns, and is surrounded by gardens.
2. It witnessed the passing of India’s Constitution. • Hypaethral Architecture: What sets this temple apart is
its hypaethral architecture, characterized by the absence
3. It will be called “Samvidhan Sadan” (Constitution
House). of a roof. This design choice allows visitors to gaze
up at the sky, connecting with the divine and nature
How many of the above statements are correct? simultaneously.
• Stone Pillars: Resembling the columns of the Indian
(a) Only One    (b) Only two Parliament, the temple features impressive stone pillars
on the interior, creating a striking visual impact.
(c) All three     (d) None
• Historical Significance: The temple’s historical purpose
Answer: (b) (2 & 3)
extends beyond religious rituals. It is believed that it was
once utilized for the study of astrology and mathematics,
reflecting the multifaceted role temples played in social
CHAUSATH YOGINI TEMPLE life ancient India.

Why in News: The Chausath Yogini Temple is at the centre EXPECTED QUESTION FOR UPSC CSE
of an interesting debate as it is believed to have possibly
Prelims MCQ
inspired the design of India’s iconic Parliament building.
Q. With reference to Chausath Yogini Temple situated
The Link to India’s Parliament Building
near Morena, consider the following statements:
• A mere look at the temple is enough for many to [UPSC CSE 2021]
register the resemblance between the temple and Old
Parliament with its 144 columns. 1. It is a circular temple built during the reign of
• Both are circular structures with pillars lining their outer Kachchhapaghata Dynasty.
walls and have a central chamber. 2. It is the only circular temple built in India.
• Though there is no concrete evidence that architects
Edwin Lutyens and Herbert Baker, who designed the 3. It was meant to promote the Vaishnava cult in
Indian Parliament, visited the Chausath Yogini Temple, the region.
local lore suggests a connection.
4. Its design has given rise to a popular belief that it
• Lutyens and Baker embarked on tours of ancient and was the inspiration behind the Indian Parliament
medieval sites across northern and central India for building.
inspiration during the planning of New Delhi.
• This journey aimed to incorporate Indian architectural How many of the above statements are correct?
elements into the design of New Delhi’s iconic structures,
(a) Only one    (b) Only two
including the Parliament building.
• While their direct exposure to the temple remains (c) Only three    (d) All four
uncertain, the enduring mystery of their architectural Answer: b (1 and 4 only)
influences persists.
About the Chausath Yogini Temple
• The Chausath Yogini Temple nestled atop a hillock UNESCO WORLD HERITAGE SITES
in Mitaoli, Madhya Pradesh, stands as a testament to IN INDIA
India’s rich architectural heritage and spiritual traditions.
• Built around 1323 by King Devapala of the Why in News: The sacred ensembles of Karnataka’s Hoysala
Kachchhapaghata dynasty, this circular marvel is Temples and Santiniketan, a town in West Bengal have
dedicated to the 64 yoginis, powerful warriors and secured a place on the UNESCO World Heritage list.
sorceresses of Hindu mythology.

HISTORY & CULTURE  127


Santiniketan’s UNESCO Recognition: Unique Features of Hoysala Temples
• Santiniketan, meaning "abode of peace," is situated in • Stellate Plan: The Hoysala temples are characterized
West Bengal by a stellate plan, which sets them apart from other
• At Santiniketan, Rabindranath Tagore laid the architectural styles in India. This unique design features
foundations of Visva-Bharati University in 1921. multiple points radiating from a central core.
• On a global scale, Santiniketan has become India's 41st
World Heritage Site to join UNESCO's WHS list. • Soapstone (Chlorite Schist): These temples are
• This recognition stands as a testament to the enduring constructed using soapstone, also known as chlorite
cultural and historical significance of this place. schist. This material is soft and amenable to detailed
carving, enabling the creation of intricate sculptures and
friezes.

• Signed Sculptures: One of the distinctive features of these


temples is that many sculptures are signed by artists, a
rarity in the history of Indian art. This provides valuable
insights into the craftsmen behind these incredible works.

• Ornate Doorways: The temple doorways are adorned


with intricate carvings, showcasing the unparalleled
craftsmanship of the artists. These carvings often depict
scenes from Hindu mythology and history.

Preservation, Recognition Cultural Significance

Sacred Ensembles of the Hoysalas • All three temples are meticulously protected by the
Archaeological Survey of India (ASI), ensuring their
• The Sacred Ensembles of the Hoysalas consist of a preservation for future generations.
collection of Hoysala temples.
• These were built in the 12th-13th centuries and are • These Hoysala temples represent not only architectural
represented here by the three components of Belur, marvels but also the rich cultural and artistic heritage
Halebid, and Somnathapura have received the of India.
prestigious UNESCO World Heritage status.
• Hoysala temples maintain a fundamental Dravidian
morphology, they exhibit substantial influences from
the Bhumija style prevalent in Central India, the
Nagara traditions of northern and western India, and
the Karnataka Dravida modes favoured by the Kalyani
Chalukyas.
Architectural Marvels
1. elur's Chennakeshava Temple: Located in Hassan
B
district, it is renowned for its intricate architecture
and detailed carvings. It is a masterpiece of Hoysala
architecture, representing a harmonious blend of art and
religion.
2. alebid's Hoysaleshwara Temple: Also situated
H
in Hassan district, it is celebrated for its remarkable
sculptures, including depictions of gods, goddesses, and
mythological beings. It showcases the Hoysala style's
Benefits of being listed on the World Heritage List
ornate design and rich symbolism.
3.  omanathapur's Keshava Temple: Found in the Mysuru
S  International recognition and prestige
district, it is distinguished by its elegant and intricate  Legal protection under the World Heritage Convention
carvings. The temple follows the Hoysala tradition of
using chlorite schist, allowing for detailed and delicate  Access to funding from the World Heritage Fund
carving work.
 Increased tourism revenue

128 HISTORY & CULTURE


HISTORY & CULTURE  129
• The multi-metal Adi Shankaracharya statue is installed
EXPECTED QUESTION FOR UPSC CSE
atop Mandhata Parvat hill, facing southwards towards
Prelims MCQ the Narmada River.
Importance of Mandhata Island
Q. Consider the following sites:
• Mandhata island, located on the Narmada River, is
1. Santiniketan       2. Ajanta Caves home to two of the 12 Jyotirlingas – Omkareshwara and
Amareshwara.
3. Humayun’s Tomb     4. Sundarbans
• The island is close to Mahakaleshwara Jyotirlinga at
How many of the above are the UNESCO World Ujjain.
Heritage Sites in India? • It is home to Shaivite, Vaisnavite, and Jain temples
dating back to the 14th and 18th centuries.
(a) Only one      (b) Only two

(c) Only three      (d) All four Adi Shankaracharya


Answer: d • Adi Shankara was an Indian philosopher and theologian
who expounded the doctrine of Advaita Vedanta.

• He advocated the oldest concept of Hinduism which


ADI SHANKARACHARYA (788-820 AD) explains the unification of the soul (atman) with the
Supreme Soul (Nirguna Brahman).
Why in News: Recently, a 108 feet tall statue of Hindu saint
• One of Shankaracharya’s most important works is
Adi Shankaracharya, named ‘Ekatmata Ki Pratima’ (Statue
his efforts to synthesize the six sub-sects, known as
of Oneness), was unveiled at Omkareshwar, in Khandwa
‘Shanmata.’
district, Madhya Pradesh.
¾ ‘Shanmata’, which literally translates to ‘six religions,’
is the worship of six supreme deities.

• He also founded ‘Dashanami Sampradaya,’ which talks


about leading a monastic life.
• While Shankaracharya was a firm believer in ancient
Hinduism, he condemned the ‘Mimamsa school of
Hinduism’ which was purely based on ritual practices.

EXPECTED QUESTION FOR UPSC CSE

Prelims MCQ

Q. This philosopher condemned the ‘Mimamsa School’


of Hinduism, founded ‘Dashanami Sampradaya’
and a sub-sects called ‘Shanmata’ and proposed the
doctrine of ‘Advaita Vedanta’. This philosopher is:
(a) Adi Shankaracharya   (b) Dharmakirti

(c) Kanada        (d) Vatsyayana

Answer: a

About Statue of Oneness


• The ‘Statue of Oneness’ depicts Adi Shankaracharya as a DR. SARVEPALLI
12-year-old child, the age he was when he is said to have RADHAKRISHNAN (1888-1975)
visited Omkareshwar.
• It is part of a larger temple tourism project in the state. Why in News: Every year, India celebrates Teachers' Day on
• The temple town is located on the Mandhata island of 5th September to commemorate the birthday of Dr. Sarvepalli
Khandwa district, which will form an important tourism Radhakrishnan.
circuit, along with Ujjain, Maheshwar and Mandu
religious towns. Theme 2023: “Teachers at the heart of education recovery”

130 HISTORY & CULTURE


About Dr. Sarvepalli Radhakrishnan
Q2. The Kothari Commission (1964-66) was appointed
to carry out which of the following functions?
(a) 
To study and evaluate the public-private
partnership (PPP) model in India.
(b) 
To examine the extent of inequitable
distribution of benefits of reservation among
the Other Backward Classes (OBCs).
(c) To examine and make recommendations on
the educational system in the country.
(d) 
To identify the socially or educationally
backward classes of India.
Answer: c

Dr. Sarvepalli Radhakrishnan was the India's first Vice


President. He was born on 5 September 1888. His life was SIR M VISVESVARAYA (1861-1962)
marked by remarkable contributions:
• Philosopher and Bridge-Builder: Radhakrishnan
introduced Indian philosophy to the Western world,
bridging the gap between Eastern and Western thought.
• Active Engagement: He actively participated in the
“Andhra Mahasabha” in 1928, championing social and
cultural causes.
• Global Representation: Radhakrishnan represented
India on the global stage, including at the League of
Nations Committee for Intellectual Cooperation and
UNESCO.
• Authorship: He authored influential books like “The
Philosophy of Rabindranath Tagore” and “Indian
Philosophy.”
Why in News: In India, National Engineers’ Day is
• Honours: His contributions earned him honours such as celebrated on 15th September every year to pay tribute to Sir
the Bharat Ratna and the Peace Prize in the German Book
Mokshagundam Visvesvaraya, a prominent civil engineer
Trade.
and administrator in colonial India.
• Exemplary Ethical Values: His life exemplified ethical
values like objectivity, compassion, and integrity. About M. Visvesvaraya
Dr. Sarvepalli Radhakrishnan’s legacy continues to inspire as a • Born in Karnataka, Visvesvaraya made significant
teacher, philosopher, and global ambassador of Indian culture. contributions to public works projects and later served as
the 19th Dewan of Mysore.
EXPECTED QUESTIONS FOR UPSC CSE
• His accomplishments included introducing the block
Prelims MCQs system of irrigation, solving water quality issues in
Sukkur, inventing automatic water gate systems, and
Q1. This is the personality who was the Vice President of contributing to the construction of the Krishnaraja Sagar
India in 1952, actively participated in the “Andhra Dam.
Mahasabha” in 1928, and was honoured by Bharat • Additionally, Visvesvaraya authored books on the Indian
Ratna in independent India. This personality is economy and advocated for modern education in India.
(a) Zakir Husain • Despite his voluntary retirement in 1918, he continued
to work on various projects and educational initiatives,
(b) V. T. Krishnamachari
leaving behind a lasting legacy.
(c) Dr. Sarvepalli Radhakrishnan
(d) H. C. Mookerjee Legacy of Sir Mokshagundam Visvesvaraya
Answer: c • Significant contributions to civil engineering and
infrastructure development in India.

HISTORY & CULTURE  131


• Advocacy for modern education and establishment of
(c) Right to equality
educational institutions.
• Pioneering work in irrigation and water management. (d) Joint public service commission
Answer: b
• Commitment to improving India’s economic and
technological landscape.
• Lasting influence on engineering and education in India.
KOKBOROK LANGUAGE
HINDI DIWAS Why in News: The Twipra Students’ Federation (TSF) had
announced a 12-hour strike across different regions of the
Why in News: September 14 is observed as Hindi Diwas, or state demanding Roman script for the Kokborok language.
Hindi Day, to commemorate the Constituent Assembly of India
making Hindi the official language of the Union government. About Kokborok language
• Kokborok is the language spoken by the Borok people
What the Constitution said on official language: belonging to Tripura.
• As part of the Munshi-Ayyangar formula, Article 343 of the • The words ‘Kok’ implies ’verbal and Borok implies
Constitution as adopted in 1950 said the official language “people or human”.
of the Union shall be Hindi in Devanagari script.
• Borok are a branch of the Boro people of Assam
• The form of numerals to be used for the official purposes belonging to the Sino-Tibetan linguistic group and racial
of the Union shall be the international form of Indian Mongoloids.
numerals.
• Kokborok belongs to the Tibeto-Burman family and has
• For a period of fifteen years from the commencement of a close affinity with other language families like Bodo,
this Constitution, the English language shall continue Garo, Dimasa etc.
to be used for all the official purposes of the Union
for which it was being used immediately before such • Doulot Ahammad wrote the first Kokborok Grammar
commencement.” (1897).

• When the 15-year period came to an end, protests broke • It is one of the state languages of Tripura.
out over the fear of imposition of Hindi in large parts of • It is declared as the official language of Tripura Tribal
non-Hindi speaking India, particularly in Tamil Nadu. Areas Autonomous District Council.
• The resistance resulted in the Centre passing the Official EXPECTED QUESTION FOR UPSC CSE
Languages Act, which stated that English would continue
to be upheld as an official language along with Hindi. Prelims MCQ
• In the years that followed, the government has made
Q. Consider the following statements with reference
several efforts to propagate Hindi as the unifying
to the ‘Kokborok language’:
language of India, the celebration of Hindi Diwas being
one among them. 1. 
It is spoken by the Sino-Tibetan speaking group
of people.
Hindi Reach
• The 2011 linguistic census accounts for 121 mother 2. It is one of the state languages of Tripura.
tongues, including 22 languages listed in the 8th
3. 
It is declared as the official language of Tripura
Schedule of the Constitution.
Tribal Areas Autonomous District Council.
• Hindi is the most widely spoken, with 52.8 crore
individuals, or 43.6% of the population, declaring it as How many of the above statements are correct?
their mother tongue.
(a) Only one     (b) Only two
• The next highest is Bengali, mother tongue for 9.7 crores
(8%) — less than one-fifth of Hindi’s count. (c) All three     (d) None
Answer: c
EXPECTED QUESTION FOR UPSC CSE

Prelims MCQ Q2. With reference to Kokborok, consider the following


statements:
Q. The ‘Munshi-Ayyangar formula’ in the constituent
assembly is associated with which of the following? 1. The language relies upon the Devanagari script.
(a) Preamble 2. The language is included under the eighth
(b) Official language Schedule of the Constitution of India.

132 HISTORY & CULTURE


• Perhaps the growing devotion of Lord Jagannath (seen as
Which of the statements given above is/are correct?
an avatar of Lord Vishnu) coincided with the building of
(a) 1 only      (b) 2 only the Lingaraja Temple.
• The Temple’s presiding deity is known as Hari-Hara;
(c) Both 1 and 2    (d) Neither 1 nor 2
• Hari represents Lord Vishnu and Hara represents Lord
Answer: d Shiva.
• Non-Hindus are not permitted to enter the temple.
• The Bindusagar Lake, which is located on the temple’s
LINGARAJ TEMPLE north side, is another attraction.
• The garden of Ekamra Van, located on the western
Why in News: The Archaeological Survey of India (ASI) bank of Bindusagar, was called after Hindu legendary
plans to study the structural stability of the Lingaraj scriptures in which Bhubaneswar, the capital city of
Temple in Bhubaneswar, a significant landmark in Kalinga Odisha, was referred to as Ekamra Van or a forest of a
architecture. single mango tree.

Key details:
• ASI claimed that this will be the first attempt to reach
the first and second levels atop the Garbha Griha to
determine whether or not those aspects of the shrine are
structurally stable.
• In terms of interiors, conservation has only been done on
the ground floor.
• The Archaeological Survey of India (temple’s custodian), has
opted to carry out the work within the current fiscal year.
About Lingaraj Temple
• The Lingaraj Temple, dedicated to Lord Shiva and
established in the 11th century AD, is the biggest temple
in Bhubaneswar (Odisha).
• It is believed to have been constructed by Somvanshi
King Yayati I.
• It is constructed of red stone and is a remarkable example
of Kalinga architecture.
• The temple is split into four sections:
¾ Garbha Griha (sanctum sanctorum)
¾ Yajna Shala (prayer hall)
¾ Bhoga Mandap (offering hall)
¾ Natya Shala (dancing hall)
• The Lingaraj Temple’s main tower, built in the Deula
style, stands tall with three storeys. The sanctum
sanctorum graces the ground floor.
Why assessment is required?
• The Deula and Jagamohan sections bear ornate
sculptural patterns and figures, celebrating the rich • The second floor of the temple can be accessed through
artistic heritage of the region. four small openings (in four directions) just below the
Beki (the neck) of the temple.
• A corridor connecting the Jagamohana and the main
temple grants access to the temple’s first floor. • To ensure that the interiors are in good shape, a laser
scanning of the two floors will be conducted to ascertain
• There are 150 subsidiary shrines throughout the large
cracks and damages in the interiors of the monument.
temple complex.
• The resulting documentation will then be used to help
• Lingaraj is known as a ‘Swayambhu’ - a self-originated
plan conservation work.
Shivling.
• Another significant component of the temple is that it Use of Photogrammetry to assess temples
represents the fusion of the Shaivism and Vaishnavism • The ASI intends to conduct a photogrammetric study of
sects in Odisha. the Lingaraj temple’s exteriors this year.

HISTORY & CULTURE  133


¾ Photogrammetry is the science of measuring using photographs taken from multiple angles.

¾ Under this process, drones will be used to capture thousands of high-resolution photographs of the exterior walls of
the temple from different angles, which can be used to detect cracks or any other damages.

• Both the studies will not only help in conservation but also save the monument’s data for future reference.

Kalinga Architecture

• The architectural styles of Indian temples are widely classified as Nagara, Vesara, Dravida, and Gadag.

• The temple architecture of Odisha, on the other hand, refers to an entirely other category for its unique representations
known as the Kalinga style of temple architecture.

• This style broadly comes under the Nagara style.

• In Kalinga Architecture, basically a temple is made in two parts, a tower and a hall. The tower is known as Deula,
while the hall is known as Jagmohan.

• Both the deula and the jagmohan have richly carved walls with architectural motifs and a profusion of figures.

• The most repeated form is the horseshoe shape, which has come from the earliest times, starting with the large windows
of the chaitya-grihas.

• It is the deula which makes three distinct types of temples in Kalinga Architecture:

¾ Rekha Deula
¾ Pidha Deula
¾ Khakhara Deula.

• The first two i.e. Rekha Deula and Pidha Deula are affiliated with Vishnu, Surya, and Shiva temples, but the third i.e.
Khakhara Deula is predominantly associated with Chamunda and Durga temples.

• The sanctum sanctorum is housed in the Rekha and Khakhara Deulas, while the Pidha Deulas serve as exterior dance
and offering halls.

134 HISTORY & CULTURE


EXPECTED QUESTION FOR UPSC CSE 3. It was fought against the alliance of the Ottoman
Empire, the Austro-Hungarian Empire and the
Prelims MCQ German Empire.
Q. 
With reference to ancient Lingaraj Temple in How many of the above statements are correct?
Bhubaneswar in Odisha, consider the following
statements: (a) Only one     (b) Only two

1. It is a prime example of Kalinga architecture. (c) All three     (d) None


Answer: (b) (2 and 3 only)
2. The deity in this temple is Harirara who is a
combined form of Shiva and Vishnu.

3. It was built using the Deula style of architecture. MITĀKṢARĀ LAW


How many of the above statements is/are correct? Why in News: In a landmark judgment, the Supreme Court
(a) Only one       (b) Only two of India has affirmed that children born of void or voidable
marriages are entitled to inherit their parents’ share in a joint
(c) All three        (d) None Hindu family property governed by the Mitakshara law.
Answer: c
What is Mitākṣarā Law?
• It is a Hindu law school that governs the succession of
105 ANNIVERSARY OF
th property in a Hindu Undivided Family.
THE BATTLE OF HAIFA • It holds that the son, grandson, and grandson’s son have
a right to the family property through birth.
Why in News: Recently, India’s Embassy in Israel and • It also recognizes the right of daughters to inherit
Haifa’s Mayor have paid tribute to the valiant Indian soldiers property, with certain limitations.
who liberated Haifa on September 23, 1918, marking the What are Voidable Marriages?
105th anniversary of the Battle of Haifa, as part of a yearly
 A
voidable marriage is one that becomes invalid due to a
commemoration.
decree issued by either the husband or wife. In contrast,
About Battle of Haifa a void marriage is invalid from its inception.
• The Battle of Haifa was fought during First World War  T
his judgment primarily applies to the Mitakshara
as a part of the Sinai and Palestine campaign, where the school of Hindu law, which is followed across India
British Empire, the Kingdom of Italy and the French except in West Bengal and Assam.
Third Republic, fought alongside the Arab Revolt,
in opposition to the Ottoman Empire, the Austro- Mitakshara vs. Dayabhaga Law
Hungarian Empire and the German Empire. • Hindu law has two primary schools, Mitakshara, and
• Significance of the war Dayabhaga, each with distinct rules of inheritance.
¾ The First World War’s most significant battle resulted • Mitakshara law, prevalent in most of India, dictates that
in the Partition of the Ottoman Empire, leading to the property is inherited only by those born into the family.
establishment of Turkey (1923), Iraq (1932), Lebanon • Dayabhaga law, practised in West Bengal and Assam,
(1943), Jordan and Syria (1946), and Israel (1948). allows any adult coparcener, irrespective of gender, to
enforce a partition of coparcenary property.
EXPECTED QUESTION FOR UPSC CSE • Dāyabhāga states that the father is the sole ruler of all
property, both ancestral and personal.
Prelims MCQ
• Unlike the Mitākṣarā, ancestral property is not seen
Q. With reference to the Battle of Haifa, consider the as communal, therefore the father does not require the
following statements: consent of his sons to act over the ancestral property.

1. It was fought in the course of Second World Supreme Court’s Interpretation
War. • Section 16 of the Hindu Marriage Act confers legitimacy
upon children born from void or voidable marriages.
2. The Jodhpur Lancers were associated with this
battle. • The Supreme Court has emphasized that this intent of
granting legitimacy should also apply to the Hindu
Succession Act, which governs inheritance.

HISTORY & CULTURE  135


• Children born from such marriages are considered
Which of the statements given above is/are correct?
“legitimate kin” and cannot be treated as illegitimate
when it comes to inheritance under the Hindu Succession (a) 1 and 2 only    (b) 2 only
Act. (c) 1 and 3 only    (d) 1, 2 and 3 only
Answer: b
• The 2005 amendment to the Hindu Succession Act
further expanded inheritance rights, allowing a
deceased person’s share in a Mitakshara joint Hindu
family property to be inherited through testamentary or OPERATION POLO
intestate succession.
Why in News: The 75th anniversary of Operation Polo is
Conclusion a significant moment to revisit and reflect upon a crucial
The Supreme Court’s decision is a significant step towards chapter in India’s history.
recognizing the legitimacy and inheritance rights of children What was Operation Polo?
born from void or voidable marriages under the Mitakshara
Operation Polo was a military operation conducted by
law. This judgment underscores the evolving nature of legal
the Indian government in September 1948 to integrate the
principles and ensures that these children are not deprived of
princely state of Hyderabad into the newly formed Republic
their rightful inheritance.
of India.

EXPECTED QUESTIONS FOR UPSC CSE The Historical Background of Operation Polo

• Following India’s independence on August 15, 1947,


Prelims MCQs the ruler of Hyderabad chose to keep the state as an
independent entity, distinct from India and Pakistan.
Q1. With reference to the ‘Mitaksara Law,’ consider the
following statements: • To facilitate a peaceful resolution, a “standstill
agreement” was signed between Hyderabad and India
1. 
It is a Hindu law school that governs the in November 1947.
succession of property in a Hindu Undivided
Family (HUF). • This agreement stipulated that the status quo would be
maintained for a year, and India would refrain from
2. It states that the father is the sole ruler of all exerting its authority over Hyderabad during this
property, both ancestral and personal. period.
Which of the statements given above is/are correct?
Hyderabad’s Strategic Significance

(a) 1 only      (b) 2 only • Hyderabad, located in the Deccan region, was one of
India’s most populous and prosperous princely states.
(c) Both 1 and 2   (d) Neither 1 nor 2
• Comprising 17 districts, including areas now in
Answer: a Maharashtra and Karnataka, Hyderabad held a strategic
position within India.

Q2. 
With reference to the history of ancient India, • Despite lacking a direct border with Pakistan, Nizam
consider the following statements: [UPSC CSE had expressed his intention to establish friendly relations
2021] with the neighbouring country.

1. Mitakshara was the civil law for upper castes and Escalation and Threat of Secession
Dayabhaga was the civil law for lower castes. • During the period following the standstill agreement,
the administration in Hyderabad took advantage of the
2. In Mitakshara system, sons can claim property
situation.
rights while the father is alive, but in Dayabhaga
system, they can only do so after the father’s death. • They increased the strength of their irregular force,
which led to several issues, including conflicts along the
3. 
The Mitakshara system pertains to property state’s borders through cross-border raids and overtures
held by male members in a family, while the made to Pakistan.
Dayabhaga system pertains to property held by • This, coupled with the intention to establish an
both male and female members in a family. independent country within India, raised concerns
within the Indian government.

136 HISTORY & CULTURE


• Consequently, the decision to take action against Hyderabad was made to remove the threat of secession.

Operation Polo’s Execution


2. The presence of the Razakars, a private militia in
• Operation Polo was led by the Indian Army, with a Hyderabad.
primary strike from the west supported by subsidiary
thrusts from the north, south, and east of Hyderabad. 3. Border disputes between India and Pakistan.
• The Indian forces comprised various brigades and units,
with well-executed strategies to address the situation.
How many of the above were significant factors leading
The Surrender to Operation POLO?
• The Nizam of Hyderabad announced a ceasefire on
September 17, 1948. (a) Only one       (b) Only two
• On September 18, Indian forces entered Hyderabad city,
and surrender ensued. (c) All three        (d) None
• This marked the end of Nizam’s rule, and a new chapter Answer: b (1 and 2 only)
began with Hyderabad’s integration into India.

EXPECTED QUESTIONS FOR UPSC CSE Descriptive Question


Prelims MCQ Q. 
Explain the factors that led to the launch of
Operation Polo in Hyderabad and its significance
Q. Consider the following factors:
in the context of post-independence India’s
1. 
Communal tensions and violence within efforts to integrate princely states. [10 marks] [150
Hyderabad. words]

HISTORY & CULTURE  137


• The voyage will be a part of the initiative to revive and
PROJECT TO RECONSTRUCT AN honor India’s old maritime trade routes.
ANCIENT ‘STITCHED SHIP’
Significance
Why in News: The Ministry of Culture has recently
joined hands with the Indian Navy and Goa-based • It aims to revive the maritime memory and instill a sense
of pride in India’s rich maritime heritage among its
Hodi Innovations to reconstruct an ancient stitched
citizens.
ship – reminiscent of the ships that sailed the oceans on
India’s ancient maritime trade routes as many as 2,000
years ago. • This also fits in with the larger decolonisation project
undertaken, in the run-up to 2047, when independent
India turns 100.
About the Project

• Ancient stitched shipbuilding method is called Tankai Project Mausam


method.
 This initiative is in synergy with the Ministry of

• The project entails collaboration across several ministries
Culture’s Project Mausam.
and departments.
 It aims to reconnect and re-establish communications

• The Indian Navy is overseeing the ship’s design and
between countries of the Indian Ocean world, to create
construction and would also be sailing the ship along
an understanding of cultural values and concerns.
ancient maritime trade routes.
 Project Mausam aims to rebuild maritime cultural

• The Ministry of Culture has fully funded the project.
connections with the 39 countries bordering the Indian
• The ministries of Shipping and External Affairs will be Ocean.
supporting the project in its execution stage.
 Project Mausam is said to be India’s answer to the

• The project was approved by the National Maritime Silk Road of China.
Implementation Committee, chaired by Home Minister.
 India plans to move to UNESCO to award transnational

What is stitched ship technique? heritage status to Project Mausam, which was launched
by India at the 38th World Heritage Session at Doha
• This age-old technique involves shaping the wooden in 2014.
planks using the traditional steaming method to conform
to the shape of the hull.

• Each plank will then be stitched to another using cords/ EXPECTED QUESTION FOR UPSC CSE
ropes, sealed with a combination of coconut fibre, resin,
and fish oil – akin to the ancient Indian shipbuilding
practice. Prelims MCQ

• This method offers flexibility and durability to ships


Q. The term, ‘Tankai Method,’ recently mentioned in
making them less susceptible to damage from shoals and
the news, is used in the context of
sandbars.

• The ancient stitching technique almost became extinct (a) Ancient trade
after the Britishers came to India, where the wooden
planks were nailed to support the recoil of canons. (b) Shifting cultivation

The Voyage (c) World Mosquito Program

• Once the ship is ready, the voyage with a team of 13 (d) Ancient ship building
Indian Navy crew members from Odisha’s Cuttack will
Answer: d
be sent to Bali in Indonesia.

138 HISTORY & CULTURE


FESTIVALS IN NEWS

Samvatsari Nuakhai
 Samvatsari is a Jain festival celebrated especially by the  Nuakhai is an agricultural festival mainly observed by
Shwetambara sect. people of Western Odisha, Southern Chhattisgarh and
adjoining areas of Simdega (Jharkhand).
 It is the last day of an eight-day observance of prayers
called Paryushana Parva or Paryushana.  It is also called Nuakhai Parab or Nuakahi Bhetghat.

 It falls on Shukla Panchami each year in the Jain  Nuakhai is observed to welcome the new rice of the
calendar month of Bhadrapada. season.

 Samvatsari is also known as the Day of Forgiveness. It traces its origin to the Vedic period in Panchyajna.

 On this day, people seek forgiveness from others for  Nuakhai is a combination of two words that signifies
wrongs done knowingly or unknowingly. eating of new rice as ‘nua’ means new and ‘khai’ means
eat.
 People say the words Michhami Dukkadam to seek
forgiveness from others.  It is observed on panchami tithi (the fifth day) of the
lunar fortnight of the month of Bhadrapada (August–
September), the day after the Ganesh Chaturthi festival.

¾ Lagan is the fixed time of the day to celebrate the


festival.

The festival is seen as a new ray of hope, held the day


 
after the Ganesh Chaturthi festival.

EXPECTED QUESTION FOR UPSC CSE

Prelims MCQ

Q. Consider the following pairs:

Tribe : Location Tribe : Location

1. Yaifo : Papua New Guinea           2. Kawahiva : Brazil

3. Awa : Brazil                 4. Palawan : South Sudan

How many of the above pairs are correctly matched?

(a) Only one                     (b) Only two

(c) Only three                    (d) All four


Answer: c (1, 2 & 3)

HISTORY & CULTURE  139


WORLD’S MOST ISOLATED TRIBES AND THEIR HABITATS

EXPECTED QUESTION FOR UPSC CSE

Prelims MCQ

Q. Consider the following pairs:

Festival State
1. Nuakhai Juhar Odisha
2. Onam Kerala
3. Boita Bandana Odisha

How many of the pairs given above is/are correct?

(a) Only one             (b) Only two

(c) All three              (d) None


Answer: c

140 HISTORY & CULTURE


SECTION

I GOVERNMENT SCHEMES

• A targeted subsidy of Rs.200 per 14.2 kg LPG cylinder


EXPANSION OF UJJWALA YOJANA for up to 12 refills per year is being provided to PMUY
beneficiaries.
Why in News: The Union Cabinet, chaired by the Prime
Minister, has approved the extension of Pradhan Mantri About Ujjwala Yojana
Ujjwala Yojana (PMUY) for release of 75 lakh LPG • Pradhan Mantri Ujjwala Yojana (PMUY) is a government
connections over three years from Financial Year 2023-24 to scheme under the Union Ministry of Petroleum &
2025-26. Natural Gas to distribute LPG cylinders to women from
Below Poverty Line (BPL) families.
Ujjawala 2.0 • The aim of the Ujjwala Scheme is to provide alternate
clean fuel to prevent health issues associated with
• As per the existing modalities of Ujjawala 2.0, the first burning unhealthy fuels like wood, firewood etc.
refill and stove will be provided free of cost to Ujjwala • Under this scheme, 8 crore identified beneficiaries were
beneficiaries. distributed 50 million LPG gas connections, in the name
of adult women of the family.

Significance of the Scheme • The Ujjwala scheme has contributed to enhancing the
• The PMUY scheme has empowered women economically safety and security of women, as they no longer need
and socially. to venture into isolated and potentially unsafe areas to
• With easier access to LPG, women are no longer collect firewood or fuel.
burdened with the task of collecting firewood or other
Initiatives to expand LPG coverage
traditional fuels.
i. PAHAL (Pratyaksh Hastantarit Labh): LPG cylinders
• This convenience allows them to participate more
actively in community life and take up other income- were sold at market price, with subsidies transferred
generating opportunities. electronically to individual bank accounts, reducing

GOVERNMENT SCHEMES  141


“ghost” accounts and illegal commercial use,
EXPECTED QUESTIONS FOR UPSC CSE
benefiting only intended beneficiaries.
GIVE IT UP: Millions of people voluntarily
ii.  Prelims MCQ
surrendered their subsidies, redirecting funds to
those needing assistance in acquiring LPG cylinders, Q. With reference to Pradhan Mantri Ujjwala Yojana
instead of forcefully removing them. (PMUY), consider the following statements:

iii. During Covid-19 lockdown, a free refill scheme 1. It is launched under the Ministry of New and
under Pradhan Mantri Garib Kalyan Yojana was Renewable Energy.
implemented, providing Rs. 9670.41 Crore to PMUY
beneficiaries. 2. The first refill and stove will be provided free of
cost to beneficiaries under Ujjawala 2.0.
iv. Per capita consumption of PMUY beneficiaries which
was 3.01 in 2018-19 has increased to 3.71 in 2022-23. 3. It provides subsidy on LPG cylinder to all BPL
PMUY beneficiaries took more than 35 Cr LPG refills
family households.
in a year (2022-23). How many of the above statements are correct?
Achievements of Ujjwala Yojana
(a) Only one     (b) Only two
• The release of 8 Crore LPG connections under the scheme
has also helped in increasing the LPG coverage from 62% (c) All three      (d) None
on 1st May 2016 to 99.8% as on 1st April 2021.
• Reduced indoor air pollution, and better respiratory Answer: b (2 and 3 only)
health.
• Improved cooking safety and hygiene. Descriptive Question
• Enhanced quality of life, and social inclusion.
Q. Evaluate how successful the Pradhan Mantri Ujjwala
• Women empowerment seen as the scheme brought ease Yojana has been in facilitating the transition from
of living. conventional fuels to cleaner LPG-based fuels for
Recommendations by CAG household purposes. [15 Marks] [250 Words]
• Aadhaar numbers of all adult family members of existing
as well as new beneficiaries should be entered in the PRADHAN MANTRI MATSYA
system to avoid duplication.
SAMPADA YOJANA (PMMSY)
• E-KYC needs to be initiated to reap twin benefits
viz. capturing correct information and authenticating Why in News: Recently, the Minister for Fisheries, Animal
genuineness of the PMUY beneficiaries. Husbandry and Dairying addressed an event to commemorate
• LPG connections issued to minor beneficiaries may be the occasion of three years of PMMSY implementation.
transferred in the name of adult family member if the
family is otherwise found eligible under PMUY. About PMMSY
• The option of subsidizing the cost of mandatory Launched in 2020 by the Ministry of Fisheries, Animal
inspection may be explored to avoid risk hazards in the Husbandry & Dairying, the Pradhan Mantri Matsya Sampada
absence of regular inspections.
Yojana (PMMSY) is a transformative initiative aimed at
• Cases of high consumption of refills should be regularly fostering the Blue Revolution by promoting sustainable and
reviewed to curb diversion. responsible development in India’s fisheries sector.
• Third Party Audit, as envisaged in the scheme, may be
got carried out to assess the implementation of scheme.
Conclusion
Incorporating digital solutions will streamline processes,
and respecting cultural norms will foster acceptance.
Community engagement initiatives can foster peer learning
and ownership. Real-time monitoring systems will facilitate
prompt intervention. Exploring innovative, cleaner cooking
alternatives and forging collaborations among stakeholders
will further the program’s impact.

142 GOVERNMENT SCHEMES


Nationwide Implementation
Whic of the statements given above is/are correct?
PMMSY is being rolled out across all States and Union
Territories, spanning a 5-year period from FY 2020-21 to FY (a) 1 only       (b) 2 only
2024-25.
(c) Both 1 and 2      (d) Neither 1 nor 2
A Cornerstone of ‘Atma Nirbhar Bharat’
Answer: c
The program was introduced as a critical component of the
‘Atma Nirbhar Bharat’ package, featuring the highest-ever
investment of Rs. 20,050 crores in the fisheries sector. Descriptive Question

Dual Components Q. What do you understand by blue revolution.


PMMSY comprises two key components: the Central Sector Explain Pradhan Mantri Matsya Sampada Yojana.
Scheme (CS) and the Centrally Sponsored Scheme (CSS). [10 Marks] [150 Words]

Ambitious Targets
• PMMSY has set ambitious targets to transform India’s AYUSHMAN BHAV HEALTH
fisheries sector: SCHEME
• Aims for enhanced fish production of 22 million metric
tons by 2024-25. Why in News: Recently, the Ayushman Bhav campaign was
inaugurated by President of India.
• Seeks to increase the contribution of the fisheries sector’s
Gross Value Added (GVA) About the Scheme
• Aims to reduce post-harvest losses to about 10%.
• Initiated by the Ministry of Health and Family
Impressive Achievements Welfare
• Since its inception, the fisheries sector has displayed
• It is a comprehensive nationwide healthcare initiative
impressive growth, with a notable increase of 14.3% from
that aims to provide saturation coverage of healthcare
2019-20 to 2021-22.
services, reaching every village and town in the
• PMMSY has issued 1.42 lakh Kisan Credit Cards to country.
fishers and fish farmers.
• This groundbreaking initiative builds upon the success of
• Over the period from 2013-14 to 2021-22, India’s annual
the Ayushman Bharat program and signifies a paradigm
fish production witnessed a substantial surge of 66.69
shift in healthcare services.
lakh tons.
• India has solidified its position as the world’s 3rd largest • The campaign embodies a whole-of-nation and whole-
fish-producing country, contributing 8% to global fish of-society approach.
production.
• It unites government sectors, civil society organizations
• Additionally, it ranks 2nd in aquaculture production. and communities under a common mission to ensure
PMMSY is not only catalyzing the growth of the fisheries that every individual receives essential health services
sector but also bolstering India’s standing on the global without any disparity or exclusion.
stage as a significant contributor to fisheries and aquaculture • The Ayushman Bhav campaign is aligned with the
production. vision of creating ‘Healthy Villages’ and ‘Healthy
Gram Panchayats,’ laying the foundation for achieving
EXPECTED QUESTIONS FOR UPSC CSE
Universal Health Coverage in the country.
Prelims MCQ • The main aim behind rolling out this program is to
make people aware of healthy behavior. It also aims to
Q. 
With reference to the ‘Pradhan Mantri Matsya provide early diagnosis of diseases people might have.
Sampada Yojana (PMMSY),’ consider the following
statements: • Components:

1. It comprises two of key components the Central ¾ Ayushman - Apke Dwar 3.0
Sector Scheme (CS) and the Centrally Sponsored
Scheme (CSS). ¾ Ayushman Melas at Health and Wellness Centres
(HWCs) and Community Health Centres (CHCs) and
2. It has issued Kisan Credit Cards to fishers and
¾ Ayushman Sabhas in every village and panchayat
fish farmers.

GOVERNMENT SCHEMES  143


• Goals:

¾ Facilitating access to Ayushman cards

¾ Generating ABHA IDs

¾ Raising awareness about critical health schemes and disease conditions, such as non-communicable diseases,
tuberculosis, and sickle cell disease.

This campaign is targeted towards those who are excluded from the heath schemes launched by the center and state
governments. These campaigns will be organized on a weekly basis. Not only this, the campaign will be used for routine
immunization, NCD screening, mental healthcare and elderly care services among others.

EXPECTED QUESTION FOR UPSC CSE

Prelims MCQ

Q. Consider the following:

1. Apke Dwar 3.0     2. Ayushman Melas

3. Poshan tracker app   4. Ayushman Sabhas

How many of the above are the components of the Ayushman Bhav Health Scheme?

(a) Only one     (b) Only two

(c) Only three     (d) All four

Answer: c (1, 2 and 4 only)

144 GOVERNMENT SCHEMES


ADOPT A HERITAGE 2.0 PROGRAMME

GOVERNMENT SCHEMES  145


 Income Ceiling: Total family income is 8 lakhs
EXPECTED QUESTION FOR UPSC CSE
per annum.
Prelims MCQ ¾ Free Coaching Scheme for SCs and OBCs:

Q. With reference to ‘Adopt a Heritage 2.0 programme,’  The objective of the Scheme is to provide
consider the following statements: coaching of good quality for economically
disadvantaged Scheduled Castes (SCs) and
1. It enables public and private sector companies to Other Backward Classes (OBCs) candidates.
adopt heritage sites.
 It aims to enable them appear in competitive
2. It designates the individuals, public and private and entrance examinations for obtaining
entities who adopt a heritage site as Monument appropriate jobs in Public/Private Sector as well
Mitra’s. as for securing admission in reputed technical
3. 
It facilitates use of the Corporate Social and professional higher education institutions.
Responsibility funds for upkeep of the heritage  Income Ceiling: Total family income is 8
sites. lakhs per annum.
How many of the above statements are correct?  The ratio of SC: OBC students is 70:30 and 30%
(a) Only one     (b) Only two slots are reserved for females in each category.
(c) All three    (d) None • In no case, a less than 50% SC students shall be permitted.
Answer: c
¾ National Overseas Scheme for SCs:

 Under this scheme financial assistance is


SHREYAS SCHEME provided to the selected students from SCs,
De-notified, Nomadic and Semi-Nomadic
Why in News: Recently, the government informed the Tribes, landless agricultural labourers and
traditional artisan categories, for pursuing
Parliament about the success of SHREYAS scheme.
masters and Ph.D. level courses abroad.
About the Scheme:
¾ National Fellowship for SCs:
• Scheme for Higher Education Youth in Apprenticeship
and Skills (SHREYAS) is a central sector scheme for  Under the scheme fellowship is provided
providing industry apprenticeship opportunities to the to Scheduled Castes students for pursuing
general graduates through the National Apprenticeship higher education leading to M.Phil/ Ph.D
Promotional Scheme (NAPS). degrees in Sciences, Humanities and Social
• The program aims to enhance the employability of Sciences in Indian Universities/Institutions/
Indian youth by providing ‘on the job work exposure’ Colleges recognized by University Grants
and earning of stipend. Commission (UGC).
• SHREYAS is a programme comprising the initiatives of
three Central Ministries: EXPECTED QUESTION FOR UPSC CSE
¾ Ministry of Human Resource Development, Prelims MCQ
¾ Ministry of Skill Development & Entrepreneurship
and Q. 
With reference to Scheme for Higher Education
Youth in Apprenticeship and Skills (SHREYAS),
¾ Ministry of Labour & Employment.
consider the following statements:
• The umbrella scheme of “SHREYAS” comprises 4 central
1. It is a central sector scheme.
sector sub-schemes namely:
¾ Top Class Education for SCs: 2. It provides industry apprenticeship opportunities
to the graduates through the National
 The Scheme aims at recognizing and promoting Apprenticeship Promotional Scheme (NAPS).
quality education amongst students belonging
3. It provides free coaching services to students
to SCs, by providing full financial support.
belong to the SCs and OBCs communities.
 The Scheme will cover SC students for pursuing How many of the above statements are correct?
studies beyond 12th class.
(a) Only one     (b) Only two
 The scholarship, once awarded, will continue (c) All three      (d) None
till the completion of the course, subject to Answer: c
satisfactory performance of the student.

146 GOVERNMENT SCHEMES


¾ There is no income limit for SCs, SafaiKaramcharis
PM DAKSHATA AUR KUSHALATA Including waste picker and DNTs.
SAMPANNA HITGRAHI (PM- ¾ The annual family income should be below Rs.3
DAKSH) YOJANA lakh for OBCs
¾ For EBCs (Economically Backward Classes) annual
Why in News: The government recently informed the family income should below Rs.1 lakh.
Parliament about PM-Daksh Scheme.
EXPECTED QUESTION FOR UPSC CSE
About the Scheme:
Prelims MCQ
• The Pradhan Mantri Dakshata Aur Kushalata
Sampanna Hitgrahi (PM-DAKSH) Yojana is a Central
Q. With reference to the ‘Pradhan Mantri Dakshata
Sector Scheme.
Aur Kushalata Sampanna Hitgrahi (PM-DAKSH)
• The main objective of the Scheme is to enhance Yojana,’ consider the following statements:
competency level of the target groups to make them
employable both in self- employment and wage- 1. It is the centrally sponsored scheme.
employment for their socio-economic development.
• This scheme introduced by the Ministry of Social Justice 2. The Scheduled tribes are one of the targeted
and Empowerment. groups under this scheme.
• Target groups:
3. 
It lays down mechanism for Marketing of
¾ Scheduled Castes (SCs), Minor Forest Produce (MFP) through Minimum
¾ Other Backward Castes (OBCs), Support Price (MSP).
¾ Economically Backward Castes (EBCs,)
How many of the above statements are correct?
¾ De-notified Tribes (DNTs)
¾ Safai Karamcharis including waste pickers etc. (a) Only one     (b) Only two
¾ Women
• Age criteria: (c) All three     (d) None
¾ The age criterion of scheme between 18-45 years. Answer: d
• Income criteria:

GOVERNMENT SCHEMES  147


SECTION

J SNAP NEWS

DUAL-USE ITEMS • Partnership among governments, companies, and other


stakeholders is critical in dual-use export regulation.
Why in News: Recently, the government has declared that it
Why there is a need for Control over Dual-Use Items?
will strengthen export controls on dual-use items in order to
• India is committed to the non-proliferation of dual-use
prevent them from reaching non-state actors and terrorists.
items.
What are Dual-Use Items? • It is a crucial part of the country’s export management
Goods, software, and technology with dual applications, system.
suitable for both civilian and military purposes. They are • It is carried out to ensure that trade in sensitive and dual-
heavily regulated because they can be classified for civilian use items complies with India’s obligations under several
use and then transformed for military purposes, or worse, regimes.
used for terrorism.
EXPECTED QUESTION FOR UPSC CSE
• Examples: Global positioning satellites, missiles, nuclear
Prelims MCQ
technology, chemical and biological weapons, night
vision technology, thermal imaging, some models of Q. The term, ‘Dual-Use Items,’ recently mentioned in
drones, aluminium pipes with precise specifications. the news refers to

Current Regulatory Framework


(a) Technology with dual applications, suitable for
both civilian and military purposes.
Most industrial countries have export controls on certain
(b) The domestically produced items can be sold
types of designated dual-use technologies and a number of
in both domestic and international markets.
treaties signed for the same-
(c) Reusable rockets in the space industry reduce
• India is a signatory to the major multilateral export with little wear and tear.
control regimes like Missile Technology Control Regime
(d) 
Multipurpose crops like fodder and forage
(MTCR), Wassenaar Arrangement (WA) and Australia
restoring soil nutrients and yielding desired
Group (AG).
produce.
• India is also a signatory to International Conventions Answer: a
on Non-Proliferation, namely Chemical Weapons
Convention (CWC) and Biological and Toxic Weapons
Convention (BWC).
• In India, the Director General of Foreign Trade (DGFT)
functions as a facilitator of exports and imports.
MANGRI ORANG
• The DGFT prepares a special SCOMET (speciality Why in News: The North East Regional Centre (NERC) of
chemicals, organisms, materials, equipment and the Indira Gandhi National Centre for the Arts (INGCA)
technologies) list of dual-use items. staged Malati Mem, a multilingual play based on the life
What is SCOMET list? and the revolutionary zeal of Mangri Orang, an Adivasi with
roots in central India.
• Dual-use items have been designated as SCOMET under
Foreign Trade Policy.
• Under India’s Foreign Trade Policy, the export of dual-
use items and technology is restricted or only authorized
with a license.
• The SCOMET control list is consistent with all
multilateral export control regimes and conventions.
• Controlling the export of dual-use items is a difficult task
in the field of technology transfer.

148 SNAP NEWS


About Mangri Orang: About Bharat Drone Shakti:
• She was a tea plantation worker and became one of the • This event intends to promote the usage of drone
leading members of the anti-opium campaign in tea gardens technology across many sectors while showcasing the
while participating in the non-cooperation movement. potential of the Indian drone industry.
• Fellow plantation workers used to call her Malati Mem, • Over 50 live airborne demonstrations of drone technology,
the second word being a shorter form of memsahib. including survey drones, agricultural drones, firefighting
• Mangri Orang is an unsung hero of India’s struggle for drones, tactical surveillance drones, heavy-lift logistics
freedom from British rule. drones, loitering munition systems, drone swarms, and
counter-drone solutions, will be shown during the event.
• She was gunned down in 1921 for leading a fight against
foreign liquor and opium pushed during the colonial period. The occasion highlights India’s rising interest in drone
• She is said to be the first female martyr of India’s freedom technology across a variety of industries, including defense
movement and civilian uses. India hopes to become a global drone hub
by 2030.

CE-20 CRYOGENIC ENGINE EXPECTED QUESTION FOR UPSC CSE


Why in News: ISRO has successfully tested the CE-20 Prelims MCQ
Cryogenic engine to be used in “Mission Gaganyaan”.
Q. Consider the following statements with reference
About CE -20 to the ‘Bharat Drone Shakti 2023’:
• Cryogenic engines use fuels at cryogenic temperatures
   1. It was organized by the Ministry of Science and
(-150 to -273 degree C) and are comparatively more
Technology.
powerful than solid and liquid propellant engines.
• It is the first cryogenic engine to use a Gas-generator cycle    2. It was associated with drones of both the military
using Liquid Oxygen and Liquid Hydrogen propellants and the civil applications.
combination.
• It has a capacity of 28 tonnes of fuel and will be used Which of the statements given above is/are correct?
as the upper stage of Geosynchronous Satellite Launch
Vehicle Mk III (LVM3). (a) 1 only      (b) 2 only

EXPECTED QUESTION FOR UPSC CSE (c) Both 1 and 2    (d) Neither 1 nor 2

Prelims MCQ
Answer: b

Q. 
With reference to the ‘CE-20 Cryogenic engine,’
consider the following statements:

1. It is the first Indian cryogenic engine to feature a


KUNBIS
gas-generator cycle. Why in News: Recently, the Maharashtra government
decided that Kunbi caste certificates will be issued to
2. 
It uses Liquid Oxygen and Liquid Hydrogen members of the Maratha community.
propellants combination.
About Kunbis:
How many of the above statements are correct?
• Kunbis (the community associated with agriculture) are
grouped under the Other Backward Class category in
(a) 1 only      (b) 2 only
Maharashtra.
(c) Both 1 and 2    (d) Neither 1 nor 2 • When Marathwada was part of Hyderabad province till
the 1960s, Marathas there were counted as Kunbis.
Answer: c
• When the region became part of Maharashtra, many
were counted as Marathas and not OBCs.
• In 1967, Punjab Rao Deshmukh demanded OBC
BHARAT DRONE SHAKTI 2023 status for Marathas of Vidarbha region and in 2004,
the Maharashtra government issued a Government
Why in News: The Indian Air Force (IAF) is collaborating
Resolution (GR) giving Kunbi status to Marathas of that
with the Drone Federation of India to co-host ‘Bharat Drone region.
Shakti 2023.’.

SNAP NEWS  149


• The communities are largely found in the state of 2. 
It is used for making refunds in respect of
Maharashtra but also exist in the states of Madhya unclaimed dividends and matured debentures.
Pradesh, Gujarat, Karnataka, Kerala and Goa.
Which of the statements given above is/are correct?

EXPECTED QUESTION FOR UPSC CSE (a) 1 only      (b) 2 only


Prelims MCQ (c) Both 1 and 2    (d) Neither 1 nor 2
Q. Consider the following statements: Answer: c
1. Reservation for OBCs in Educational Institutions
was added by the 93rd Amendment Act of 2005.
2. The 100th Amendment Act of 2018 conferred a
constitutional status on the National Commission
VANADIUM DISCOVERY IN
for Backward Classes (NCBC). GUJARAT
3. The 102nd Amendment Act of 2018 inserted a new Why in News: Vanadium has been discovered in sediment
Article 342-A enabling the President to specify collected from the Gulf of Khambhat in Gujarat, India.
the socially and educationally backward classes.
How many of the above statements are correct? About Vanadium:
(a) Only one    (b) Only two • Vanadium is a scarce transition metal.
(c) All three   (d) None • It is characterized by its hard, silvery-grey appearance
Answer: (b) (1 and 3 only) and is ductile and malleable.
• Vanadium possesses good structural strength and
corrosion resistance.
• It occurs naturally in around 65 different minerals,
INVESTOR EDUCATION AND including patronite, vanadinite, roscoelite, and
PROTECTION FUND AUTHORITY cacnotite.
Why in News: Recently, IEPFA celebrated 7th Foundation • Vanadium is also found in bauxite, fossil fuel deposits,
Day in New Delhi. and in small percentages in meteorites.
• The primary sources of vanadium include mined iron
About IEPFA:
ore, carbonaceous shale or phyllites, and steel slag.
• Investor Education and Protection Fund Authority was
established in 2016 for administration of Investor Vanadium Reserves:
Education and Protection Fund. • China is the largest producer and consumer of vanadium.
• It was established under the provisions of section 125 of
the Companies Act, 2013. • Russia and South Africa also have significant vanadium
reserves.
• Nodal Ministry: Ministry of Corporate Affairs
• It is entrusted with the responsibility of administration • Brazil is the largest exporter of vanadium
of the Investor Education Protection Fund (IEPF).
Applications of Vanadium:
• Composition:
• In the iron and steel industry, vanadium is primarily
¾ Chairperson- Secretary, Ministry of Corporate Affairs used as an alloying element.
¾ Chief Executive Officer- Joint Secretary, Ministry of • It is valuable in the automotive and aviation industries
Corporate Affairs due to its high strength-to-weight ratio, which contributes
to increased fuel efficiency.

EXPECTED QUESTION FOR UPSC CSE • Vanadium is used in non-metallurgical applications as


a catalyst and in industries such as ceramics, chemicals,
Prelims MCQ pigments, health preparations, and electronics.
• In nuclear reactors, vanadium is employed because of its
Q. Consider the following statements with reference low neutron-absorbing properties.
to the ‘Investor Education and Protection Fund
Authority’: • Vanadium also plays a crucial role in battery technologies,
serving as secondary batteries for power plants and
1. 
It was established under the Companies Act, in rechargeable vanadium redox batteries (VRBs) for
2013. commercial applications.

150 SNAP NEWS


Vanadium Redox Battery (VRB):
EXPECTED QUESTION FOR UPSC CSE
• VRB is a type of mobile battery used for energy
storage. Prelims MCQ
• It converts energy stored in an electrolyte into electricity
Q. The term, ‘Pirola,’ recently mentioned in the news,
by exchanging electrons between vanadium ions.
refers to
• VRBs are known for their high energy efficiency, long
cycle life, and safety features. (a) Covid 19 variant     (b) Mountain range
• They are used in various applications, including large-
(c) Jaina Philosopher    (d) Ransomware
scale energy storage and grid stability, especially in
power plants.
Answer: (a)
• VRBs are instrumental in the development of renewable
energy systems as they allow for the storage of excess
energy generated from sources like wind and solar
power for later use.
KUMKIS
Why in News: Operation Gaja in Tamil Nadu shines spotlight
EXPECTED QUESTION FOR UPSC CSE on Chittoor’s kumkis.

Prelims MCQ About Kumkis:

Q. Consider the following minerals: • Kumkis are elephants trained to capture and tame wild
elephants.
1. Patronite   2. Sillimanite
• Elephant calves that are separated from the herd in the
3. Roscoelite   4. Cacnotite forest and captured wild elephants are tamed and trained
to become Kumki elephants.
How many of the above are the naturally occurring
¾ The training period is three years.
ores of the Vanadium metal?
• After training, the elephants join the ‘Kumki service’
(a) Only one    (b) Only two and are government servants until the age of 60.

(c) Only three   (d) All four • After retiring at 60 years, the elephants can relax at any
Answer: c (1, 3 and 4 only) elephant sanctuary.

• Kumki elephants are usually used to chase the wild


elephants that stray into human habitats, back into the
PIROLA forest.

Why in News: Rise of Covid-19 infections noticed in multiple • The services of Kumki elephants are also utilised to
countries driven by a new Coronavirus variant called BA.2.86 rescue wounded elephants, and to lend support to the
which is informally being termed ‘Pirola. wild elephants that have difficulty standing.

What is Pirola?

• Pirola, a recently identified highly mutating Omicron


version of the Covid-19 virus, was discovered in
Denmark at first and has since spread to the US, Canada,
and Israel.

• Pirola is distinct from earlier Omicron variants because it


has more than 30 mutations in the spike protein.

• This enables it to be immune, evasive, and extremely


contagious.

• It is now listed as one of the “variants under monitoring”


by the WHO.

SNAP NEWS  151


• Amigurumi can be worked as one piece or, more usually,
EXPECTED QUESTION FOR UPSC CSE
in sections which are sewed or crocheted together.
Prelims MCQ
EXPECTED QUESTION FOR UPSC CSE
Q. 
The term, ‘Kumkis,’ recently mentioned in the
news, refers to Prelims MCQ

(a) Tribe inhabiting the Naga hills in north east Q. The term, ‘Amigurumi,’ recently mentioned in the
India. news, refers to which of the following?
(a) A type of Jajmani Sari
(b) Trader guild existed during the Gupta Age.
(b) Japanese art of knitting
(c) 
The followers of the Advaita Vedanta (c) Wool obtained from Chiru
philosophy of the Adi Shankaracharya.
(d) Sino-Tibetan language
Answer: (b)
(d) Elephants trained to capture and tame wild
elephants.
Answer: (d)
THE ORGANIZED CRIME AND
CORRUPTION REPORTING
AMIGURUMI DOLLS PROJECT
Why in News: The Organised Crime and Corruption
Why in News: A Singapore-based firm ‘1 Million Heroes’ has
Reporting Project (OCCRP) has made fresh allegations of
taken up the responsibility to train women in relief camps
stock manipulation against the Adani Group.
across Manipur on how to crochet amigurumi dolls.
About Organised Crime and Corruption Reporting Project
(OCCRP)
• The Organized Crime and Corruption Reporting Project
(OCCRP) is a global network of investigative journalists
with staff on six continents.
• It was founded in 2006 and specializes in organized
crime and corruption.
• OCCRP works with and supports 50+ independent
media outlets in Europe, Africa, the Caucasus, and
Central Asia.
• Initially funded by the United Nations Democracy Fund
(UNDEF), the OCCRP network first opened an office in
Sarajevo, Bosnia and Herzegovina
• The idea is to have a global network of journalists with
easy communication and information-sharing so that
global networks of corruption and crime can be better
understood and exposed.
About the dolls: • The OCCRP is supported both by state and non-state
• Amigurumi is the Japanese art of knitting or crocheting institutional actors, as well as smaller individual donors.
small, stuffed yarn creatures.
• Amigurumi vary in size and there are no restrictions EXPECTED QUESTION FOR UPSC CSE
about size or look. Prelims MCQ
• Amigurumi may be used as children’s toys but are
generally purchased or made solely for aesthetic Q. 
With reference to the ‘Organized Crime and
purposes. Corruption Reporting Project (OCCRP),’ consider
the following statements:
• Amigurumi can be knitted, though they are usually
crocheted out of yarn or thread, using the basic techniques 1. It was established by the International Criminal
of crochet (such as single crochet stitch, double crochet, Police Organization (Interpol).
and invisible decrease).

152 SNAP NEWS


2. It is a global network of investigative journalists EXPECTED QUESTION FOR UPSC CSE
with staff on six continents.
Prelims MCQ
Which of the statements given above is/are correct?
Q. 
The term, ‘NOTAM,’ recently mentioned in the
(a) 1 only      (b) 2 only news, refers to
(a) A traditional performing art form in the state
(c) Both 1 and 2    (d) Neither 1 nor 2 of Kerala.

Answer: (b) (b) Buddhist monasteries where monks formally


met to practice debating.
(c) A method of the shifting cultivation practiced
in hilly region of the Tamil Nadu.
NOTICE TO AIRMEN (NOTAM) (d) 
Notice containing information essential to
personnel concerned with flight operations.
Why in News: The government has issued a Notice to Airmen Answer: (d)
(NOTAM) for the upcoming G20 Leaders’ Summit.

About NOTAM:
GUJARAT DECLARATION (WHO)
• A NOTAM is a notice containing information essential
to personnel concerned with flight operations. Why in News: World Health Organization (WHO) has
released the outcome document of first WHO Traditional
• It was originally used way back in 1947 and were Medicine Global Summit 2023 in form of “Gujarat
modeled after “Notice to Mariners”, which advised ship Declaration”.
captains of hazards in navigating the high seas.
Key details:
• NOTAMs basically provide information about potential
• The declaration reaffirmed global commitments towards
hazards or changes to flight operations that may affect
indigenous knowledges, biodiversity and traditional,
safety.
complementary and integrative medicine.
• The information can include things like:
• It reiterates that India as host of the WHO Global
¾ temporary flight restrictions, Traditional Medicine Centre at Jamnagar, Gujarat has
an important role in in taking forward the summit action
¾ changes to air traffic control procedures, agenda and other relevant priorities.

¾ runway closures, etc. • It will focus on the integration of traditional medicines


in national health systems and help unlock the power of
• NOTAMs are typically issued on a short-term basis, and traditional medicine.
they are meant to be used by pilots, air traffic controllers, • Gujarat declaration talks about scaling up efforts to
and other aviation professionals to stay informed about further implement evidence-based TCIM (Traditional
changes to flight operations. complimentary integrative medicine) interventions and
approaches in support of the goal of universal health
coverage (UHC) and all health-related Sustainable
Development Goals (SDGs).
• This was the First global summit on Traditional
Medicine and was organised by the World Health
Organization (WHO), co-hosted by the Ministry of
Ayush at Gandhinagar, Gujarat.

EXPECTED QUESTION FOR UPSC CSE

Prelims MCQ

Q. With reference to the ‘Gujarat Declaration, 2023,’


consider the following statements:

1. First global summit on Traditional Medicine by


World Health Organization (WHO).

SNAP NEWS  153


• Bordering countries include Egypt, Saudi Arabia,
2. 
Its objective is to promote resilient growth Yemen, Sudan, Eritrea, and Djibouti.
by urgently and effectively addressing debt
vulnerabilities in developing countries. • It connects to the Indian Ocean in the south through the
Bab el Mandeb Strait and the Gulf of Aden.
3. Its aims to use strict scientific methods to tap into
the full potential of traditional, complementary, • In the north, it is bordered by the Sinai Peninsula, the
and integrative medicine (TCIM). Gulf of Aqaba, and the Gulf of Suez, leading to the Suez
Canal.
How many of the above statements are correct?

(a) Only one      (b) Only two


G-20 SATELLITE MISSION FOR
(c) All three      (d) None ENVIRONMENT AND CLIMATE
Answer: (b) (1 and 3 only) OBSERVATION
Why in News: India proposed to launch a G-20 Satellite
Mission for Environment and Climate Observation, with an
ANCIENT RED SEA TRADE ROUTE
aim to help the countries of the Global South.
Why in News: The India-Middle East-Europe Economic
Corridor announced at the G20 Summit harkens to an ancient Key details:
trade route between the subcontinent and the Roman Empire
• The climate and weather data obtained from this will be
– the ancient red sea trade route.
shared with all the countries, especially the countries of
About Ancient Red Sea Trade Route the Global South.
• The Red Sea route refers to an ancient maritime trade • India invites all G-20 countries to join this initiative.
route that connected India with the Roman Empire
through the Red Sea. • India had earlier launched a satellite for the benefit of the
SAARC countries, popularly called SAARC Satellite, in
• This route facilitated the exchange of various goods,
2017 as a part of its ‘neighbourhood first policy’.
including luxuries like spices, ivory, pearls, gemstones,
and pepper from India to the Roman Empire. ¾ Its aim was to provide crucial information on tele-
• Pepper was India’s major export and was highly sought medicine, tele-education, banking and television
in Rome broadcasting opportunities to its South Asian
neighbours.

BRANDT LINE
What is Brandt Line?
• The Brandt Line conceived in the 1980s, visually
illustrates the global North-South divide, highlighting
stark economic disparities between the wealthier
Northern Hemisphere and the less affluent Southern
Hemisphere.

• Most economically developed countries, situated in


North America, Europe, and parts of Asia, lie in the
Northern Hemisphere, with Australia and New Zealand
as exceptions.

• In contrast, the Southern Hemisphere, particularly the


tropical regions, hosts many economically disadvantaged
nations facing poverty, underdevelopment, and limited
About Red Sea resources.
• The Red Sea is an inlet of the Indian Ocean located
• This concept originated from the “North-South: A
between Africa and Asia.
Programme for Survival” report, which seeks to address
• It is one of the most saline bodies of water globally. international inequality.

154 SNAP NEWS


EXPECTED QUESTION FOR UPSC CSE

Prelims MCQ

Q. The Brandt Line, recently mentioned in the news, NAIROBI DECLARATION


refers to which of the following?
Why in News: Africa Climate Summit 2023 ends with
(a) A way of visualizing wealth disparities between ‘Nairobi Declaration’.
the Global North and Global South.
Key details:
• The inaugural Africa Climate Summit (ACS23) in
(b) The border marked by river between the United
Nairobi culminated in the ‘Nairobi Declaration’, giving
States and Mexico.
the continent a common voice ahead of upcoming key
global engagements.
(c) 
Straight-Line Borders resulted from colonial- • Theme: Driving green growth and climate finance
era agreements. solutions for Africa and the World.
(d) 
It refers to a geometric concept in three- • This declaration will serve as a basis for Africa’s common
dimensional (3D) Euclidean space. position in the global climate change process.
• Highlights of the declaration:
Answer: a
¾ Making tough calls to secure funding for climate
mitigation and adaptation
¾ Green investment opportunities in Africa’s emerging
blue and green economies
BHOJSHALA AND KAMAL
¾ It also urges world leaders to back a proposed carbon tax
MAULA’S MOSQUE on fossil fuel trade, maritime transport and aviation.
¾ While calling the international community to order,
Why in News: Security has been heightened at Bhojshala
the declaration wants them to honour the annual
and Kamal Maula’s Mosque in Madhya Pradesh’s Dhar
climate finance pledge of $100bn they made, as a
district after an idol of Vagdevi (goddess Saraswati) was matter of urgency.
discovered inside the 11th-century monument, leading to
¾ Accelerating all efforts to reduce emissions in line
communal tensions.
with the Paris Agreement
• Despite the idol’s removal by authorities, around 200 ¾ Accelerating phasing down coal and abolishing all
police personnel were deployed as a Hindu group fossil fuel subsidies
threatened agitation to restore it. ¾ Operationalising Loss and Damage facility agreed at
• The site has historical and religious significance to both COP27.
the Hindu and Muslim communities. ¾ With many African countries having defaulted on
loans from developed countries, the declaration
• It is also noteworthy because it is protected by the proposes debt relief interventions.
Archaeological Survey of India (ASI), and the ASI has  It recommends extending the debt repayment
issued guidelines allowing both Hindus and Muslims period and instituting a 10-year grace period
to offer prayers on specific days, making it a place where to allow the borrowed funds to serve intended
two different religious traditions coexist. development goals.

SNAP NEWS  155


 The Nairobi summit largely focused on calls to (c) 
A specialized B2B fair in the Indian cotton
unlock investment in clean energy. textile sector offering a one-stop destination for
worldwide importers to source Indian Textiles.
One of the big pledges at the summit was a $4.5 billion
finance initiative towards unlocking Africa’s clean energy. (d) 
Economic empowerment of persons with
disabilities by encouraging disabled
EXPECTED QUESTION FOR UPSC CSE entrepreneurs/artisans.
Answer d
Prelims MCQ

Q. Consider the following statements with reference


to the ‘Africa Climate Summit 2023’: VAGUS NERVE
Why in News: Research has indicated a link between vagus
1. It resulted in the Nairobi Declaration, 2023. nerve dysfunction and long COVID.

2. 
It was organized by the United Nations What is the vagus nerve?
Economic Commission for Africa.
• They’re a pair of nerves, one on each side, that run
from the brainstem, through the neck, to the chest and
Which of the statements given above is/are correct?
stomach.
(a) 1 only     (b) 2 only

(c) Both 1 and 2   (d) Neither 1 nor 2


Answer a

DIVYA KALAMELA
Why in News: Department of Empowerment of Persons with
Disabilities (Divyangjan) organizes unique event - Divya
Kala Mela’.

Key details:
• It showcases the products and craftsmanship of Divyang
entrepreneurs/artisans from across the country.
• This is a unique initiative of DEPwD towards economic
empowerment of PwD/Divyangjan.
• The Divya Kala Mela presents a bigger platform for
marketing and showcasing the products and skills of
Divyangjan (PwD).
• The Divya Kala Mela, Varanasi is the seventh one in
series starting from 2022.

EXPECTED QUESTION FOR UPSC CSE

Prelims MCQ
• They form a key part of the parasympathetic nervous
Q. Which of the following were the objective of the system, the system that is responsible for relaxing and
Divya Kala Mela recently seen in the news? resting your body after a bout of activity and for a
number of vital functions including the heart rate, blood
(a) 
To create awareness about various skill pressure and digestion.
development programs for Scheduled Tribes.
• It also plays a role in the immune system.
(b) 
To provide plartform for the number of
employers and job seekers come together for • The vagal nerves are the longest cranial nerves, going
the purpose of applying and interviewing for from the brain to the large intestine, connecting with the
jobs neck, heart, lungs, abdomen and digestive tract.

156 SNAP NEWS


• They are the 10thof the 12 cranial nerves and contain 75% • It is named after Ramon Magsaysay, the Seventh
of your parasympathetic nervous system’s nerve fibre. president (1953-57) of the Philippines.
• They work bi-directionally, allowing the brain and body
to communicate with each other. • Ramon Magsaysay Award Foundation (RMAF) honour
• Researchers are increasingly looking at ways through individuals or organizations in Asia every year for social
which stimulation of these nerves, may potentially help contribution activities.
with various health conditions.
• Indians who Received the Ramon Magsaysay Award are:
EXPECTED QUESTION FOR UPSC CSE
 V
inoba Bhave, Mother Theresa, Verghese Kurien,
Prelims MCQ Jayaprakash Narayana, Satyajit Ray, Kiran Bedi,
Aruna Roy, Sonam Wangchuk and Ravish Kumar
Q. With reference to the Vagal nerves, consider the
following statements:
EXPECTED QUESTION FOR UPSC CSE
1. Their dysfunction can cause both slow and fast
heart rates.
Prelims MCQ
2. They form a key part of the parasympathetic
nervous system. Q. Consider the following statements with reference to
the Ramon Magsaysay Award:
3. 
These are the longest cranial nerves in the
human body. 1. The award is named after the third president of
the Philippines.
How many of the above statements are correct?

(a) Only one     (b) Only two 2. Vinoba Bhave was the recipient of the very first
Ramon Magsaysay Award.
(c) All three      (d) None
Answer c Which of the statements given above is/are correct?

(a) 1 only      (b) 2 only

RAMON MAGSAYSAY AWARD (c) Both 1 and 2    (d) Neither 1 nor 2
Why in News: Assam-based oncologist Ravi Kannan is
among the winners of the 2023 Ramon Magsaysay Award.
Answer b

ASIAN GAMES
Why in News: First batch of Indian team left for the upcoming
Asian Games for Hangzhou, China.

Evolution of Asian Games:

• Before the Asian Games, the Far Eastern Championship


Games were held.

• After the Second World War, many Asian countries


Oncologist gained independence and Guru Dutt Sondhi, a member
of the Indian International Olympic Committee, proposed
An oncologist is a healthcare provider who specializes in the idea of Asian Games as a sporting event, where all
diagnosing and treating cancer. Asian nations can be represented.

About Ramon Magsaysay Award • The first-ever Asian Games were held in New Delhi in
1951.
• Established in 1957, the Ramon Magsaysay Award is
regarded as the Asian version of the Nobel Prize

SNAP NEWS  157


Asian Games and India:
• India is a founder member of Asian Games and also the
host of the first Asian Games.
• 1982 Asian Games were also held in New Delhi.
• India is one of the seven countries to have participated
in all the editions of the Asian Games.
• India along with Japan is the only country to have won
at-least one gold medal in every event of Asian Games.
• India has always ranked in the top 10 in the medals tally
in Asian Games, except in the 1990 games.
• The torch of the 19th Asian Games is named “Eternal
19th Asian Games Hangzhou, China: Flame”. The design was inspired by the Liangzhu
• A total of 40 sports and 61 disciplines will compete over Culture of ancient Chinese Civilisation.
54 competition sites in Hangzhou and five co-host cities.
• Competitions in archery, artistic swimming, boxing, ASIA CUP
breaking, hockey, modern pentathlon, sailing, tennis,
and water polo will also serve as Olympic qualifiers, with Why in News: Recently, India defeated Sri Lanka in a one-
Paris 2024 quota spots available in those nine sports. sided final to win the Asia Cup for the 8th time.
• The Mascots of the 19th Asian Games are a group of
About Asia Cup:
robots named:
¾ Chenchen • The Asia Cup is a men’s international cricket
¾ Congcong tournament contested between Asian countries in either
One Day International format (50 overs) and Twenty20
¾ Lianlaian
International cricket format (20 overs).

158 SNAP NEWS


• It was established in 1983 when the Asian Cricket
• India boycotted the 1986 tournament due to strained
Council was founded as a measure to promote goodwill
cricketing relations with Sri Lanka.
between Asian countries.
• India, with eight titles (seven ODI and one T20I), is the
• The Asia Cup is the only continental championship in cricket
most successful team in the tournament.
and the winning team becomes the champion of Asia.
• Sri Lanka has played the most Asia Cups (15) followed
• It alternates every 2-years between ODI and T20 formats.
by India, Pakistan and Bangladesh (14 each).
• The first Asia Cup was held in 1984 in Sharjah in the
United Arab Emirates.

EXPECTED QUESTION FOR UPSC CSE

Prelims MCQ

Q. With reference to the Asia Cup in cricket tournaments, consider the following statements:

1. It was started in 1983 with the foundation of the Asian Cricket Council.

2. The first tournament was held in 1984 in Sharjah in the United Arab Emirates.

3. Sri Lanka became the most successful team in the tournament by winning eight titles.

How many of the above statements are correct?

(a) Only one      (b) Only two

(c) All three       (d) None

Answer: b (1 and 2 only)

PLACES IN NEWS
Location Description
Tungareshwar Sanctuary • It serves as a crucial corridor connecting the Sanjay Gandhi National Park (also
known as Borivali National Park) and the Tansa Wildlife Sanctuary.
(Maharashtra)
• Fauna: Oriental dwarf kingfisher (a migrant bird known as the Jewel of the forest),
the Leopard, Wild Boar, Barking Deer, Langur, Bonnet and Rhesus Macaque, and
Black-naped Hare, as well as a multitude of birds, including the Crested Serpent-
eagle, Jungle Owlet, White-eyed Buzzard, Oriental Honey-buzzard, Emerald Dove
and Heart-spotted Woodpecker
• Tungareshwar Temple (Lord Shiva) is situated here.
Kostyantynivka Missile Explosion • A recent missile explosion in Kostyantynivka has been attributed to a wayward
(Eastern Ukraine) Ukrainian rocket. This incident has raised concerns and garnered attention both
nationally and internationally.
Sahel Region • Sahel is the vast region that stretches from the Atlantic Ocean to the Red Sea along
Sahara desert’s southern rim.
(Africa)
• Thus, Sahel region is wedged between Sahara Desert in the north and tropical
forests and the savannah to the south.

SNAP NEWS  159


Burkina Faso • Burkina Faso has ordered the French embassy’s defence attache to leave the
country.
(West Africa)
• Burkina Faso is a landlocked country located in western Africa. It shares its
borders with several countries, including Mali to the north and west, Niger to the
northeast, Benin to the southeast, and Côte d’Ivoire, Ghana, and Togo to the south.

Sudan • UNHCR appealed for $ 1 billion to support Sudan which is facing civil war.
(North-eastern Africa)
Dili (Timor-Leste) • India has announced the opening of an embassy in Dili, the capital of Timor-Leste.

160 SNAP NEWS


SECTION
ARTICLE FROM YOJANA &
K ­KURUKSHETRA (SEPTEMBER 2023)
• Accountability in financial administration is enforced at
Accountability and Financial three levels:
Administration [From Yojana] ¾ internal audit mechanisms by executive ministries
• Public administration in India, dating back to the 4th and departments
century BCE, is based on the ancient Arthashastra, which ¾ external audits by the CAG and
deals with politics, statecraft, and governance.
¾ independent examination of subjects by Legislative
• The institution of the Comptroller and Auditor General Committees.
(CAG) came into existence in the year 1860 and later
• The CAG plays a crucial role in promoting transparency
gained Constitutional status in 1950.
and good governance by providing timely, independent,
• The CAG plays a crucial role in public financial and credible assurance on public resources.
management, advising the government on maintaining
• CAG has a nationwide audit mandate encompassing all
accounts for Union and State Governments.
three tiers of the federal structure of governance:
• The Constitution ensures the independence of the CAG
¾ the Union Government
from the legislature and the executive.
¾ the State/UT Governments and
• The CAG has complete discretion in the discharge of
his functions, including the selection of audit issues, ¾ the local bodies thus, ensuring that accountability is
the manner of conduct and reporting of audits, and the enforced up to the last mile.
organisation and management of his office. • The audit jurisdiction includes attached & subordinate
offices, autonomous bodies, Statutory authorities and
Public Sector Undertakings (PSUs).
• The CAG Audit Reports presented to the Parliament/
State Legislatures are selected by the Legislative
Committees for detailed examination.
• The CAG functions as a friend, philosopher, and guide to
the legislative committees by suggesting topics of focus
and highlighting issues of concern.
• The committees carry out an in- depth examination of the
CAG’s Audit observations and call upon the executive
ministries and departments to explain their actions on the
irregularities pointed out in the reports.
• The CAG’s audits are of three types:
¾ financial attest audit to ascertain the accuracy and
correctness of the financial statements of the entity;
¾ compliance audit to check adherence to applicable
rules, regulations, and procedures; and
¾ performance audit to assess whether systems within
the entity operate with economy, efficiency, and
effectiveness.
• The subject matter for audits is selected through a
meticulous risk assessment process.
Significance of CAG Audit
• Audit observations are communicated to management
• The CAG’s business is conducted through the Indian
through inspection reports, separate audit reports, and
Audit & Accounts Department, which together form the
management letters, allowing for corrective action.
Supreme Audit Institution (SAI) of India.

ARTICLE FROM YOJANA & ­KURUKSHETRA (SEPTEMBER 2023)  161


• Significant audit observations are reported to Parliament • Performance audits have been conducted to assess the
and State Legislatures through CAG’s Audit Reports, Direct Benefit Transfer (DBT) system’s efficacy and
which cover Union Government Appropriation and efficiency, focusing on correct identification, coverage,
Finance Accounts, State Finances Audit Reports, and timely payments.
General Purpose Financial Reports, and Compliance
and Performance Audit Reports. • The audit of various Government schemes, including
MNREGA, PM Awaas Yojana, DDLIGJY, SAUBHAGYA,
• These reports also provide macro fiscal analysis, critical AIBP, and NRHM, focuses on financial management,
changes in major fiscal aggregates, trends, fiscal highlighting systemic issues and irregularities.
sustainability, debt profile, and key Public Account
transactions. • The Audit observations and recommendations have
• They also contain practical, constructive, and action- provided valuable insights for the Government in steering
oriented recommendations to enhance financial positive change through course corrections, changes in
accountability and prudence. implementation design, and removal of bottlenecks.
• Audit efforts are crucial for good governance, as they
Conclusion
lead to policy changes, design improvements, mid-course
corrections, and strengthening systems.
• Addressing emerging challenges in today’s world requires
• Audit reports on tax receipts reveal instances of under- a holistic approach, considering existing inequalities,
assessment of taxes, non-recovery of receivables, and unique systems, the digital divide, and sustainability.
losses that require recovery.
• Based on an audit of receipts during 2021 -22, the • The CAG’s institution upholds the trust of taxpayers
Union and State Governments accepted that recoveries and investors in Indian enterprises and contributes
of Rs 25,571 crore needed to be made. The Income Tax constructively to establishing a robust financial
Department carried out recoveries of Rs 415 crore in the administration.
last three years to rectify errors in assessment of taxes • The Strategic Plan of the CAG’s institution for 2023-
pointed out by audit. 2030 envisions strengthening the linkage between the
• CAG’s audit of receipts has also helped ensure public Accounts and Audit verticals by building data analytics
funds are realized in accordance with prevailing laws. capacities for financial information analysis.
• During audits of search and seizure assessments, the audit • Fundamental beliefs in the institutional values,
raised observations about the absence of provisions in the professionalism, competence, social awareness, and
Income Tax Act, non-compliance, non-centralization of commitment to provide credible assurance on the
search assesses, and non-levy of penalties. collection and utilisation of public resources are the key
• In response, the Government made amendments to the components that support the legacy of the prestigious
Income Tax Act, including a new Section 79A preventing institution of the CAG of India as a torchbearer of good
set off of losses on undisclosed income and Sub-Section governance.
(1A) in Section 149 allowing taxable income from a
previous period to be assessed under the Act.
Reforms in the institution of CAG Make in India: Challenges,
• India’s Public Financial Administration has undergone Opportunities and Outcomes
several reforms, including the adoption of an Integrated [From Kurukshetra]
Financial Management System (IFMS) and automation
of governance mechanisms. About Make in India
¾ This has increased transparency and improved • It aimed at encouraging companies and individuals
accountability in financial administration. across the globe to facilitate investment, foster innovation,
build world-class infrastructure, and build a hub for
• The CAG’s institution has integrated accounting software
manufacturing, design, and innovation in India.
with the IFMS, improving efficiency and accuracy.
• It is promoting India’s manufacturing domain to
• A data-based approach to auditing has facilitated faster
transform the nation into a global manufacturing
delivery and improved outcomes.
centre.
• An institutional framework for digital GST revenues has
• It promoted labour- and capital-intensive industries with
been established.
different stance as well as tried to bring in timely and
• Data analytics tools have helped identify outliers and adequate research and development into manufacturing
strengthen controls in financial administration and firms and modern services.
governance mechanisms.

162 ARTICLE FROM YOJANA & K


­ URUKSHETRA (SEPTEMBER 2023)
Scope and Sectoral Coverage

• A total of 27 economic sectors were identified to give


push during the entire life of business activities within
the respective sector.

¾ 15- Related to manufacturing sectors (Action Plan-


The Department of Promotion of Industry and
Internal Trade)

¾ 12- Service Sectors (Action Plan- Department of


Commerce)

• Outreach activities are carried out through Ministries,


State Government, and Indian Missions abroad for
enhancing international cooperation and promoting
domestic and foreign investment in the country.

Pillars of Make in India

ARTICLE FROM YOJANA & ­KURUKSHETRA (SEPTEMBER 2023)  163


Attracting FDI into India h) Phased manufacturing programmes
a) 
The Government has taken various steps to boost
Advantages of Make in India
domestic and foreign investment in India, such as,
The main goal of Make in India is to generate and offer
b) Introduction of Goods and Services Tax jobs for everyone, particularly for the nation’s younger
c) Reduction in Corporate Tax generation.

d) Innovation to improve EoDB The people that live close to these places will have a better
financial situation.
e) FDI policy reforms
It helps Indian industries grow, which will increase the
f) Measures for reduction in compliance burden Country’s income.
g) 
Policy measures to boost domestic manufacturing It stimulates the development of new technology and
through public procurement orders strengthening the nation’s labor force’s abilities in India.

• The complicated labour regulations for units with more


The holistic developmental approach under ‘make in India’
than 100 employees.
initiative will help in achieving higher credit ratings for the
country, and making India global manufacturing hub so as to • India’s bureaucratic procedures and corruption within
the government makes India far less attractive for
attract more and more investors to invest in India.
investors.
Challenges • The size of the industrial units is small. Therefore, it
• The cost and complexity of land acquisition make it a cannot attain the desired economies of scale.
very expensive and time-consuming process. A clear and • India’s manufacturing sector’s productivity is low and
strong land acquisition policy is necessary along with the skills of the labour force are insufficient.
appealing R&R packages. • The cost of electricity is almost the same in India and
China. However, the outages are far higher in India.

164 ARTICLE FROM YOJANA & K


­ URUKSHETRA (SEPTEMBER 2023)
Fostering Balanced Regional Development • It will help to resolve the issues of economic difference
The ‘Make in India’ initiative has a special approach on and regional imbalance.
development decentralization. Its important manifestation The ODOP initiative will provides hand holding support
vision is ‘One-District One-Product’ initiative. to the local economy to improve backward and forward
linkages for the selected products. The processes within the
‘One-District One-Product’ initiative would actively contribute to the economic growth
• The focusing of one product in each district will help to of a rural region and the country as a whole.
preserve and develop unique local agri produce, local
crafts, traditional art with increased income. Impacts of Make in India
• It will increase in the incomes and local employment The initiative’s prime objectives have been to strengthen
(resulting in decline in migration for employment). manufacturing sectors and expand the scope of growth,
• It will lead to improvement in product quality and skill employment, income, and contribution in the economy from
development. manufacturing sector.
• It will transform the products in an artistic way (through
The initiative has had a positive impact on the economy. The
packaging, branding).
following are some of the major achievements of the last 8
• It will help to connect the production with tourism. years:

Concluding Remarks
The Make in India aims at sustainable economic growth. This
ambitious initiative has the potential to make India a centre
of manufacturing and creation of world-class infrastructure
across the country. This initiative turned post-Covid situations
into an opportunity for growth via the implementation of
Atmanirbhar packages, introduction of PLI scheme.
With the aim of achieving sustainable growth with distributive
justice, the initiative has attempted its best to make India a
global economic hub. Several steps were taken to encourage
investment by improving the business environment in India
through feasible reform measures.
While MSMEs, the services sector and startups, are going to
play a critical role in the success of Make in India. There is
lot to do through this initiative to ensure balanced regional
growth and address issues of poverty, unemployment, and
disparities of income and wealth.

ARTICLE FROM YOJANA & ­KURUKSHETRA (SEPTEMBER 2023)  165


SECTION

L DRAINAGE SYSTEMS OF INDIA

The Indus River System • It flows between the Ladakh and the Zaskar Ranges in
India.
Origin - Near Bokhar Chu in the Tibetan region at an altitude
of 4,164 m in the Kailash Mountain range. • The river empties into the Arabian Sea south of Karachi
after forming a huge delta.
Features:
• One of the largest river basins of the world. Tributaries
• Westernmost of all Himalayan rivers in India.
• Right-bank tributaries: Shyok, Gilgit, Hunza, Nubra,
• Known as ‘Singi Khamban; or Lion’s mouth in Tibet.
Kabul, Khurram, Tochi, Gomal, Sangar, Kunar
Course: • Left-bank tributaries: Zanskar, Suru, Soan, Jhelum,
• The Indus River flows north-west from its source. Chenab, Ravi, Beas, Satluj rivers.

Some Major Tributaries of Indus

Shyok River Origin: From Rimo Glacier of Karakoram Range


• Flows through Northern Ladakh region

Nubra River Origin: from the Nubra Glacier


• Meanders towards southeast to join Shyok River at the base of Ladakh range
• Nubra Valley- 3048m - formed out of the Nubra River

Chenab River Origin: near the Bara Lacha Pass in Lahaul-Spiti of Zanskar Range.
• Formed by the confluence of Chandra and Bhaga rivers at Tandi (Lahaul and Spiti District of
Himachal Pradesh)
• Upper reaches k/a Chandrabhaga
• Flows through Jammu region into Punjab plains in Pakistan
• Water allocated to Pakistan under Indus Water Treaty
• Baglihar Dam on this river
• World’s highest railway bridge k/a Chenab Bridge is on this river.

Jhelum River Origin: from a spring at Verinag at the foot of Pir Panjal
• Largest tributary- Kishenganga (Neelum).
• Waters allocated to Pakistan under Indus Water Treaty
• Ends in a confluence with Chenab in Pakistan.

Kishanganga River Origin: Drass in Kargil district of JandK


• Enters Pakistan from India near Line of Control and then runs west till it meets Jhelum
• Also known as Neelam either due to its sky cold water or due to the precious stone “Neelam”
found in this area
• Famous for - trout fish.

166 DRAINAGE SYSTEMS OF INDIA


Ravi River Origin: Dhauladhar range in Chamba district of HP.
• Source in Kullu hills near Rohtang Pass in Himachal Pradesh.
• Follows a northwesterly course.
• Waters allocated to India under Indus Waters Treaty
• Ranjit Sagar Dam (a.k.a Thein dam) on this river
Sutlej River Origin: From Rakas Lake in Southern slopes of Kailash Mountain near Mansarover Lake.
• a.k.a Red River.
• Enters HP at Shipki La andflows in S-W direction through Kinnaur, Shimla, Kullu, Solan,
Mandi, andBilaspur districts.
• World’s highest gravity dam- Bhakra Nangal Dam, on this river.
• Waters allocated to India under Indus Water Treaty
Beas River Origin: Rohtang pass in HP
• Before entering Pakistan merges with Sutlej River at Hari-Ke-Pattan (Punjab)
• Length - 460km - covers 256km through HP

Ganga River System Ganga – Brahmaputra Delta

Origin: The Ganga originates as the Bhagirathi from the • Before entering Bay of Bengal, Ganga + Brahmaputra,
Gangotri glacier. forms the largest delta of world ● b/w Bhagirathi/
• Before it reaches Devprayag in the Garhwal Division, the Hugli and Padma/Meghna.
Mandakini, Pindar, Dhauliganga and Bishenganga rivers
• Area - 58,752 sq km.
merge into the Alaknanda and the Bheling drain into the
Bhagirathi.
• Coastline highly indented
• The Pindar River rises from East Trishul and Nanda
Devi unites with the Alaknanda at Karan Prayag. The • Major part is a low-lying swamp flooded by marine
Mandakini meets at Rudraprayag. water during high tide.
• The water from both Bhagirathi and the Alaknanda flows
in the name of the Ganga at Devprayag.
Major Left bank tributaries of Ganga
Concept of Panch Prayag
• Vishnuprayag: where the river Alaknanda meets river Ramganga Origin: southern slopes of
Dhauli Ganga Dudhatoli Hill in Chamoli,
• Nandprayag: where river Alaknanda meets river Uttarakhand.
Nandakini • Meets the Ganga near
• Karnaprayag: where river Alaknanda meets river Pinder Kannauj.
• Rudraprayag: where river Alaknanda meets river
Mandakini
Gomti Origin: Gomat Taal (aka
• Devprayag: where river Alaknanda meets river
Bhagirathi -GANGA Fulhaar jheel) near Madho
Tanda, Pilibhit, UP.
Major Tributaries:
• Meets the Ganges in
• Right Bank Tributaries: Yamuna River, Chambal River, Ghazipur.
Banas River, Sind River, Betwa River, Ken River, Son
River, Damodar River ¾ Location of famous
Markandey Mahadeo
• Left Bank Tributaries: Ramganga River, Gomti River,
temple.
Ghaghra River, Kali River, Gandak River, Burhi Gandak,
Kosi River

DRAINAGE SYSTEMS OF INDIA  167


Ghaghara (Karnali or Origin: in the glaciers of Major Right bank tributaries of Ganga
Kauriala) Mapchachungo.
Son Origin: near Amarkantak in MP, east of
• Source: near Gurla headwater of the Narmada River
Mandhata peak, south of
Mansarovar in Tibet • Lower valley of Son - extension of
Narmada Valley.
• Major left-bank tributary
of Ganga (confluence at Rihand Origin: from Matiranga Hills (southwest of
Chhapra in Bihar). the Mainpat plateau)
• Joins Son at Sonbhadra, UP.
Kali (Sarda) Origin: Rises in the high glaciers
• Rihand Dam across Rihand River for
of trans-Himalaya.
hydropower generation
• Forms the boundary b/w ¾ Reservoir of dam k/a Govind
Nepal and Kumaon, India. Ballabh Pant Sagar
Saryu/ Sarju Origin: at a ridge south of Nanda North Koel Origin: Rises on the Ranchi plateau and
Kot Mountain, Bageshwar enters Palamu division below Netarhat near
district in Uttarakhand Rud

• Left-bank tributary of • Meanders through the northern part of


Ghaghara Betla National Park
Yamuna Origin: Yamunotri glacier on Bandarpunch
Rapti Origin: Rises south of a Peak in the Garhwal region in Uttarakhand
prominent E-W ridgeline (6,000 meters).
midway b/w western
• Largest and the most important tributary.
Dhaulagiri and Mahabharat
Range in Nepal
• Fed by underground water Yamuna River System
• Recurrent floods - so,
known as “Gorakhpur’s Origin: Yamunotri Glacier on southwestern slopes or
Sorrow” Bandarpoonch peak in Mussoorie range of lower Himalayas.

• States: Uttarakhand, Himachal Pradesh, Haryana enters


Gandak Origin: Union of Kali and Delhi.
Trisuli rivers
• Merges with the Ganga near Triveni Sangam, Allahabad
• Merges with Ganga River (Prayagraj)
opposite Patna at Sonepur. • Largest tributary of the Ganga in northern plains.
• Burhi Gandak flows parallel
• Major tributary: Tons (rises from the Bandarpunch
to and east of Gandak River
glacier).
Burhi Gandak Origin: from Chautarva Chaur
• Total length till Allahabad: 1,376 km.
near Bisambharpur, West
Champaran, Bihar • Highly fertile alluvial, Yamuna-Ganges Doab region.

• Flows parallel to and east Major tributaries of Yamuna River


of Gandak River in an old
channel. Tons • Longest tributary of Yamuna
• Flows through Garhwal
Kosi [Saptakoshi (has 7 • Antecedent transboundary
Himalayan tributaries)] river flowing through Nepal • Originates at an elevation of 3900 m.
and India • Joins Yamuna below Kalsi near
• One of largest tributaries Dehradun, Uttarakhand.
of Ganga (Confluence at Giri River • Joins Yamuna upstream of Paonta
Kursela in Kathiar district) below Mokkampur.
• Unstable nature - “The • Main source of water in South-
Sorrow of Bihar” Eastern Himachal Pradesh.

168 DRAINAGE SYSTEMS OF INDIA


Hindon River • Sandwiched b/w Ganga on the • Turns into south and creates a deep gorge through
left and Yamuna on the right. ● Himalayas near Namcha Barwa.
Originates from upper Shiwalik • Enters India under the name of the Dihang.
(Lower Himalayas). • Sadia - receives Dibang coming from north and Lohit
• A purely rain fed river coming from east.
Betwa River • Originates at an elevation of 470 m • Enters Assam valley where it is k/a Brahmaputra.
in Bhopal, Madhya Pradesh. • Flows west up to Dhubri and further below, runs to
• Joins Yamuna River near south and enters Bangladesh.
Hamirpur. • Braided channel.
• Basin is saucer shaped with • Left bank tributaries: Lhasa River, Nyang River, Parlung
sandstone hills around the Zangbo, Lohit River, Dhansiri River, Kolong River
perimeter. • Right bank tributaries: Kameng River, Manas River,
Ken River • An inter-state river (Madhya Beki River, Raidak River, Jaldhaka River, Teesta River,
Pradesh and Uttar Pradesh). Subansiri River
• Originates near village Ahirgawab
in Jabalpur, Madhya Pradesh.
Godavari River System
• Joins Yamuna River, near Chilla
village of U.P. • Second-longest course in India with brownish water.
Sind River • Rises in Vidisha District of Madhya • Also known as Dakshin (South) Ganga or Vriddh (Old)
Pradesh. Ganga.
• Flows generally in north- east • Seasonal - dried during summers, and widens during
direction. monsoons.
• Probably river Sindhu mentioned • Origin: Trimbakeshwar, near Nasik in Maharashtra.
in epic Vishnu Purana. • Flows southeast across south-central India.
Chambal • k/a Charmanvati in ancient time • States: Madhya Pradesh, Telangana, Andhra Pradesh,
• Largest river flowing through and Odisha.
Rajasthan state. • Drains into the Bay of Bengal.
• Rises in Vindhyan range near • Forms a fertile delta at Rajahmundry.
Mhow in Indore District of Madhya
• Asia’s largest rail-cum-road bridge which links Kovvur
Pradesh.
and Rajahmundry on it.
• Major dams
• Left bank tributaries: Banganga, Kadva, Shivana, Purna,
¾ Gandhi Sagar - boundary Kadam, Pranahita, Indravati, Taliperu, Sabari
of Madhya Pradesh and
• Right bank tributaries: Nasardi, Pravara, Sindphana,
Rajasthan
Manjira, Manair, Kinnerasani
¾ Rana Pratap Sagar - Rawat
Bhata (Rajasthan) .
¾ Jawahar Sagar Dam. Krishna River System
¾ Kota Barrage • Second-largest east-flowing peninsular river.
• Origin: Mahabaleshwar.
• States: Maharashtra, Karnataka, Telangana, Andhra
Brahmaputra River System Pradesh
• One of the largest rivers in the world. • Causes heavy soil erosion during monsoon.
• In Tibet, it is known as Yarlung Tsangpo. • Bounded by Balaghat range (north), Eastern Ghats
• Enters by the names of Siang and Dihang in India. (south and the East), and Western Ghats (west).
• Source of origin: Chemayundung glacier of Kailash • Total length- 1,400 km.
range near Mansarovar Lake. • Drains into Bay of Bengal.
• Mouth: Drains into Bay of Bengal before forming a huge • Major dams- Almatti Dam, Srisailam Dam, Nagarjuna
delta along with the Ganga. Sagar Dam, and Prakasam Barrage.
• Course: • Fed by seasonal monsoon rains.
• Tibet- runs parallel to Himalayas for about 1,200 km. • Right bank tributaries: Venna, Koyna, Panchganga,

DRAINAGE SYSTEMS OF INDIA  169


Dudhganga, Ghataprabha, Malaprabha and Tungabhadra • In terms of water potential and flood producing capacity,
• Left Bank tributaries: Bhima, Dindi, Peddavagu, Halia, it ranks second to the Godavari.
Musi, Paleru, and Munneru • Major part covered with agricultural land- 54.27% of total
area.
• One of the most-active silt-depositing streams in the
Narmada River System Indian subcontinent.
• Largest (1312km.) west flowing river of peninsular India. • Drains into Bay of Bengal
• Largest Rift valley river in India. • Left bank Tributaries: Seonath, Hasdeo, Mand, and Ib
• Largest Estuary River in India. • Right bank Tributaries: Ong, Tel, and Jonk
• Flows westwards through a rift valley b/w Vindhya
Range (north) and Satpura Range (south).
• Rises from Maikala range near Amarkantak in Madhya
Cauvery River System
Pradesh.
• Aka ‘Dakshin Bharat ki Ganga’ or ‘the Ganga of the
• States- Gujarat, Maharashtra and Madhya Pradesh South’.
• Waterfalls: Kapildhara, Dugdhadhara , Dhuadhaar, • Origin: Talakaveri, Brahmagiri range, Cherangala village,
Mandhar, Dardi, Shahatradhara Coorg, Karnataka.
• Left Bank tributaries: Burhner, Banjar, Sher, Shakkar, • Length: 800 km.
Dudhi, Tawa, Ganjal, Kundi, Goi, Karjan
• States- Tamil Nadu, Karnataka, Kerala and Puducherry
• Right bank tributaries: Hiran, Tendoni, Barna, Kolar,
• Breaks into a large number of distributaries forming a
Man, Uri, Hatni, Orsang
wide delta k/a “garden of southern India”
• Drains into the Bay of Bengal.
Mahanadi River System • Left Bank tributaries: Harangi, Hemavati, Shimsha, and
• 3rd largest of peninsular India. Arkavati
• Largest river of Odisha. • Right Bank tributaries: Lakshmantirtha, Kabbani,
Suvarnavati, Bhavani, Noyil, and Amaravati
• States- Chhattisgarh, Odisha, Jharkhand, Maharashtra,
and Madhya Pradesh • Descends from South Karnataka Plateau to Tamil Nadu
Plains through Shivasamudram waterfalls (101 m high).
• Source - northern foothills of Dandakaranya, Raipur,
Chhattisgarh

170 DRAINAGE SYSTEMS OF INDIA

You might also like